Está en la página 1de 244

Introduccin

Este libro est dirigido a estudiantes de las Olimpiadas de Matemticas que


a
a
quieren prepararse en el estudio de desigualdades, tema ahora frecuente en los
concursos de matemticas de distintos niveles. En este volumen se presentan
a
las desigualdades clsicas y desde luego las ms utiles para enfrentar y resolver
a
a
problemas de optimizacin.
o
Para la presentacin de los temas, el libro se ha dividido en cuatro cap
o
tulos.
El cap
tulo 1 de desigualdades numricas contiene las desigualdades bsicas. La
e
a
mayor de ellas son desigualdades numricas que en general carecen de una ina
e
terpretacin geomtrica, sin embargo cuando es posible darla sta se incluye. Se
o
e
e
hace notar la importancia de algunas de ellas, por ejemplo, la desigualdad entre
la media geomtrica y la media aritmtica, la desigualdad de Cauchy-Schwarz, la
e
e
desigualdad del reacomodo, la desigualdad de Jensen, la desigualdad de Muirhead, entre otras. Para todas ellas, adems de su justicacin, se presentan
a
o
varios ejemplos que muestran cmo utilizarlas en problemas del tipo de olimpiao
das de matemticas. Tambin se hace notar cmo la estrategia de sustitucin
a
e
o
o
se utiliza para deducir varias desigualdades.
En el cap
tulo 2 de desigualdades geomtricas, se ejemplica el uso de las desie
gualdades numricas bsicas del cap
e
a
tulo 1 para resolver problemas geomtricos.
e
Se trabaja tambin, en esta parte, con desigualdades que tienen un fuerte contee
nido geomtrico, iniciando con hechos bsicos como la desigualdad del tringulo
e
a
a
y la desigualdad de Euler. Introducimos ejemplos donde el carcter simtrico en
a
e
las variables ayuda a resolver algunos problemas, se hace tambin nfasis en
e e
cmo los cambios de variable se utilizan para deducir varias de ellas. Resalo
tamos entre stos la transformacin de Ravi y la correspondencia entre una
e
o
desiguladad en trminos de las longitudes de los lados de un tringulo a, b, c y
e
a
las desigualdades correspondientes en trminos de s, r y R, el semiper
e
metro, el

II

Introduccin
o

inradio y el circunradio del tringulo, respectivamente. Se incluyen tambin vaa


e
rios problemas geomtricos clsicos haciendo hincapi en los mtodos utilizados
e
a
e
e
para resolverlos.
En el cap
tulo 3 se presentan ciento veinte problemas de desigualdades que han
aparecido en concursos recientes, cubriendo todos los niveles desde olimpiadas
nacionales, regionales hasta competencias internacionales.
En el cap
tulo 4, se dan las soluciones a cada uno de los doscientos diez ejercicios
de los cap
tulos 1 y 2, as como a los problemas presentados en el cap

tulo 3.
La mayor de las soluciones de los ejercicios o problemas que han aparecido
a
en competencias internacionales de matemticas se extrajeron de las soluciones
a
ociales de cada uno de los concursos. Esta es la razn por la cual no damos
o
crditos individuales por ellas.
e
Una gran parte de los ejercicios y problemas de desigualdades se pueden resolver
utilizando distintas tcnicas, es por ello que encontrar algunos ejercicios repee
a
tidos, pero en diferentes secciones. Esto le indicar que puede encontrar, con
a
la tcnica desarrollada en la seccin correspondiente, una manera de resolver el
e
o
ejercicio utilizando dicha herramienta.
El material presentado en este libro ha sido acumulado durante los ultimos

quince aos, principalmente durante las sesiones de trabajo con estudiantes que
n
han ganado el concurso nacional de la Olimpiada Mexicana de Matemticas;
a
estos estudiantes se estaban preparando para las competencias internacionales
en las que Mxico participa.
e
Quisieramos agradecer a Rafael Mart
nez Enr
quez, Leonardo Ignacio Mart
nez
Sandoval, David Mireles Morales, Jess Rodr
u
guez Viorato y Pablo Sobern
o
Bravo por su cuidadosa revisin del texto y sus valiosos comentarios, lo cual
o
permiti mejorar la presentacin del libro.
o
o
Radmila Bulajich Manfrino
Jos Antonio Gmez Ortega
e
o
Rogelio Valdez Delgado

Contenido

Introduccin
o
1. Desigualdades Numricas
e
1.1. El orden en los nmeros reales . . .
u
1.2. La funcin cuadrtica ax2 + 2bx + c.
o
a
1.3. Una desigualdad fundamental,
media geomtrica-media aritmtica .
e
e
1.4. Una desigualdad maravillosa,
la desigualdad del reacomodo . . . .
1.5. Funciones convexas . . . . . . . . .
1.6. Una desigualdad util . . . . . . . . .

1.7. La estrategia de sustitucin . . . . .


o
1.8. Teorema de Muirhead . . . . . . . .

III

. . . . . . . . . . . . . . .
. . . . . . . . . . . . . . .

1
1
5

. . . . . . . . . . . . . . .

.
.
.
.
.

.
.
.
.
.

.
.
.
.
.

.
.
.
.
.

.
.
.
.
.

.
.
.
.
.

.
.
.
.
.

.
.
.
.
.

2. Desigualdades Geomtricas
e
2.1. Dos desigualdades bsicas . . . . . . . . . . . . . .
a
2.2. Desigualdades entre los lados de un tringulo . . .
a
2.3. Uso de desigualdades en la geometr del tringulo
a
a
2.4. La desigualdad de Euler y algunas aplicaciones . . .
2.5. Funciones simtricas de a, b y c . . . . . . . . . . .
e
2.6. Desigualdades con reas y per
a
metros . . . . . . .
2.7. Teorema de Erds-Mordell . . . . . . . . . . . . .
o
2.8. Problemas de optimizacin . . . . . . . . . . . . .
o

.
.
.
.
.
.
.
.

.
.
.
.
.
.
.
.

.
.
.
.
.
.
.
.

.
.
.
.
.
.
.
.

.
.
.
.
.
.
.
.

.
.
.
.
.
.
.
.

59
. 59
. 63
. 68
. 77
. 82
. 87
. 93
. 102

3. Problemas Recientes de Desigualdades

.
.
.
.
.

.
.
.
.
.

.
.
.
.
.

.
.
.
.
.

.
.
.
.
.

.
.
.
.
.

.
.
.
.
.

15
24
38
46
51

115

Contenido

IV

4. Soluciones a los Ejercicios y Problemas


135
4.1. Soluciones a los ejercicios del cap
tulo 1 . . . . . . . . . . . . . 135
4.2. Soluciones a los ejercicios del cap
tulo 2 . . . . . . . . . . . . . 161
4.3. Soluciones a los problemas del cap
tulo 3 . . . . . . . . . . . . . 187
Notacin
o

235

Bibliograf
a

237

Indice

239

Cap
tulo 1

Desigualdades Numricas
e

1.1.

El orden en los nmeros reales


u

Los nmeros reales tienen la importante propiedad de poseer un orden. El orden


u
en los nmeros reales nos permitir comparar dos nmeros y decidir cual de ellos
u
a
u
es mayor o bien si son iguales. A n de evitar justicaciones tediosas, asumiremos
que en los nmeros reales hay un conjunto P que llamaremos el conjunto de
u
nmeros positivos, y simblicamente escribiremos x > 0, para decir que un
u
o
nmero x est en P . Aceptaremos tambin las tres propiedades siguientes.
u
a
e
Propiedad 1.1.1 Cada nmero real x tiene una y slo una de las siguientes
u
o
caracter
sticas:
(i) x = 0.
(ii) x P (esto es x > 0).
(iii) x P (esto es x > 0).
Propiedad 1.1.2 Si x,y P , entonces x + y P
(en s
mbolos x > 0, y > 0 x + y > 0).
Propiedad 1.1.3 Si x, y P , entonces xy P
(en s
mbolos x > 0, y > 0 xy > 0).
Si tenemos a la recta real como representacin geomtrica de los nmeros
o
e
u
reales, es decir, una recta dirigida donde se ha localizado el cero 0, el cual

Desigualdades Numricas
e

divide en dos partes a la recta, los nmeros positivos son la parte que contiene
u
al uno 1. Por lo general el 1 est colocado a la derecha de 0 en el sentido
a
que tiene la recta. Hacemos notar que el 1 es positivo, ya que si fuera negativo,
como cumple con la propiedad 1 x = x para cada x, tendr
amos que cualquier
nmero x = 0 cumplir con x P y x P , lo cual contradice la propiedad
u
a
1.1.1.
Ahora podemos denir la relacin, a es mayor que b, si a b P (en s
o
mbolos
a > b). Anlogamente, a es menor que b, si b a P (en s
a
mbolos a < b).
Observemos que a < b es equivalente a b > a. Denimos tambin a es menor
e
o igual que b, si a < b a = b, (en s
o
mbolos a b).
Denotamos a los nmeros reales por R y al conjunto P de nmeros reales
u
u
positivos por R+ .
Ejemplo 1.1.4 (i) Si a < b y c es cualquier nmero, entonces a + c < b + c.
u
(ii) Si a < b y c > 0, entonces ac < bc.
En efecto, para mostrar (i) tenemos que a + c < b + c (b + c) (a + c) >
0 b a > 0 a < b. Para ver (ii), tenemos que: a < b b a > 0 y como
c > 0, resulta que (b a)c > 0, luego bc ac > 0 y entonces ac < bc.
Ejercicio 1.1 Dados dos nmeros a y b, una y slo una de las siguientes
u
o
armaciones se cumple, a = b, a > b o a < b.
Ejercicio 1.2 Muestre las siguientes armaciones:
(i) a < 0, b < 0 ab > 0.
(ii) a < 0, b > 0 ab < 0.
(iii) a < b, b < c a < c.
(iv) a < b, c < d a + c < b + d.
(v) a < b b < a.
1
(vi) a > 0 > 0.
a
1
(vii) a < 0 < 0.
a
a
(viii) a > 0, b > 0 > 0.
b
(ix) 0 < a < b, 0 < c < d ac < bd.
(x) a > 1 a2 > a.
(xi) 0 < a < 1 a2 < a.

1.1 El orden en los nmeros reales


u

Ejercicio 1.3 (i) Si a > 0, b > 0 y a2 < b2 , entonces a < b. (ii) Si b > 0,
o
tenemos que a > 1 si y slo si a > b.
b
Para un nmero real x se dene el valor absoluto, el cual se denota por |x|,
u
como
|x| =

x
x

para x 0
para x < 0.

Geomtricamente, |x| es la distancia del nmero x (en la recta real) al origen


e
u
0. Tambien, |a b| es la distancia entre los nmeros reales a y b en la recta
u
real.
Ejercicio 1.4 Para cualesquiera nmeros reales x, a y b, se tiene que:
u
(i) |x| 0, y es igual a cero solamente cuando x = 0.
(ii) |x| = |x|.
(iii) |x|2 = x2 .
(iv) |ab| = |a| |b|.
|a|
a
, con b = 0.
=
(v)
b
|b|

Proposicin 1.1.5 (Desigualdad del tringulo) Para dos nmeros reales a y


o
a
u
b, siempre se tiene que
|a + b| |a| + |b| .
Adems la igualdad ocurre solamente cuando ab 0.
a
Demostracin. Como ambos lados de la desigualdad son nmeros positivos;
o
u
2
por el ejercicio 1.3 bastar entonces vericar que |a + b| (|a| + |b|)2 .
a
|a + b|2 = (a + b)2 = a2 + 2ab + b2 = |a|2 + 2ab + |b|2 |a|2 + 2 |ab| + |b|2
= |a|2 + 2 |a| |b| + |b|2 = (|a| + |b|)2 .

En las relaciones anteriores hay una sola desigualdad y sta es inmediata ya


e
que ab |ab|. Notemos que cuando ab 0 se tiene que ab = |ab| = |a| |b|, y
entonces se da la igualdad.
La forma general de la desigualdad del tringulo, para nmeros reales x1 ,
a
u
x2 , ... , xn , es
|x1 + x2 + . . . + xn | |x1 | + |x2 | + . . . + |xn |.

Desigualdades Numricas
e

La igualdad se tiene cuando todos los xi tienen el mismo signo. Esta se demuestra de manera similar, o bien por induccin. Otra versin de la desigualdad
o
o
anterior que su usa muy a menudo es
|x1 x2 . . . xn | |x1 | + |x2 | + . . . + |xn |.
Ejercicio 1.5 Sean x, y, a, b nmeros reales muestre que:
u
(i) |x| b b x b.
(ii) ||a| |b|| |a b|.
(iii) x2 + xy + y 2 0.
(iv) x > 0, y > 0 x2 xy + y 2 > 0.
Ejercicio 1.6 Para nmeros reales a, b, c muestre que
u
|a| + |b| + |c| |a + b| |b + c| |c + a| + |a + b + c| 0.

Ejercicio 1.7 Sean a, b nmeros reales con 0 a b 1, muestre que:


u
ba
1,
(i) 0
1 ab
a
b
(ii) 0
+
1,
1+b 1+a
1
(iii) 0 ab2 ba2 .
4
Ejercicio 1.8 Muestre que si n, m son nmeros enteros positivos, entonces
u

m+2n
m
2 si y slo si 2 < m+n .
o
n <
Ejercicio 1.9 Si a b, x y, entonces ax + by ay + bx.
Ejercicio 1.10 Si x, y > 0, entonces

x2
y

y2
x

x + y.

Ejercicio 1.11 (Repblicas Checa y Eslovaca, 2004) Sean a, b, c, d R con


u
a + d = b + c, muestre que
(a b)(c d) + (a c)(b d) + (d a)(b c) 0.

1.2 La funcin cuadrtica ax2 + 2bx + c.


o
a

Ejercicio 1.12 Sea f (a, b, c, d) = (a b)2 + (b c)2 + (c d)2 + (d a)2 .


Para a < b < c < d, muestre que
f (a, c, b, d) > f (a, b, c, d) > f (a, b, d, c).

Ejercicio 1.13 (IMO, 1960) Para qu valores de x se cumple la desigualdad


e
4x2

< 2x + 9?
(1 1 + 2x)2
Ejercicio 1.14 Muestre que para cualquier entero positivo, la parte fraccionaria

de 4n2 + n es menor que 1 .


4
Ejercicio 1.15 (Lista corta IMO, 1996). Sean a, b, c nmeros positivos que
u
satisfacen abc = 1, muestre que
a5

1.2.

ab
bc
ca
+ 5
+ 5
1.
5 + ab
5 + bc
+b
b +c
c + a5 + ca

La funcin cuadrtica ax2 + 2bx + c.


o
a

Una desigualdad muy util en los nmeros reales es x2 0, la cual es vlida para

u
a
cualquier nmero real x (basta ver las propiedades 1.1.1 y 1.1.3 y el ejercicio
u
1.2 de la seccin anterior). De sta se deducen muchas otras desigualdades,
o
e
en particular la usaremos para maximizar o minimizar una funcin cuadrtica
o
a
ax2 +2bx+c. Estas funciones cuadrticas aparecen con frecuencia en problemas
a
de optimizacin o en desigualdades.
o
Un ejemplo comn consiste en probar que si a > 0, la funcin cuadrtica
u
o
a
b2
b
2 + 2bx + c tendr un m
nimo es c a . En
ax
a
nimo en x = a y el valor m
efecto,
b2
b
ax2 + 2bx + c = a x2 + 2 x + 2
a
a
b
= a x+
a

+c

+c
b2
,
a

b2
a

Desigualdades Numricas
e
2

b
como x + a 0 y como el valor m
nimo de esta ultima expresin es cero

o
2
b
cuando x = a , tenemos que el valor m
nimo de la cuadrtica es c ba .
a
b
Si a < 0, la funcin cuadrtica ax2 + 2bx + c, tendr un mximo en x = a y
o
a
a
a

su valor ah es c

b2
2
a . En efecto, como ax
2
b
0 (ya que a < 0),
a

+ 2bx + c = a x +

b 2
a

+c

b2
a

el valor ms grande de esta ultima


a

puesto que a x +
2
expresin es cero, luego la funcin cuadrtica siempre es menor o igual a c ba
o
o
a
b
y toma ese valor en x = a .
Ejemplo 1.2.1 Si x, y son nmeros positivos con x + y = 2a, el producto xy
u
es mximo cuando x = y = a.
a
Si x + y = 2a, entonces y = 2a x. Por lo que, xy = x(2a x) = x2 + 2ax =
(x a)2 + a2 tiene un valor mximo cuando x = a y entonces y = x = a.
a
Esto se puede interpretar geomtricamente como: de los rectngulos de per
e
a
metro jo, el de mayor rea es el cuadrado. Ya que si x, y son los lados del
a
rectngulo, el per
a
metro es 2(x + y) = 4a y su rea es xy, que es mxima
a
a
cuando x = y = a.
Ejemplo 1.2.2 Si x, y son nmeros positivos con xy = 1, la suma x + y es
u
m
nima cuando x = y = 1.
1
x.

1
x

+ 2, luego,

1
x + y es m
nimo cuando x x = 0, esto es, cuando x = 1. Por lo tanto,
x = y = 1.
Tambin podemos interpretar geomtricamente esto de la siguiente forma, de
e
e
los rectngulos de rea 1, el cuadrado es el de menor per
a
a
metro. En efecto, si
x, y son los lados del rectngulo, tenemos que su rea es xy = 1 y su per
a
a
metro
2

1
1
a
metro
es 2(x + y) = 2 x + x = 2
x x + 2 4. Adems, el per

1
es 4 si y slo si x x = 0, esto es, cuando x = y = 1.
o
Si xy = 1, entonces y =

As x + y = x +
,

Ejemplo 1.2.3 Para cualquier nmero positivo x, se tiene que x +


u

1
x

2.

1
1
a
Basta observar que x + x =
x x + 2 2. Adems, la igualdad se logra

1
si y slo si x x = 0, esto es cuando x = 1.
o

Ejemplo 1.2.4 Si a, b > 0 entonces


a = b.

a
b

b
a

2, y la igualdad se da si y slo si
o

1.3 Media geomtrica-media aritmtica


e
e

Basta aplicar el ejemplo anterior con x = a .


b
Ejemplo 1.2.5 Dados a, b, c > 0, se puede construir un tringulo con lados
a
de longitud a, b y c si y slo si pa2 + qb2 > pqc2 , para cualesquiera p, q con
o
p + q = 1.
Recordemos que a, b y c son las longitudes de los lados de un tringulo si y slo
a
o
si a + b > c, a + c > b, b + c > a.
Sea Q = pa2 + qb2 pqc2 = pa2 + (1 p)b2 p(1 p)c2 = c2 p2 + (a2 b2
c2 )p + b2 . Q es una funcin cuadrtica1 de p y
o
a
Q > 0 =

a2 b2 c2

a2 b2 c2 2bc

a2 (b + c)2

4b2 c2 < 0

a2 b2 c2 + 2bc < 0

a2 (b c)2 < 0

[a + b + c] [a b c] [a b + c] [a + b c] < 0
[b + c a][c + a b][a + b c] > 0.

Ahora [b + c a][c + a b][a + b c] > 0, si los tres factores son positivos o si


uno es positivo y dos de ellos negativos. Esto ultimo es imposible, pues si por

ejemplo, [b+ca] < 0 y [c+ab] < 0, tendr


amos al sumar estas desigualdades
que c < 0, lo cual es falso. As los tres factores son necesariamente positivos.

Ejercicio 1.16 Supongamos que el polinomio ax2 + bx+ c satisface que: a > 0,
a + b + c 0, a b + c 0, a c 0 y b2 4ac 0. Muestre que sus ra
ces
son reales y se encuentran en el intervalo 1 x 1.
Ejercicio 1.17 Si a, b, c son nmeros positivos, entonces no suceden simultu
a
1
neamente las desigualdades a(1 b) > 4 , b(1 c) > 1 , c(1 a) > 1 .
4
4

1.3.

Una desigualdad fundamental,


media geomtrica-media aritmtica
e
e

La primera desigualdad que consideramos fundamental en los problemas de


optimizacin, es la desigualdad entre la media geomtrica y la media aritmtica
o
e
e
Una funcin cuadrtica ax2 + bx + c con a > 0, es positiva cuando su discriminante
o
a

`
2
b 2
= b2 4ac es negativo. En efecto, esto se sigue de que ax2 +bx+c = a x + 2a + 4acb .
4a

b b2 4ac
Recuerde que las ra
ces son
, y son reales cuando 0, en caso contrario no
2a
hay ra
ces reales, por lo que ax2 + bx + c toma un mismo signo, esta expresin ser positiva
o
a
si a > 0.
1

Desigualdades Numricas
e

de dos nmeros no negativos a y b, que est dada por


u
a

a+b
.
(M G M A)
ab
2
Adems la igualdad ocurre si y slo si a = b.
a
o

a+b
e
Los nmeros ab y 2 se conocen como la media geomtrica y la media
u
aritmtica de a y b, respectivamente. Para demostrar la desigualdad basta
e
observar que,

2
a + b 2 ab
a+b
1
a b 0.
ab =
=
2
2
2

Desde luego la igualdad se logra si y slo si a = b, es decir, cuando a = b.


o

Ejercicio 1.18 Para x 0, 1 + x 2 x.


Ejercicio 1.19 Para x > 0, x +

1
x

2.

Ejercicio 1.20 Para x, y R+ , x2 + y 2 2xy.


Ejercicio 1.21 Para x, y R+ , 2(x2 + y 2 ) (x + y)2 .
1
x

Ejercicio 1.22 Para x, y R+ ,

1
y

Ejercicio 1.23 Para a, b, x R+ , ax +


a
b

Ejercicio 1.24 Para a, b > 0,

Ejercicio 1.25 Para 0 < b a,

b
a

4
x+y .

b
x

2 ab.

2.

1 (ab)2
8
a

a+b
2

ab

1 (ab)
8
b

Presentamos ahora una demostracin geomtrica y visual de las siguientes deo


e
sigualdades, para x, y > 0,
1
x

2
+

1
y

x+y

.
xy
2

(1.1)

g
B

D
x

h
E
C

O
y

1.3 Media geomtrica-media aritmtica


e
e

Construyamos una semicircunferencia de dimetro BC = x+ y y sean x = BD,


a
y = DC. Sea A el punto donde la perpendicular a BC por D intersecta a
la semicircunferencia y sea E el pie de la perpendicular desde D al radio AO.
Denotemos por g = AD, h = AE. Por ser ABD y CAD tringulos rectngulos
a
a
semejantes, tenemos que
x
g
= ,
y
g

luego

g=

xy.

Tambin, por ser AOD y ADE tringulos rectngulos semejantes, tenemos


e
a
a

xy
h
2
2xy
.
=
luego
h=
= x+y ,
1
xy
x+y
+1
2
x

Finalmente, la geometr nos dice que en un tringulo rectngulo un cateto


a
a
a
x+y
siempre es menor que la hipotenusa. Por lo que, h g 2 , que reescribimos
como
x+y
2

xy
.
1
1
2
x + y
El nmero
u

1
1
+y
x

se conoce como la media armnica de x y y. La desigualdad


o

de la izquierda en la ecuacin (1.1) se conoce como la desigualdad entre la


o
media armnica y la media geomtrica.
o
e
Algunas desigualdades se pueden demostrar a travs de repetir la aplicacin
e
o
de una desigualdad sencilla, adems del uso de una buena idea para dividir el
a
problema, esto se puede ver resolviendo los siguientes ejercicios.
Ejercicio 1.26 Para x, y, z R+ , (x + y)(y + z)(z + x) 8xyz.
Ejercicio 1.27 Para x, y, z R, x2 + y 2 + z 2 xy + yz + zx.

Ejercicio 1.28 Para x, y, z R+ , xy + yz + zx x yz + y zx + z xy.


Ejercicio 1.29 Para x, y R, x2 + y 2 + 1 xy + x + y.
Ejercicio 1.30 Para x, y, z R+ ,

1
x

Ejercicio 1.31 Para x, y, z R+ ,

xy
z

1
y

+
yz
x

1
z

1
xy

zx
y

x + y + z.

1
yz

1 .
zx

Ejercicio 1.32 Para x, y, z R, x2 + y 2 + z 2 x y 2 + z 2 + y x2 + z 2 .


La desigualdad entre la media geomtrica y la media aritmtica se puede extene
e
der para ms nmeros. Por ejemplo, podemos ver la siguiente desigualdad entre
a u

10

Desigualdades Numricas
e

la media aritmtica y la media geomtrica para cuatro nmeros no negativos a,


e
e
u

b, c, d, que arma que a+b+c+d 4 abcd, de la siguiente manera:


4
1
a+b+c+d
=
4
2

a+b c+d
+
2
2

1
ab + cd
2

4
ab cd = abcd.

Observemos que usamos tres veces la desigualdad GM A para dos nmeros,


M
u

en cada caso: con a y b, con c y d, y con ab y cd. Ms an, la igualdad se


a u
da si y slo si a = b, c = d y ab = cd, esto es cuando los nmeros satisfacen
o
u
a = b = c = d.
Ejercicio 1.33 Para x, y R, x4 + y 4 + 8 8xy.
Ejercicio 1.34 Para a, b, c, d R+ , (a + b + c + d)
Ejercicio 1.35 Para a, b, c, d R+ ,

a
b

b
c

c
d

d
a

1
a

1
b

1
c

1
d

16.

4.

Hay un muy buen truco para ver que la desigualdad


tambin es verdadera para
e
3
tres nmeros no negativos a, b y c, esto es, que abc a+b+c . Considere
u
3

los siguientes cuatro nmeros a, b, c y d = 3 abc. Como la desigualdad es


u

4
verdadera para cuatro nmeros, tenemos a+b+c+d 4 abcd = d3 d = d.
u
4

3
Luego, a+b+c d 1 d = 4 d, por lo que, a+b+c d = 3 abc.
4
4
3
Estas ideas se pueden usar para justicar la versin general de la desigualdad
o
para n nmeros no negativos. Si a1 , a2 , ... , an son n nmeros no negativos,
u
u
tomamos los nmeros A y G denidos como
u
A=

a1 + a2 + + an
n

G=

a1 a2 an .

Estos nmeros se conocen como la media aritmtica y la media geomtrica


u
e
e
de los nmeros a1 , a2 , ... , an , respectivamente.
u
Teorema 1.3.1 (Desigualdad entre las medias geomtrica y aritmtica)
e
e

a1 a2 an

a1 + a2 + + an
.
n

Primera demostracin. (de Cauchy)


o
Llamemos Pn a la armacin G A, para n nmeros. La demostracin la
o
u
o
haremos por induccin sobre n, pero una induccin del siguiente tipo:
o
o
(1) Se muestra que la armacin vale para 2 nmeros, esto es que P2 es cierta.
o
u

1.3 Media geomtrica-media aritmtica


e
e

11

(2) Se muestra que la armacin: Pn Pn1 .


o
(3) Se muestra que la armacin: Pn P2n .
o
Cuando es vlido (1), (2) y (3), todas las armaciones Pn con n 2 son
a
verdaderas. Demostramos ahora estas armaciones:
(1) Este apartado lo hemos hecho en la primera parte.

(2) Sean a1 , a2 , ... , an nmeros no negativos y sea g = n1 a1 an1 . Con


u
este nmero agregado a los nmeros a1 , ... , an1 obtenemos n nmeros
u
u
u
a los que aplicamos Pn ,
a1 + + an1 + g

n a1 a2 an1 g =
n

gn1 g = g.

Luego, a1 + + an1 + g ng y entonces se sigue que


por lo que Pn1 es verdadera.

a1 ++an1
n1

g,

(3) Sean a1 , a2 , ... , a2n nmeros no negativos, entonces


u
a1 + a2 + + a2n = (a1 + a2 ) + (a3 + a4 ) + + (a2n1 + a2n )

2 ( a1 a2 + a3 a4 + + a2n1 a2n )
1

2n ( a1 a2 a3 a4 a2n1 a2n ) n
1

= 2n (a1 a2 a2n ) 2n .
Hemos aplicado primero varias veces la armacin P2 que sabemos es cierta y
o

despus la armacin Pn a los nmeros a1 a2 , a3 a4 , ... , a2n1 a2n .


e
o
u
Segunda demostracin. Sea A = a1 ++an . Tomemos dos de los nmeros
o
u
n
ai , uno menor que A y otro mayor que A, si es que existen. Sin prdida de
e
generalidad, supongamos que son a1 = A h y a2 = A + k con h, k > 0.
Cambiaremos a1 y a2 por otros dos nmeros que incrementen el producto y que
u
dejen ja la suma, denimos
a = A, a = A + k h.
1
2
Como a + a = A + A + k h = A h + A + k = a1 + a2 , es claro que,
2
1
a + a + a3 + + an = a1 + a2 + a3 + + an , pero a a = A(A + k h) =
1 2
2
1
A2 + A(k h) y a1 a2 = (A + k)(A h) = A2 + A(k h) hk, luego,
a a > a1 a2 y entonces a a a3 an > a1 a2 a3 an .
1 2
1 2

12

Desigualdades Numricas
e

Si A = a = a = a3 = = an , ya no hay nada que demostrar (se da la


1
2
igualdad), en caso contrario hay dos elementos, uno mayor que A y otro menor
que A, y repetimos el argumento. Como cada vez que realizamos esta operacin
o
se crea un nmero igual a A, el proceso no puede ir ms all de n pasos.
u
a
a
Ejemplo 1.3.2 Encuentre el valor mximo de x(1 x3 ) para 0 x 1.
a
La idea de la prueba es transformar el producto en uno cuya suma sea constante.
Si y = x(1 x3 ), es claro que el lado derecho de 3y 3 = 3x3 (1 x3 )(1 x3 )(1
x3 ), visto como el producto de cuatro nmeros 3x3 , (1x3 ), (1x3 ) y (1x3 ),
u
tiene suma constante igual a 3. La desigualdad entre la media geomtrica y la
e
media aritmtica para los cuatro nmeros nos garantiza que
e
u
3

3y
3
. Adems,
a
434
1
x= .
3
4

luego, y

esto es, si

3x3 + 3(1 x3 )
4

3
4

el valor mximo se alcanza cuando 3x3 = 1 x3 ,


a

Ejercicio 1.36 Sean xi > 0, i = 1, ..., n. Muestre que


(x1 + x2 + + xn )

1
1
1
+
+ +
x1 x2
xn

n2 .

Ejercicio 1.37 Si {a1 , ..., an } es una permutacin de {b1 , ..., bn } R+ , eno


tonces
an
b1
b2
bn
a1 a2
+
+ +
n y
+
+ +
n.
b1
b2
bn
a1 a2
an
Ejercicio 1.38 Si a > 1, entonces an 1 > n a

n+1
2

n1
2

Ejercicio 1.39 Si a, b, c > 0 y (1 + a)(1 + b)(1 + c) = 8, entonces abc 1.


Ejercicio 1.40 Si a, b, c > 0 entonces

a3
b

b3
c

c3
a

ab + bc + ca.

Ejercicio 1.41 Si a, b, c 0, entonces


a2 b2 + b2 c2 + c2 a2 abc(a + b + c).

1.3 Media geomtrica-media aritmtica


e
e

13

Ejercicio 1.42 Si a, b, c > 0, entonces


a2 b + b2 c + c2 a

ab2 + bc2 + ca2 9a2 b2 c2 .

Ejercicio 1.43 Sean a, b, c > 0 tales que abc = 1. Muestre que


1 + ab 1 + bc 1 + ac
+
+
3.
1+a
1+b
1+c

Ejercicio 1.44 Si a, b, c > 0, muestre que


9
2
a+b+c

1
1
1
+
+
a+b b+c c+a

Ejercicio 1.45 Si Hn = 1 +

1
2

1 1 1
+ + .
a b
c

1
+ + n , muestre que

n(n + 1) n < n + Hn , para

n 2.

Ejercicio 1.46 Si x1 , x2 , ... , xn > 0, son tales que


Muestre que
x1 x2 xn (n 1)n .

1
1+x1

+ +

1
1+xn

= 1.

Ejercicio 1.47 (Lista corta IMO, 1998) Sean a1 , a2 , ... , an nmeros positivos
u
con a1 + a2 + + an < 1, muestre que
1
a1 a2 an [1 (a1 + a2 + + an )]
n+1 .
(a1 + a2 + + an ) (1 a1 ) (1 a2 ) (1 an )
n
Ejercicio 1.48 Sean a1 , a2 , ... , an nmeros positivos, tales que
u
1
1+an = 1. Muestre que

a1 + + an (n 1)

1
1
+ +
a1
an

1
1+a1

+ +

14

Desigualdades Numricas
e

Ejercicio 1.49 (APMO, 1991) Sean a1 , a2 , ... , an , b1 , b2 , ... , bn nmeros


u
positivos, tales que a1 + a2 + + an = b1 + b2 + + bn . Muestre que
a2
1
a2
n
1
+ +
(a1 + + an ).
a1 + b1
an + bn
2

Ejercicio 1.50 Sean a, b, c nmeros positivos, muestre que


u
1
1
1
1
+
+

.
a3 + b3 + abc b3 + c3 + abc c3 + a3 + abc
abc

Ejercicio 1.51 Sean a, b, c nmeros positivos con a + b + c = 1, muestre que


u
1
+1
a

1
+1
b

1
+1
c

64.

Ejercicio 1.52 Sean a, b, c nmeros positivos con a + b + c = 1, muestre que


u
1
1
a

1
1
b

1
1
c

8.

Ejercicio 1.53 (Repblicas Checa y Eslovaca, 2005). Sean a, b, c nmeros


u
u
positivos que satisfacen abc = 1, muestre que
a
b
c
3
+
+
.
(a + 1)(b + 1) (b + 1)(c + 1) (c + 1)(a + 1)
4

Ejercicio 1.54 Sean a, b, c nmeros positivos que satisfacen


u
1, muestre que
abc 8.
Ejercicio 1.55 Sean a, b, c nmeros positivos, muestre que
u
2ab
2bc
2ca
+
+
a + b + c.
a+b b+c c+a

1
1
1
1+a + 1+b + 1+c

1.4 Una desigualdad maravillosa

15

Ejercicio 1.56 Sean a1 , a2 , ... , an , b1 , b2 , ... , bn nmeros positivos, muestre


u
que
n
n
1
(ai + bi )2 4n2 .
ai bi
i=1

i=1

Ejercicio 1.57 (Rusia, 1991) Sean x, y, z nmeros reales no negativos, muestre


u
que

(x + y + z)2
x yz + y zx + z xy.
3

Ejercicio 1.58 (Rusia, 1992) Para nmeros reales positivos x, y, z, muestre


u
que

x4 + y 4 + z 2 8xyz.

Ejercicio 1.59 (Rusia, 1992) Muestre que para cualesquiera nmeros reales x,
u
y > 1, se tiene
x2
y2
+
8.
y1 x1

1.4.

Una desigualdad maravillosa,


la desigualdad del reacomodo

Considere dos colecciones de nmeros reales ordenados en forma creciente,


u
a1 a2 an

b1 b2 b n .

Para cualquier permutacin (a , a , ..., a ) de (a1 , a2 , ..., an ), se tiene que


o
n
1 2
a1 b1 + a2 b2 + + an bn a b1 + a b2 + + a bn
2
n
1

an b1 + an1 b2 + + a1 bn

(1.2)
(1.3)

Adems, la igualdad en (1.2) es cierta si y slo si (a , a , ..., a ) = (a1 , a2 , ..., an ).


a
o
n
1 2
Y la igualdad en (1.3) es cierta si y slo si (a , a , ..., a ) = (an , an1 , ..., a1 ).
o
n
1 2
A la desigualdad (1.2) se le conoce como la desigualdad del reacomodo.

16

Desigualdades Numricas
e

Corolario 1.4.1 Para cualquier permutacin (a , a , ..., a ) de (a1 , a2 , ..., an ),


o
n
1 2
se tiene que
a2 + a2 + + a2 a1 a + a2 a + + an a .
1
2
n
1
2
n
Corolario 1.4.2 Para cualquier permutacin (a , a , ..., a ) de (a1 , a2 , ..., an ),
o
n
1 2
se tiene que
a
a
a
1
+ 2 + + n n.
a1 a2
an
Demostracin (de la desigualdad del reacomodo)
o
Supongamos que b1 b2 bn . Sean
S = a1 b1 + a2 b2 + + ar br + + as bs + + an bn

= a1 b1 + a2 b2 + + as br + + ar bs + + an bn .

La diferencia entre S y S es que los coecientes de br y bs , donde r < s, estn


a
intercambiados. Por lo tanto,
S S = ar br + as bs as br ar bs = (bs br )(as ar ).
Entonces podemos armar que S S si y slo si as ar . Repitiendo este
o
proceso tenemos que la suma S es la mayor cuando a1 a2 an .
Ejemplo 1.4.3 (IMO, 1975) Considere dos colecciones de nmeros x1 x2
u
xn , y1 y2 yn y una permutacin (z1 , z2 , ..., zn ) de (y1 , y2 , ..., yn ).
o
Muestre que
(x1 y1 )2 + + (xn yn )2 (x1 z1 )2 + + (xn zn )2 .
Elevando al cuadrado y reacomodando la desigualdad anterior, tenemos que es
equivalente a

i=1

x2 2
i
n
2
i=1 yi

xi y i +
i=1

i=1
n
2
i=1 zi ,

pero como
=
es equivalente a demostrar

2
yi

i=1

x2 2
i

2
zi ,

xi zi +
i=1

i=1

entonces la desigualdad que tenemos que probar


n

n
i=1

xi zi

que es la desigualdad (1.2) del teorema.

xi yi ,
i=1

1.4 Una desigualdad maravillosa

17

Ejemplo 1.4.4 (IMO, 1978) Sean x1 , x2 , ... , xn enteros positivos distintos,


muestre que
1
xn
1 1
x1 x2
+ 2 + + 2 + + + .
12
2
n
1 2
n
Sea (a1 , a2 , . . . , an ) una permutacin de (x1 , x2 , . . . , xn ) con a1 a2
o
1
1
1
1
an y sea (b1 , b2 , . . . , bn ) = n2 , (n1)2 , , 12 , esto es bi = (n+1i)2 para los

ndices i = 1, . . . , n.
Considere la permutacin (a , a , . . . , a ) de (a1 , a2 , . . . , an ) denida por a =
o
n
1 2
i
xn+1i , para i = 1, . . . , n.
Utilizando la desigualdad (1.3) tenemos que
xn
x1 x2
+ 2 + + 2
2
1
2
n

= a b1 + a b2 + + a bn
1
2
n
an b1 + an1 b2 + + a1 bn

= a1 bn + a2 bn1 + + an b1
an
a1 a2
+ 2 + + 2.
=
2
1
2
n

Como 1 a1 , 2 a2 , ... , n an , tenemos que


x1 x2
1
xn
a1 a2
an
1
2
n
1 1
+ + + 2 2 + 2 + + 2 2 + 2 + + 2 = + + + .
12 22
n
1
2
n
1
2
n
1 2
n
Ejemplo 1.4.5 (IMO, 1964) Sean a, b, c las longitudes de los lados de un
tringulo. Muestre que
a
a2 (b + c a) + b2 (a + c b) + c2 (a + b c) 3abc.
Como la expresin es una funcin simtrica en a, b y c, podemos suponer sin
o
o
e
perder generalidad que c b a. En tal caso, a (b + c a) b (a + c b)
c (a + b c).
Por ejemplo, la primera desigualdad se justica como sigue

a (b + c a) b (a + c b) ab + ac a2 ab + bc b2
(a b) c (a + b) (a b)

(a b) (a + b c) 0.

Por la ecuacin (1.3) de la desigualdad del reacomodo, tenemos que


o
a2 (b+ca)+b2 (c+ab)+c2 (a+bc) ba(b+ca)+cb(c+ab)+ac(a+bc)
a2 (b+ca)+b2 (c+ab)+c2 (a+bc) ca(b+ca)+ab(c+ab)+bc(a+bc).

Por lo tanto, 2 a2 (b + c a) + b2 (c + a b) + c2 (a + b c) 6abc.

18

Desigualdades Numricas
e

Ejemplo 1.4.6 (IMO, 1983) Sean a, b, c los lados de un tringulo, muestre


a
que
a2 b(a b) + b2 c(b c) + c2 a (c a) 0.
Consideremos el caso c b a, (los otros casos se desarrollan de manera
similar).
Como en el ejemplo anterior, tenemos que a(b+ca) b(a+cb) c(a+bc)
y como adems a 1 1 , por la desigualdad (1.2) obtenemos
a 1
b
c
1
1
1
a(b + c a) + b(c + a b)+ c(a + b c)
a
b
c
1
1
1
a(b + c a) + b(c + a b) + c(a + b c).
c
a
b
a (b a) b(c b) c (a c)
+
+
+ a + b + c.
c
a
b
a(b a) b(c b) c(a c)
De donde tenemos que
+
+
0. Multiplicando por
c
a
b
abc, obtenemos

Por lo tanto,

a+b+c

a2 b (a b) + b2 c(b c) + c2 a(c a) 0.
Ejemplo 1.4.7 (Desigualdad de Cauchy-Schwarz ) Para cualesquiera nmeu
ros reales x1 , . . ., xn , y1 , . . ., yn , la siguiente desigualdad se cumple
2

xi yi
i=1

2
yi

x2
i
i=1

i=1

La igualdad se da si y slo si existe una R con xi = yi , para cada


o
i = 1, 2, ..., n.
Si x1 = x2 = = xn = 0 o y1 = y2 = = yn = 0 el resultado es evidente.

n
n
2
2
De lo contrario, sean S =
i=1 xi y T =
i=1 yi , claramente S = 0 y
yi
xi
T = 0. Denamos ai = S y an+i = T , para i = 1, 2, ..., n. Usando el corolario
1.4.1,
n

2 =
i=1

x2
i
+
S2

n
i=1

2
yi
=
T2

2n

a2
i
i=1

a1 an+1 + a2 an+2 + + an a2n + an+1 a1 + + a2n an


= 2

x1 y1 + x2 y2 + + xn yn
.
ST

1.4 Una desigualdad maravillosa

19

La igualdad ocurre si y slo si ai = an+i , para i = 1, 2, . . . , n, y esto es cierto


o
S
si y slo si xi = T yi , para i = 1, 2, . . . , n.
o
Podemos dar otra demostracin de la desigualdad de Cauchy-Schwarz utilizando
o
la identidad de Lagrange
2

xi y i

2
yi

x2
i

=
i=1

i=1

i=1

i=1 j=1

(xi yj xj yi )2 .

La importancia de la desigualdad de Cauchy-Schwarz ser evidente a lo largo


a
del libro. Como veremos, es una herramienta util para resolver un gran nmero

u
de ejercicios y problemas.
Ejemplo 1.4.8 (Desigualdad de Nesbitt) Para a, b, c R+ , se tiene que
a
b
c
3
+
+
.
b+c c+a a+b
2
Sin perder generalidad, podemos suponer que a b c, de esta hiptesis se
o
1
1
1
sigue que a + b c + a b + c y b+c c+a a+b .
Usando la desigualdad del reacomodo (1.2) dos veces, tenemos
b
c
b
c
a
a
+
+

+
+
b+c c+a a+b
b+c c+a a+b
a
b
c
c
a
b
+
+

+
+
.
b+c c+a a+b
b+c c+a a+b
Por lo tanto, 2

b
c
a
+
+
b+c c+a a+b

b+c c+a a+b


+
+
b+c c+a a+b

= 3.

Otra forma, de demostrar la desigualdad es usando la desigualdad (1.3) dos


veces,
c+a a+b b+c
+
+
b+c c+a a+b
a+b b+c c+a
+
+
b+c c+a a+b

3
3.

Luego, al sumar las dos expresiones obtenemos 2a+b+c + 2b+c+a + 2c+a+b 6,


b+c
c+a
a+b
y entonces
2b
2c
2a
+
+
3.
b+c c+a a+b

20

Desigualdades Numricas
e

Ejemplo 1.4.9 (IMO, 1995) Sean a, b, c R+ , con abc = 1. Muestre que


1
1
1
3
+ 3
+ 3
.
+ c) b (c + a) c (a + b)
2

a3 (b

Sin perder generalidad podemos suponer c b a. Sean x =


z = 1 , entonces
c

1
a,

y =

1
b

1
1
1
+ 3
+ 3
+ c) b (c + a) c (a + b)

S =

a3 (b

x3

1 1
+
y z

y3

1 1
+
z x

z3
1 1
+
x y

y2
z2
x2
+
+
.
y+z z+x x+y

Como x y z, tenemos que x + y z + x y + z y tambin que


e
y
z
x
z+x x+y . Por la desigualdad del reacomodo (1.2), obtenemos
y+z
x2
y2
z2
+
+
y+z z+x x+y

x2
y2
z2
+
+
y+z z+x x+y

xy
yz
zx
+
+
y+z z+x x+y

xz
yx
zy
+
+
,
z+x z+x x+y

que a su vez nos lleva a 2S x + y + z 3 3 xyz = 3. Por lo tanto, S 3 .


2
Ejemplo 1.4.10 (APMO, 1998) Sean a, b, c R+ , muestre que
1+

a
b

a+b+c
c
2 1+
3
a
abc

1+

b
c

1+

b
c

1+

a+b+c
c
2 1+
3
a
abc

Observemos que
1+

a
b

1+

a b c
+ +
b c a

a c b
+ +
c b a

a+b+c
abc
2 1+
3
abc
abc

1+

2(a + b + c)
a b c a c b

+ + + + +
.
3
b c a c b a
abc

1.4 Una desigualdad maravillosa

21

Deniendo a = x3 , b = y 3 , c = z 3 . Tenemos ahora que mostrar que


2 x3 + y 3 + z 3
x3 y 3
z3
x3 z 3
y3
+ 3+ 3+ 3 + 3+ 3
.
y3
z
x
z
y
x
xyz
Pero, si consideramos (a1 , a2 , a3 , a4 , a5 , a6 ) =
(a , a , a , a , a , a ) =
1 2 3 4 5 6
=

x2
y2

y2

, z2 ,

z2
x2

x2
z2

z2
y2

y z x z y x
z, x, y, y, x, z

x y z x z y
y , z , x, z , y, x

y (b1 , b2 , b3 , b4 , b5 , b6 ) =

y2

, x2 , tenemos que

x3 y 3
z3
x3 z 3
y3
x2 y y 2 z
z 2 x x2 z
z2 y
y2 x
+ 3+ 3+ 3 + 3+ 3 2 + 2 + 2 + 2 + 2 + 2
y3
z
x
z
y
x
y z z x x y
z y y x x z
=

x2
y2
z2
x2
z2
y2
+
+
+
+
+
yz zx xy zy yx xz

2 x3 + y 3 + z 3
.
xyz

Ejemplo 1.4.11 (Desigualdad de Tchebyshev) Sean a1 a2 an y


b1 b2 bn , entonces
a1 + a2 + + an b1 + b2 + + bn
a1 b1 + a2 b2 + + an bn

.
n
n
n
Aplicando varias veces la desigualdad del reacomodo obtenemos
a1 b1 + + an bn = a1 b1 + a2 b2 + + an bn

a1 b1 + + an bn a1 b2 + a2 b3 + + an b1

a1 b1 + + an bn a1 b3 + a2 b4 + + an b2
.
.
.
.
.
.
.
.
.

a1 b1 + + an bn a1 bn + a2 b1 + + an bn1 ,
al sumar todas las expresiones, obtenemos
n (a1 b1 + + an bn ) (a1 + + an ) (b1 + + bn ) .
Notemos que la igualdad ocurre solamente cuando a1 = a2 = = an o
b1 = b2 = = bn .

Ejercicio 1.60 Cualesquiera tres nmeros reales positivos a, b y c cumplen que


u
a3 + b3 + c3 a2 b + b2 c + c2 a.

22

Desigualdades Numricas
e

Ejercicio 1.61 Cualesquiera tres nmeros reales positivos a, b y c, con abc = 1,


u
satisfacen que
a3 + b3 + c3 + (ab)3 + (bc)3 + (ca)3 2(a2 b + b2 c + c2 a).

Ejercicio 1.62 Cualesquiera tres nmeros reales positivos a, b y c cumplen que


u
c2
b c a
a2 b2
+ 2+ 2 + + .
2
b
c
a
a b c

Ejercicio 1.63 Cualesquiera tres nmeros reales positivos a, b y c cumplen que


u
1
1
1
a+b+c
2 + 2 + 2.
abc
a
b
c

Ejercicio 1.64 Si a, b y c son la longitudes de los lados de un tringulo, muestre


a
que
a
b
c
+
+
3.
b+ca c+ab a+bc
Ejercicio 1.65 Si a1 , a2 , ... , an R+ y s = a1 + a2 + + an , entonces
a2
an
n
a1
.
+
+ +

s a1 s a2
s an
n1

Ejercicio 1.66 Si a1 , a2 , ... , an R+ y s = a1 + a2 + + an , entonces


s
s
n2
s
.
+
+ +

s a1 s a2
s an
n1
Ejercicio 1.67 Si a1 , a2 , ... , an R+ y a1 + a2 + + an = 1, entonces
a2
an
n
a1
.
+
+ +

2 a1 2 a2
2 an
2n 1

1.4 Una desigualdad maravillosa

23

Ejercicio 1.68 (Desigualdad media cuadrtica-media aritmtica) Sean


a
e
x1 , x2 , ... , xn R+ , entonces
x1 + x 2 + + x n
x2 + x2 + + x2
n
1
2

.
n
n

Ejercicio 1.69 Para nmeros reales positivos a, b, c, que satisfacen a+b+c = 1,


u
muestre que
1
ab + bc + ca .
3
Ejercicio 1.70 (Media armnica, geomtrica y aritmtica) Sean
o
e
e
x1 , x2 , ... , xn R+ , muestre que
1
x1

1
x2

n
+ +

1
xn

x1 + x2 + + xn
n
.
x1 x2 xn
n

Donde las igualdades se dan si y slo si x1 = x2 = = xn .


o
Ejercicio 1.71 Sean a1 , a2 , ... , an nmeros positivos con a1 a2 an = 1.
u
Muestre que
n1
n1
n1
a1 + a2 + + an

1
1
1
+
+ + .
a1 a2
an

u
Ejercicio 1.72 (China, 1989) Sean a1 , a2 , ... , an nmeros positivos, tales que
a1 + a2 + + an = 1. Muestre que

a1
an
1
( a1 + + an ).
+ +

1 a1
1 an
n1

Ejercicio 1.73 Sean a, b y c nmeros positivos tales que a + b + c = 1. Muestre


u
que:

(i) 4a + 1 + 4b + 1 + 4c + 1 < 5,

(ii) 4a + 1 + 4b + 1 + 4c + 1 21.

24

Desigualdades Numricas
e

Ejercicio 1.74 Sean a, b, c, d R+ , con ab + bc + cd + da = 1, muestre que


b3
c3
d3
1
a3
+
+
+
.
b+c+d a+c+d a+b+d a+b+c
3

Ejercicio 1.75 Sean a, b, c nmeros positivos, con abc = 1, muestre que


u
a b c
+ + a + b + c.
b c a

Ejercicio 1.76 Sean x1 , x2 , ... , xn (n > 2) nmeros reales tales que la suma
u
de cualesquiera n 1 de ellos es mayor que el elemento que quedo fuera de la
suma. Sea s = n xk . Muestre que
k=1
n
k=1

1.5.

x2
s
k

.
s 2xk
n2

Funciones convexas

Una funcin f : [a, b] R es convexa en el intervalo I = [a, b], si para cada


o
t [0, 1] y para a x < y b, se tiene la desigualdad
f (ty + (1 t)x) tf (y) + (1 t)f (x).

(1.4)

Geomtricamente, la desigualdad anterior se interpreta diciendo que la grca


e
a
de f entre x y y queda por debajo del segmento que une a los puntos (x, f (x))
y (y, f (y)).

(x, f (x))

(y, f (y))

1.5 Funciones convexas

25

En efecto, la ecuacin de la recta que pasa por (x, f (x))y (y, f (y)) es
o
L(s) = f (x) +

f (y) f (x)
(s x).
yx

Luego, al evaluar en s = ty + (1 t)x, obtenemos que


f (y) f (x)
(t(y x)) = f (x) + t(f (y) f (x))
yx
= tf (y) + (1 t)f (x).

L(ty + (1 t)x) = f (x) +

Por lo que, la desigualdad (1.4) es equivalente a


f (ty + (1 t)x) L(ty + (1 t)x).
Proposicin 1.5.1 (1) Si f es convexa en el intervalo [a, b], entonces es cono
vexa en cualquier subintervalo [x, y] [a, b] .
(2) Si f es convexa en [a, b], entonces, para cualesquiera x, y [a, b], se tiene
f

x+y
2

1
(f (x) + f (y)).
2

(1.5)

(3) (Desigualdad de Jensen) Si f es convexa en [a, b], entonces para cualesquiera t1 , ... , tn [0, 1] con n ti = 1 y para x1 , ... , xn [a, b], se
i=1
tiene
f (t1 x1 + + tn xn ) t1 f (x1 ) + + tn f (xn ).
(4) En particular, para x1 , ... , xn [a, b], podemos armar que
f

x1 + + x n
n

1
(f (x1 ) + + f (xn )) .
n

Demostracin. (1) Se deja como ejercicio al lector.


o
(2) Basta tomar t =

1
2

en la ecuacin (1.4).
o

t1
(3) f (t1 x1 + + tn xn ) = f ((1 tn ) ( 1tn x1 + +

(1 tn ) f

tn1
1tn xn1 )

+ tn xn )
tn1
+ + 1tn xn1 + tn f (xn ), por convexidad
tn1
t1
o
1tn f (x1 ) + + 1tn f (xn1 ) + tn f (xn ), por induccin
t1
1tn x1

(1 tn )
= t1 f (x1 ) + + tn f (xn ).

1
(4) Solamente tenemos que aplicar (3) con t1 = t2 = = tn = n .

26

Desigualdades Numricas
e

Observaciones 1.5.2 (i) Podemos ver que (4) es vlida bajo la unica suposia

cin de que f satisfaga la relacin f x+y f (x)+f (y) , para cualesquiera x,


o
o
2
2
y [a, b].
(ii) Podemos ver que (3) es vlida para t1 , . . . , tn [0, 1] nmeros racionales,
a
u
x+y
bajo la unica condicin de que f satisfaga la relacin f 2 f (x)+f (y) para

o
o
2
cualesquiera x, y [a, b].
La demostracin de (i), la haremos por induccin. Llamemos Pn a la armacin
o
o
o
f

x1 + + x n
n

1
(f (x1 ) + + f (xn )) ,
n

para x1 , . . ., xn [a, b] . Es claro que son vlidas las armaciones P1 y P2 .


a
Ahora, mostremos que Pn Pn1 .
Sean x1 , . . ., xn [a, b] y sea y = x1 ++xn1 . Entonces, por ser cierto Pn ,
n1
tenemos
f

x1 + + xn1 + y
n

1
1
1
f (x1 ) + + f (xn1 ) + f (y).
n
n
n

Pero el lado izquierdo es f (y), por lo que nf (y) f (x1 )+ +f (xn1 )+f (y),
luego
1
f (y)
(f (x1 ) + + f (xn1 )) .
n1
Finalmente veamos que Pn P2n .
Sea D = f

x1 ++xn +xn+1 ++x2n


2n

=f

u+v
2

, donde u =

x1 ++xn
n

yv =

xn+1 ++x2n
.
n

Como f
D

u+v
2

1
2 (f (u) + f (v)), tenemos que

x1 + + x n
xn+1 + + x2n
1
1
(f (u) + f (v)) =
f
+f
2
2
n
n
1
(f (x1 ) + + f (xn ) + f (xn+1 ) + + f (x2n )) ,
2n

donde hemos usado dos veces la armacin Pn .


o
Para demostrar (ii), nuestro punto de partida ser considerar la armacin
a
o
1
f x1 ++xn n (f (x1 ) + + f (xn )), para x1 , . . ., xn [a, b] y n N.
n
n
r
r
u
Sean t1 = s1 , . . ., tn = sn , nmeros racionales en [0, 1] con
i=1 ti = 1.
n
1
pi
Si m es el m
nimo comn mltiplo de las si , entonces ti = m con pi N y
u
u

1.5 Funciones convexas


n
i=1 pi

27

= m, luego

f (t1 x1 + + tn xn ) = f

pn
p1
x1 + + xn
m
m

= f (x1 + + x1 ) + + (xn + + xn )
m

p1 trminos
e

pn trminos
e

(f (x1 ) + + f (x1 )) + + (f (xn ) + + f (xn ))


m
p1 trminos
e

p1
pn
= f (x1 ) + + f (xn )
m
m
= t1 f (x1 ) + + tn f (xn ).

pn trminos
e

Observacin 1.5.3 Si f : [a, b] R es una funcin continua2 en [a, b] y


o
o
cumple el inciso (2) de la proposicin, entonces f es convexa.
o
Hemos visto que si f cumple (2), entonces
f (qx + (1 q)y) qf (x) + (1 q)f (y),
para cualesquiera x, y [a, b] y q [0, 1] un nmero racional. Como cualquier
u
real t se puede aproximar por una sucesin de nmeros racionales qn , tenemos
o
u
que si estos qn estn en [0, 1], entonces
a
f (qn x + (1 qn )y) qn f (x) + (1 qn )f (y).
Ahora bien, la continuidad de f nos garantiza que al tomar el l
mite obtenemos
f (tx + (1 t)y) tf (x) + (1 t)f (y).
Decimos que f : [a, b] R es cncava si f es convexa.
o
Observacin 1.5.4 Una funcin f [a, b] R es cncava en [a, b] si y slo si
o
o
o
o
f (ty + (1 t)x) tf (y) + (1 t)f (x), para 0 t 1 y a x < y b.
Veamos ahora algunos criterios para decidir si una funcin es convexa.
o
2
Una funcin f : [a, b] R es continua en el punto c [a, b] si l xc f (x) = f (c), y f
o
m
es continua en [a, b] si es continua para todo punto en el intervalo. Equivalentemente, f es
continua en c, si para toda sucesin de puntos {cn } que converge a c, la sucesin {f (cn )}
o
o
converge a f (c).

28

Desigualdades Numricas
e

Criterio 1.5.5 Una funcin f : [a, b] R es convexa si y slo si el conjunto


o
o
{(x, y) | a x b, f (x) y} es convexo3 .
Demostracin. Supongamos que f es convexa y, sean A = (x1 , y1 ) y B =
o
(x2 , y2 ) dos puntos del conjunto U = {(x, y) | a x b, f (x) y}. Para
mostrar que tB + (1 t)A = (tx2 + (1 t)x1 , ty2 + (1 t)y1 ) pertenece a U ,
bastar ver que a tx2 + (1 t)x1 b y f (tx2 + (1 t)x1 ) ty2 + (1 t)y1 .
a
La primera condicin es inmediata de que tanto x1 como x2 pertenecen a [a, b].
o
Para la segunda condicin, como f es convexa, sucede que
o
f (tx2 + (1 t)x1 ) tf (x2 ) + (1 t)f (x1 )
adems, como f (x2 ) y2 y f (x1 ) y1 , se tiene que
a
f (tx2 + (1 t)x1 ) ty2 + (1 t)y1 .
Ahora, veamos que f es convexa si U es convexo.
Sean x1 , x2 [a, b] y, consideremos A = (x1 , f (x1 )) y B = (x2 , f (x2 )).
Claramente, A y B estn en U , y por ser U convexo tambin est en U el
a
e
a
segmento que los une, es decir, los puntos de la forma tB + (1 t)A para
t [0, 1]. Esto es,
(tx2 + (1 t)x1 , tf (x2 ) + (1 t)f (x1 )) U ,
pero esto implica que f (tx2 + (1 t)x1 ) tf (x2 ) + (1 t)f (x1 ), por lo que
f es convexa.

Criterio 1.5.6 Una funcin f : [a, b] R es convexa si y slo si para cada


o
o
f (x)f (x0 )
es no-decreciente para
x0 [a, b] se cumple que la funcin P (x) = xx0
o
x = x0 .
Demostracin. Supongamos que f es convexa. Para probar que P (x) es noo
decreciente tomemos x < y y mostremos que P (x) P (y). Podemos tener
las siguientes tres situaciones: x0 < x < y, x < x0 < y x < y < x0 .
o
3
Un subconjunto C del plano es convexo si para cada par de puntos A, B en C se tiene
que el segmento de recta que stos determinan est contenido en C. Como el segmento entre
e
a
A y B son los puntos de la forma tB + (1 t)A, con 0 t 1, lo que se pide es que ste
e
pertenezca a C.

1.5 Funciones convexas

29

Consideremos el primer caso, los otros dos se demuestran de manera semejante.


Notemos primero que
f (y) f (x0 )
f (x) f (x0 )

x x0
y x0
(f (x) f (x0 ))(y x0 ) (f (y) f (x0 ))(x x0 )

P (x) P (y)

f (x)(y x0 ) f (y)(x x0 ) + f (x0 )(y x)


yx
x x0
+ f (x0 )
f (x) f (y)
y x0
y x0
x x0
yx
yx
x x0
y+
x0 f (y)
+ f (x0 )
.
f
y x0
y x0
y x0
y x0
De aqu el resultado ya es evidente.
,

Criterio 1.5.7 Si la funcin f : [a, b] R es derivable4 con derivada noo


decreciente, entonces f es convexa. En particular, si f es dos veces derivable y
f (x) 0 entonces la funcin es convexa.
o
Demostracin. Desde luego f (x) 0, para x [a, b], garantiza que f (x)
o
es no-decreciente. Vemos que f (x) no-decreciente implica que la funcin es
o
convexa.
Sea x = tb + (1 t)a, un punto de [a, b]. Por el teorema del valor medio5 ,
existen c (a, x) y d (x, b) tales que
f (x) f (a) = (x a)f (c) = t(b a)f (c)
y

f (b) f (x) = (b x)f (d) = (1 t)(b a)f (d).

Luego, por ser f (x) no-decreciente, tenemos que


(1 t) (f (x)f (a)) = t(1t)(ba)f (c) t(1t)(ba)f (d) = t(f (b)f (x))
que reacomodando los trminos nos da
e
f (x) tf (b) + (1 t)f (a).
Una funcin f : [a, b] R es derivable en un punto c [a, b] si l xc f (x)f (c) = f (c)
o
m
xc
existe, y es derivable en A [a, b] si lo es en cada punto de A.
5
Teorema del valor medio: Para una funcin continua f : [a, b] R, que tambin es
o
e
derivable en (a, b) existe un nmero x (a, b) tal que f (x)(b a) = f (b) f (a). Consultar
u
[21], pg. 266.
a
4

30

Desigualdades Numricas
e

Veamos, ahora, una interpretacin geomtrica de la convexidad (y la cono


e
cavidad).
Sean x, y, z puntos en el intervalo [a, b] con x < y < z. Si los vrtices
e
del tringulo XY Z tienen coordenadas X = (x, f (x)), Y = (y, f (y)), Z =
a
(z, f (z)), entonces el rea del tringulo est dada por
a
a
a
1
= det A, donde
2

1 x f (x)
A = 1 y f (y) .
1 z f (z)

El rea puede ser positiva o negativa, dependiendo si el tringulo XY Z se


a
a
recorre en sentido positivo (contrario al avance de las manecillas del reloj) o en
sentido negativo. Para una funcin convexa tendremos que > 0 y para una
o
funcin cncava < 0, como muestran las grcas siguientes.
o o
a

(y, f (y))

(x, f (x))

(z, f (z))

(z, f (z))
(y, f (y))

(x, f (x))

En efecto,
> 0 det A > 0
(z y)f (x) (z x)f (y) + (y x)f (z) > 0
yx
zy
f (x) +
f (z).
f (y) <
zx
zx
Si tomamos t = yx , tenemos que 0 < t < 1, 1 t =
zx
y f (tz + (1 t)x) < tf (z) + (1 t)f (x).

zy
zx ,

y = tz + (1 t)x

Ahora veamos, una serie de ejemplos, donde resaltamos el uso de las funciones
convexas para establecer desigualdades.
Ejemplo 1.5.8 La funcin f (x) = xn , n 1 es convexa en R+ y la funcin
o
o
n , con n par, es convexa en R.
f (x) = x

1.5 Funciones convexas

31

Lo anterior se sigue de que, f (x) = n(n 1)xn2 0, en las respectivas


situaciones.
Como aplicaciones de este hecho obtenemos lo siguiente:
2

(i) Ya que a+b a +b , tenemos que a+b a +b , que es la desigualdad


2
2
2
2
entre la media aritmtica y la media cuadrtica.
e
a
(ii) Como

a+b n
2

1, se cumple

an +bn
tenemos que,
2
1
que an + bn n1 .
2

para a y b positivos tales que a + b =

(iii) Tenemos que, para nmeros positivos a y b, 1 +


u
Esto se sigue de que,
a+b
a

+
2

a+b
b

a n
+
b

1+

1
a
+f
f 1+
2
b

2n = f (2) f

1
2

1+

a
b

b n
a

2n+1 .

1+

b
a

+ 1+

b
a

Ejemplo 1.5.9 La funcin exponencial f (x) = ex es convexa en R, ya que


o
f (x) = ex > 0, para todo x R.
Veamos varias aplicaciones de esta propiedad:
(i) (Desigualdad M GM A con pesos) Si x1 , ... , xn , t1 , ... , tn son nmeros
u
positivos y n ti = 1, entonces
i=1
xt1 xtn t1 x1 + + tn xn .
n
1
En efecto, como xti = eti log xi y ex es convexa, tenemos que
i
xt1 xtn
n
1

= et1 log x1 etn log xn = et1 log x1 ++tn log xn

t1 elog x1 + + tn elog xn = t1 x1 + + tn xn .

1
En particular, si tomamos ti = n , para 1 i n, obtenemos otra
demostracin de la desigualdad entre la media geomtrica y la media
o
e
aritmtica para n nmeros.
e
u
(ii) (Desigualdad de Young) Sean x, y nmeros reales positivos. Si a, b > 0
u
1
1
1
satisfacen la condicin a + b = 1, entonces xy a xa + 1 y b .
o
b

Solamente es necesario aplicar la parte (i) como sigue,


1

xy = (xa ) a y b

1
b

1 a 1 b
x + y .
a
b

32

Desigualdades Numricas
e

(iii) (Desigualdad de Hlder) Sean x1 , x2 , ... , xn , y1 , y2 , ... , yn , nmeros


o
u
1
positivos y a, b > 0 tales que a + 1 = 1, entonces
b

i=1

1/a

xi y i

b
yi

xi yi

n
n
a
b
i=1 xi =
i=1 yi
1 b
1 a
a xi + b yi , luego

i=1

1
a

a
x
i

xa +
i
n
a
i=1 xi

Supongamos ahora que


xi
y = Byi . Como
1/b
A1/a i
n

n
i=1

n
a
i=1 xi

i=1

i=1

i=1

Supongamos primero que


Por la parte (ii), xi yi

1/b

xa
i

1
b

= 1.

n
b
yi =
i=1

1 1
+ = 1.
a b

n
b
i=1 yi

=A y
n

yi

=1 y

= B. Tomemos x =
i
n
b
i=1 yi

i=1

= 1,

tenemos que
n

1
Por lo tanto,

x y i
i

=
i=1

i=1
n
i=1 xi yi

1
xi yi
= 1/a 1/b
1/a B 1/b
A
A B

xi y i .
i=1

A1/a B 1/b .

Como un caso particular, cuando a = b = 2, se obtiene la desigualdad de


Cauchy-Schwarz.
Veamos ahora una consecuencia de la desigualdad de Hlder, la cual es una
o
generalizacin de la desigualdad del tringulo.
o
a
Ejemplo 1.5.10 (Desigualdad de Minkowski) Sean a1 , a2 , ... , an , b1 , b2 , ...
, bn nmeros positivos y sea p > 1, entonces
u
1
p

(ak + bk )p
k=1

1
p

(ak )p
k=1

1
p

(bk )p

+
k=1

Observemos que
(ak + bk )p = ak (ak + bk )p1 + bk (ak + bk )p1

1.5 Funciones convexas

33

luego,
n

(ak + bk )p =
k=1

ak (ak + bk )p1 +
k=1

bk (ak + bk )p1 .

(1.6)

k=1

Usemos la desigualdad de Hlder en cada trmino de la suma de la derecha en


o
e
(1.6), con q tal que 1 + 1 = 1, para obtener
p
q
n
k=1

ak (ak + bk )p1

(ak + bk )q(p1)

k=1

k=1
1
p

bk (ak + bk )p1

1
q

(ak )p

n
k=1

1
p

(bk )p
k=1

1
q

(ak + bk )q(p1)

k=1

Sustituyendo estas desigualdades en (1.6), y observando que q(p 1) = p, obtenemos la desigualdad requerida. La desigualdad de Minkowski es una igualdad
si permitimos p = 1. Para 0 < p < 1, la desigualdad se invierte.
Ejemplo 1.5.11 (Lista corta IMO, 1998) Si r1 , ... , rn son nmeros reales
u
mayores que 1, muestre que
1
n
1
.
+ +

n r r + 1
1 + r1
1 + rn
1
n
1
es convexa para R+ , ya que
1 + ex
0, para x > 0.

Notemos primero que la funcin f (x) =


o
f (x) =

ex
(1+ex )2

y f (x) =

Ahora, si ri > 1, entonces


convexa, tenemos que

ex (ex 1)
(ex +1)3
ri = exi

1
x ++x
( 1 n n)

e
por tanto,

+1

1
n

para alguna xi > 0. Como f (x) =

1
1+ex

es

1
1
+ +
x1
1+e
1 + exn

1
1
n

+ +
.
r1 r n + 1
1 + r1
1 + rn

Ejemplo 1.5.12 (China, 1989) Para cualesquiera n nmeros x1 , ... , xn (0, 1)


u
y con n xi = 1, se cumple que
i=1
n
n
x
i=1 xi
i
.

1 xi
n1
i=1

34

Desigualdades Numricas
e
x
1x

Usaremos el hecho que la funcin f (x) =


o
(x) > 0.
debe a que f
1
n

n
i=1

xi
1
=
n
1 xi

n
i=1

f (xi ) f

i=1

es convexa en (0, 1), esto se

1
xi
n

=f

1
n

1
=
,
n n1

por lo tanto,
n
i=1

Bastar ahora ver que


a
de Cauchy-Schwarz,

xi
n

.
1 xi
n1

n
i=1 xi

n
i=1

xi

n, pero esto se sigue de la desigualdad

n
n.
i=1 1 =

n
i=1 xi

Ejemplo 1.5.13 (Hungr


a-Israel, 1999) Sean k y l dos enteros positivos dados,
y sean aij , 1 i k y 1 j l, kl nmeros positivos dados. Muestre que si
u
q p > 0 entonces

q
p

ap
ij
j=1

i=1

1
q

i=1

l
j=1

1
p p
q

aq .
ij

Denamos bj = k ap , para j = 1, 2, . . . , l, y denotemos la parte izquierda


i=1 ij
de la desigualdad a probar por L y su parte derecha por R. Entonces
l

Lq =

q
p
bj

j=1
l

qp

ap
ij

bj p

i=1

j=1
k

=
i=1

qp
p

bj
j=1

ap .
ij

1.5 Funciones convexas

35

Usando la desigualdad de Hlder obtenemos


o

Lq

i=1

=
i=1

qp

bj p
l

j=1

qp
q

p
bj

qp
q

bj

k
i=1

qp
q

j=1

q
p

j=1

q
qp

l
j=1
l
j=1

l
j=1

p
q
q
aij

p
q
q

(ap ) p
ij

p
q
q
aij
= Lqp Rp .

La desigualdad L R se sigue dividiendo ambos lados de Lq Lqp Rp entre


Lqp y tomando la ra p-sima.
z e
Ejercicio 1.77 (i) Para a, b R+ , con a + b = 1, muestre que
a+

1
a

+ b+

1
b

25
.
2

(ii) Para a, b, c R+ , con a + b + c = 1, muestre que


1
a+
a

1
+ b+
b

1
+ c+
c

100
.
3

Ejercicio 1.78 Para 0 a, b, c 1, muestre que


a
b
c
+
+
+ (1 a)(1 b)(1 c) 1.
b+c+1 c+a+1 a+b+1

Ejercicio 1.79 (Rusia, 2000) Demuestre que para nmeros reales x, y con
u
0 x, y 1, se tiene que

1
+
1 + x2

1
1+

y2

2
.
1 + xy

36

Desigualdades Numricas
e

Ejercicio 1.80 Muestre que la funcin f (x) = sen x es cncava en el intervalo


o
o
[0, ]. Utilice lo anterior para vericar que los ngulos A, B, C de un tringulo
a
a

3
cumplen con sen A + sen B + sen C 2 3.
Ejercicio 1.81 Si A, B, C, D son ngulos en el intervalo [0, ], entonces:
a
(i) sen A sen B sen2

A+B
2

y la igualdad se cumple si y slo si A = B.


o

(ii) sen A sen B sen C sen D sen4


(iii) sen A sen B sen C sen3

A+B+C+D
4

A+B+C
3

Si adems A, B, C son los ngulos internos de un tringulo,


a
a
a

(iv) sen A sen B sen C 3 3.


8
(v) sen A sen B sen C 1 .
2
2
2
8
(vi) sen A + sen B + sen C = 4 cos A cos B cos C .
2
2
2
Ejercicio 1.82 (Desigualdad de Bernoulli) (i) Para todo nmero real x > 1
u
y todo entero positivo n, se cumple (1 + x)n 1 + nx.
(ii) Use la desigualdad para dar otra demostracin de la desigualdad M GM A.
o

Ejercicio 1.83 (Desigualdad de Schr) Si x, y, z son reales positivos y n es


u
un entero positivo, se cumple
xn (x y)(x z) + y n (y z)(y x) + z n (z x)(z y) 0.
Para el caso n = 1, la desigualdad puede tomar una de las siguientes formas:
(a) x3 + y 3 + z 3 + 3xyz xy(x + y) + yz(y + z) + zx(z + x).
(b) xyz (x + y z)(y + z x)(z + x y).
(c) Si x + y + z = 1, 9xyz + 1 4(xy + yz + zx).
Ejercicio 1.84 (Canad, 1992) Cualesquiera tres nmeros reales no negativos
a
u
x, y y z cumplen
x(x z)2 + y(y z)2 (x z)(y z)(x + y z).

1.5 Funciones convexas

37

Ejercicio 1.85 Si a, b y c son reales positivos, pruebe que


b
c
9
a
+
+

.
2
2
2
(b + c)
(c + a)
(a + b)
4(a + b + c)

Ejercicio 1.86 Sean a, b y c nmeros reales positivos, pruebe que


u
1+

6
3

.
ab + bc + ca
a+b+c

Ms an, si abc = 1, pruebe que


a u
1+

3
6

.
a+b+c
ab + bc + ca

Ejercicio 1.87 (Desigualdad entre medias potenciales) Sean x1 , x2 , . . . ,


xn nmeros reales positivos y sean t1 , t2 , . . . , tn nmeros reales positivos tales
u
u
que su suma sea 1. Sean r y s dos nmeros reales positivos distintos de cero
u
tal que r > s. Muestre que
1

(t1 xr + + tn xr ) r (t1 xs + + tn xs ) s ,
1
n
1
n
la igualdad se da si y slo si x1 = x2 = = xn .
o
Ejercicio 1.88 (Dos extensiones de la desigualdad de Hlder.) Sean x1 ,
o
x2 , . . . , xn , y1 , y2 , . . . , yn , z1 , z2 , . . . , zn nmeros reales positivos.
u
1
(i) Si a, b, c son nmeros reales positivos tales que a + 1 = 1 , entonces
u
b
c
1
c

(xi yi )c

i=1

1
a

1
b

xi a

yi b

i=1

(ii) Si a, b, c son nmeros reales positivos tales que


u

i=1

1
a

xi yi zi

1
b

1
1
1
a+b+c

i=1

zi c
i=1

= 1, entonces

1
c

yi b

xi a
i=1

i=1

38

Desigualdades Numricas
e

Ejercicio 1.89 (Desigualdad de Popoviciu) Si I es un intervalo y f : I R


es una funcin convexa, entonces, para a, b, c I, muestre que
o
2
f
3

a+b
2

+f

b+c
2

c+a

2
f (a) + f (b) + f (c)

+f
3

+f

a+b+c
3

Ejercicio 1.90 Sean a, b, c nmeros reales no negativos, muestre que:


u

3
(i) a2 + b2 + c2 + 3 a2 b2 c2 2(ab + bc + ca).
(ii) a2 + b2 + c2 + 2abc + 1 2(ab + bc + ca).
Ejercicio 1.91 Sean a, b, c nmeros reales positivos, muestre que
u
b+c c+a a+b
+
+
a
b
c

1.6.

a
b
c
+
+
b+c c+a a+b

Una desigualdad til


u

Primero, estudiemos dos identidades algebraicas muy utiles, que se pueden de


ducir al considerar un factor especial de a3 + b3 + c3 3abc.
Sea P el polinomio cbico
u
P (x) = x3 (a + b + c)x2 + (ab + bc + ca)x abc.
Como a, b, c satisfacen la ecuacin P (x) = 0, se tiene que
o
a3 (a + b + c)a2 + (ab + bc + ca)a abc = 0,
b3 (a + b + c)b2 + (ab + bc + ca)b abc = 0,

c3 (a + b + c)c2 + (ab + bc + ca)c abc = 0.

Sumando estas tres igualdades, obtenemos


a3 + b3 + c3 3abc = (a + b + c)(a2 + b2 + c2 ab bc ca).

(1.7)

La identidad anterior implica el siguiente resultado, si a + b + c = 0, entonces


a3 + b3 + c3 = 3abc.

1.6 Una desigualdad util

39

Tambin tenemos que la expresin


e
o
a2 + b2 + c2 ab bc ca
puede ser escrita como
1
a2 + b2 + c2 ab bc ca = [(a b)2 + (b c)2 + (c a)2 ].
2

(1.8)

De esta forma, obtenemos otra versin de la identidad (1.7),


o
1
a3 + b3 + c3 3abc = (a + b + c)[(a b)2 + (b c)2 + (c a)2 ].
2

(1.9)

Esta presentacin de la identidad implica una demostracin ms corta de la


o
o
a
desigualdad M G M A para tres variables. De (1.9), es claro que si a, b, c son
nmeros positivos, entonces a3 + b3 + c3 3abc. Ahora, si x, y, z son nmeros
u
u

3
3 y y c = 3 z, podemos deducir que
positivos, tomando a = x, b =
x+y+z

3 xyz,
3
con igualdad si y slo si x = y = z.
o
Observe que se puede utilizar la identidad (1.8) para dar otra demostracin del
o
ejercicio 1.27.
Ejercicio 1.92 Para nmeros reales x, y, z, muestre que
u
x2 + y 2 + z 2 |xy + yz + zx|.

Ejercicio 1.93 Para nmeros positivos reales a, b, c, muestre que


u
a2 + b2 + c2
1 1 1
+ +
.
a b
c
abc

Ejercicio 1.94 Si x < y < z son nmeros reales, muestre que


u
(x y)3 + (y z)3 + (z x)3 > 0.

40

Desigualdades Numricas
e

Ejercicio 1.95 Sean a, b, c las longitudes de los lados de un tringulo, muestre


a
que
3
3
3
3 a + b + c + 3abc
mx {a, b, c}.
a
2
Ejercicio 1.96 (Rumania, 2007) Para nmeros reales no-negativos x, y, z,
u
muestre que
x3 + y 3 + z 3
3
xyz + |(x y)(y z)(z x)|.
3
4
Ejercicio 1.97 (UK, 2008) Encuentra el m
nimo de x2 + y 2 + z 2 , donde x, y,
z son nmeros reales que satisfacen que x3 + y 3 + z 3 3xyz = 1.
u
Una desigualdad simple, que puede ser utilizada para demostrar un gran nmero
u
de desigualdades algebraicas, es la siguiente.
Teorema 1.6.1 (Una desigualdad util) Si a, b, x, y son nmeros reales y x,

u
y son positivos, entonces se tiene la siguiente desigualdad
(a + b)2
a2 b2
+

.
x
y
x+y

(1.10)

Demostracin. La demostracin es muy simple. Simplicando, podemos exo


o
presar la desigualdad como
a2 y(x + y) + b2 x(x + y) (a + b)2 xy,
la cual se reduce a la desigualdad (ay bx)2 0 que es obvia. Vemos que la
a
b
igualdad se tiene si y slo si ay = bx, esto es, si x = y .
o
Otra forma de justicar este resultado es utilizando la desigualdad de CauchySchwarz de la siguiente manera,
(a + b)2 =

b
a

x+ y
x
y

a2 b2
+
x
y

(x + y).

Usando el teorema dos veces, podemos extender la desigualdad para tres pares
de nmeros,
u
(a + b)2 c2
(a + b + c)2
a2 b2 c2
+
+

,
x
y
z
x+y
z
x+y+z

1.6 Una desigualdad util

41

y un simple argumento inductivo muestra que


a2
a2
(a1 + a2 + + an )2
a2
1
+ 2 + + n
,
x1 x2
xn
x1 + x2 + + xn

(1.11)

para todos los nmeros reales a1 , a2 , ... , an y x1 , x2 , ... , xn > 0, con igualdad
u
si y slo si
o
a1
a2
an
=
= =
.
x1
x2
xn
La desigualdad (1.11) es conocida tambin como la desigualdad de Cauchye
Schwarz en la forma de Engel o el Principio del m
nimo de Arthur Engel.
Como una primera aplicacin de esta desigualdad, damos otra demostracin de
o
o
la desigualdad de Cauchy-Schwarz,
a2 + a2 + + a2 =
1
2
n

a2 b2
a2 b2 a2 b2
n n
2 2
1 1
2 + b2 + + b2 ,
b1
n
2

entonces
a2 b2
a2 b2 a2 b2
(a1 b1 + a2 b2 + + an bn )2
1 1
+ 22 2 + + n2 n
.
bn
b2
b2
b2 + b 2 + + b 2
n
1
1
2
Luego, podemos concluir
(a2 + a2 + + a2 )(b2 + b2 + + b2 ) (a1 b1 + a2 b2 + + an bn )2
1
2
n
1
2
n
y la igualdad se tiene si y slo si
o
a1
a2
an
=
= =
.
b1
b2
bn
Existen otras formas de la desigualdad de Cauchy-Schwarz a la Engel, como las
del siguiente ejemplo.
Ejemplo 1.6.2 Sean a1 , ... , an , b1 , ... , bn nmeros reales positivos. Muestre
u
que:
(i)
(ii)

an
(a1 + + an )2
a1
+ +

.
b1
bn
a1 b1 + + an bn
1
an
a1
+ + 2
bn
a1 + + an
b2
1

a1
an
+ +
b1
bn

42

Desigualdades Numricas
e

Estas dos desigualdades se siguen de la desigualdad util aplicada de las siguientes

maneras:
an
a2
a2
(a1 + + an )2
a1
+ +
= 1 + + n
.
(i)
b1
bn
a1 b1
an bn
a1 b1 + + an bn
a2
1

a2
n

a1
an
1
b2
b2
(ii) 2 + + 2 = 1 + + n
bn
a1
an
a1 + + an
b1

a1
an
+ +
b1
bn

Ejemplo 1.6.3 (APMO, 1991) Sean a1 , ... , an , b1 , ... , bn , nmeros reales


u
positivos, tal que a1 + a2 + + an = b1 + b2 + + bn . Muestre que
a2
1
a2
n
1
+ +
(a1 + + an ).
a1 + b1
an + bn
2
Observemos que (1.11) implica
a2
(a1 + a2 + + an )2
a2
n
1
+ +

a1 + b1
an + bn
a1 + a2 + + an + b1 + b2 + + bn
=

(a1 + a2 + + an )2
2(a1 + a2 + + an )

1
= (a1 + a2 + + an ).
2
El siguiente ejemplo es una demostracin de la desigualdad entre la media
o
cuadrtica y la media aritmtica.
a
e
Ejemplo 1.6.4 (Desigualdad media cuadrtica-media aritmtica) Para
a
e
nmeros reales positivos x1 , ... , xn , se tiene que
u
x1 + x2 + + xn
x2 + x2 + + x 2
n
1
2

.
n
n
Observemos que usando (1.11), se tiene que
x2 + x2 + + x2
(x1 + x2 + + xn )2
n
2
1

n
n2
y sta implica la desigualdad de arriba.
e
En algunos casos los numeradores no son cuadrados, pero un simple truco nos
permite escribirlos como cuadrados y entonces podemos aplicar la desigualdad.
El siguiente ejemplo es una primera aplicacin de este truco, la cual nos permite
o
dar una demostracin ms corta del ejemplo 1.4.9.
o
a

1.6 Una desigualdad util

43

Ejemplo 1.6.5 (IMO, 1995) Sean a, b, c nmeros positivos tal que abc = 1.
u
Demuestre que
1
1
3
1
+ 3
+ 3
.
+ c) b (a + c) c (a + b)
2

a3 (b
Observemos que

1
1
1
1
1
1
a2
b2
c2
+
+
=
+
+
a3 (b + c) b3 (c + a) c3 (a + b)
a(b + c) b(c + a) c(a + b)

1
( a + 1 + 1 )2
ab + bc + ca
b
c
=
2(ab + bc + ca)
2abc

3 3 (abc)2
3
= ,
2
2

donde la primera desigualdad se sigue de (1.11) y la segunda es una consecuencia


de la desigualdad M G M A.
Otro ejemplo del uso de la desigualdad (1.11) es la siguiente demostracin de
o
la desigualdad de Nesbitt.
Ejemplo 1.6.6 (Desigualdad de Nesbitt) Para a, b, c R+ , se tiene que
b
c
3
a
+
+
.
b+c c+a a+b
2
b
Multipliquemos los tres trminos de la izquierda de la desigualdad por a , b , c ,
e
a
c
respectivamente, y ahora usemos la desigualdad (1.11) para obtener

b2
c2
(a + b + c)2
a2
+
+

.
a(b + c) b(c + a) c(a + b)
2(ab + bc + ca)
De la ecuacin (1.8), obtenemos a2 + b2 + c2 ab bc ca 0 y entonces
o
2 3(ab + bc + ca). Por lo tanto
(a + b + c)
a
b
c
(a + b + c)2
3
+
+

.
b+c c+a a+b
2(ab + bc + ca)
2
Ejemplo 1.6.7 (Repblicas Checa y Eslovaca, 1999) Para nmeros reales pou
u
sitivos a, b y c, muestre la desigualdad
b
c
a
+
+
1.
b + 2c c + 2a a + 2b

44

Desigualdades Numricas
e

Observemos que
a
b
c
a2
b2
c2
+
+
=
+
+
.
b + 2c c + 2a a + 2b
ab + 2ca bc + 2ab ca + 2bc
Entonces usando (1.11) tenemos que
b2
c2
(a + b + c)2
a2
+
+

1,
ab + 2ca bc + 2ab ca + 2bc
3(ab + bc + ca)
donde la ultima desigualdad se sigue como en el ejemplo anterior.

Ejercicio 1.98 (Sudfrica, 1995) Para nmeros positivos a, b, c, d, muestre


a
u
que
1 1 4 16
64
+ + +

.
a b
c
d
a+b+c+d
Ejercicio 1.99 Sean a y b nmeros reales positivos, muestre que
u
8(a4 + b4 ) (a + b)4 .

Ejercicio 1.100 Sean x, y, z nmeros reales positivos, muestre que


u
2
2
9
2
+
+

.
x+y y+z z+x
x+y+z

Ejercicio 1.101 Sean a, b, x, y, z nmeros reales positivos, muestre que


u
x
y
z
3
+
+

.
ay + bz az + bx ax + by
a+b

Ejercicio 1.102 Sean a, b, c nmeros reales positivos, muestre que


u
a2 + b2 b2 + c2 c2 + a2
+
+
a + b + c.
a+b
b+c
c+a

Ejercicio 1.103 (i) Sean x, y, z nmeros reales positivos, muestre que


u
y
z
1
x
+
+
.
x + 2y + 3z y + 2z + 3x z + 2x + 3y
2

1.6 Una desigualdad util

45

(ii) (Moldavia, 2007) Sean w, x, y, z nmeros reales positivos, muestre que


u
x
y
z
2
w
+
+
+
+ .
x + 2y + 3z y + 2z + 3w z + 2w + 3x w + 2x + 3y
3
Ejercicio 1.104 (Croacia, 2004) Sean x, y, z nmeros reales positivos, muestre
u
que
y2
z2
3
x2
+
+
.
(x + y)(x + z) (y + z)(y + x) (z + x)(z + y)
4
Ejercicio 1.105 Para nmeros reales positivos a, b, c, d, muestre que
u
a
b
c
d
+
+
+
2.
b+c c+d d+a a+b
Ejercicio 1.106 Sean a, b, c, d, e nmeros reales positivos, muestre que
u
a
b
c
d
e
5
+
+
+
+
.
b+c c+d d+e e+a a+b
2
Ejercicio 1.107 (i) Muestre que, para todos los nmeros reales positivos a, b,
u
c, x, y, z con a b c y z y x, se tiene la siguiente desigualdad
(a + b + c)3
a3 b3 c3
+
+

.
x
y
z
3(x + y + z)
(ii) (Belorusia, 2000) Muestre que, para todos los nmeros reales a, b, c, x, y,
u
z, se sigue la siguiente desigualdad
(a + b + c)3
a3 b3 c3
+
+

.
x
y
z
3(x + y + z)
Ejercicio 1.108 (Grecia, 2008) Para x1 , x2 , . . . , xn nmeros enteros positivos,
u
muestre que
x2 + x2 + + x2
n
2
1
x1 + x2 + + xn

kn
t

x1 x2 x n ,

donde k = mx {x1 , x2 , . . . , xn } and t = m {x1 , x2 , . . . , xn }. Bajo que


a
n
condiciones es vlida la igualdad?
a

46

1.7.

Desigualdades Numricas
e

La estrategia de sustitucin
o

Una estrategia util para resolver problemas de desigualdades es el uso de susti


tuciones. Es decir, sustituir unas variables por otras de manera que el problema
tenga una expresin ms amable para trabajar. Con una sustitucin conveniente
o
a
o
se puede lograr, por ejemplo, que cambien un poco los trminos dif
e
ciles de
manejar, simplicar expresiones o bien reducir trminos. An cuando no hay
e
u
recetas para hacer esto, en esta seccin se dan algunas ideas de lo que puede
o
hacerse, como siempre lo mejor es trabajar con algunos ejemplos.
Una primera sugerencia es que para problemas donde hay una condicin extra,
o
hay que intentar utilizarla para simplicar la desigualdad. En el siguiente ejemplo
con la condicin extra se eliminan denominadores buscando hacer ms fcil el
o
a a
problema.
Ejemplo 1.7.1 Si a, b, c son nmeros reales positivos y menores que 1, con
u
a + b + c = 2, se tiene que
a
1a

b
1b

c
1c

8.

Despus de hacer la sustitucin x = 1 a, y = 1 b, z = 1 c, tenemos que


e
o
x + y + z = 3 (a + b + c) = 1, a = 1 x = y + z, b = z + x, c = x + y. Por
lo tanto, la desigualdad es ahora equivalente a
y+z
x

z+x
y

x+y
z

8,

y sta, a su vez, es equivalente a


e
(x + y)(y + z)(z + x) 8xyz.
Pero sta ultima desigualdad es inmediata. Basta aplicar tres veces la desiguale

dad M G M A de la siguiente manera (x + y) 2 xy (ver el ejercicio 1.26).

Puede suceder que la condicin debe usarse simplemente para dirigirse a la


o
solucin, como en los siguientes dos ejemplos.
o
Ejemplo 1.7.2 (Mxico, 2007) Si a, b, c son nmeros reales positivos que sae
u
tisfacen a + b + c = 1, muestre que

a + bc + b + ca + c + ab 2.

1.7 La estrategia de sustitucin.


o

47

Usando la condicin a + b + c = 1, se tiene que


o
a + bc = a(a + b + c) + bc = (a + b)(a + c),
luego, por la desigualdad M G M A,

a + bc =

(a + b)(a + c)

2a + b + c
.
2

Anlogamente,
a

2b + c + a
2c + a + b
c + ab
y
.
2
2
Por lo que, al sumar las tres desigualdades, se obtiene

b + ca

a + bc + b + ca + c + ab
2a + b + c 2b + c + a 2c + a + b
4a + 4b + 4c
+
+
=
= 2.
2
2
2
2
La igualdad se alcanza cuando a + b = a + c, b + c = b + a y c + a = c + b, lo
que nos lleva a que, a = b = c = 1 .
3

Ejemplo 1.7.3 Si a, b, c son nmeros reales positivos con ab + bc + ca = 1


u
muestre que,
a
b
c
3

+
+
.
2+1
2+1
2+1
2
a
b
c
Notemos que (a2 + 1) = a2 + ab + bc + ca = (a + b)(a + c), y, anlogamente,
a
b2 + 1 = (b + c)(b + a) y c2 + 1 = (c + a)(c + b). Ahora la desigualdad, que
estamos considerando, es equivalente a
a
+
(a + b)(a + c)

b
+
(b + c)(b + a)

c
3
.
2
(c + a)(c + b)

Por la desigualdad M G M A aplicada a cada sumando del lado izquierdo,


tenemos que
a
b
c
+
+

(a + b)(a + c)
(b + c)(b + a)
(c + a)(c + b)
a
b
c
a
b
c
1
1
1
+
+
+
+
+

2 a+b a+c
2 b+c b+a
2 c+a c+b

3
.
2

Los problemas muchas veces sugieren qu sustitucin debe hacerse. En el sie


o
guiente ejemplo la sustitucin nos permite convertir al menos a uno de los
o
trminos de la desigualdad en una expresin ms sencilla.
e
o
a

48

Desigualdades Numricas
e

Ejemplo 1.7.4 (India, 2002) Si a, b, c son nmeros reales positivos, muestre la


u
siguiente desigualdad
c+a a+b b+c
a b c
+ +
+
+
.
b c a
c+b a+c b+a
b
c
Al hacer, x = a , y = c , z = a , el lado izquierdo de la desigualdad es ahora muy
b
sencillo, x + y + z. Veamos como queda el lado derecho. El primer sumando se
modica as
,

1+ ab
1+ a
1 + xy
c+a
1x
c
bc
=
=
=
=x+
.
b
b
c+b
1+y
1+y
1+ c
1+ c
Anlogamente,
a
1y
a+b
=y+
a+c
1+z

b+c
1z
=z+
.
b+a
1+x

Ahora, la desigualdad es equivalente a tener


x1 y1 z1
+
+
0.
1+y
1+z 1+x
Solamente que, ahora, contamos con una condicin extra xyz = 1.
o
La desigualdad anterior la podemos rescribir como
(x2 1)(z + 1) + (y 2 1)(x + 1) + (z 2 1)(y + 1) 0

y est es equivalente a,
a

x2 z + y 2 x + z 2 y + x2 + y 2 + z 2 x + y + z + 3.
Pero, por la desigualdad M G M A, x2 z + y 2 x + z 2 y 3 3 x3 y 3 z 3 = 3.
1
Tambin, tenemos x2 + y 2 + z 2 3 (x + y + z)2 = x+y+z (x + y + z)
e
3

3 xyz(x + y + z) = x + y + z, donde la primera desigualdad se obtuvo de aplicar


la desigualdad (1.11).
Para hacer una sustitucin a veces es necesario preparar un poco el terreno
o

como muestra el siguiente ejemplo. Este tambin sirve para sealar que se puede
e
n
necesitar hacer ms de un proceso de sustitucin.
a
o
Ejemplo 1.7.5 Sean a, b, c nmeros reales positivos, muestre que se cumple
u
que
(a + b)(a + c) 2 abc(a + b + c).

1.7 La estrategia de sustitucin.


o

49

Al dividir entre a2 cada lado de la desigualdad y tomar x =


desigualdad se transforma en
(1 + x)(1 + y) 2

b
a,

y =

c
a,

la

xy(1 + x + y).

1
Ahora, al dividir entre xy ambos lados y hacer la sustitucin r = 1+ x , s = 1+ 1 ,
o
y
la desigualdad a demostrar ahora es

rs 2 rs 1.

Esta ultima, es equivalente a (rs 2)2 0, que es inmediata.

Es una situacin comn que los problemas tengan varias formas de resolverse,
o
u
resulta tambin que pueden aceptar varias sustituciones que ayuden a resolverlo.
e
Esto se muestra en el siguiente ejemplo.
Ejemplo 1.7.6 (Corea, 1998) Si a, b, c son nmeros reales positivos, tales que
u
a + b + c = abc, muestre que

1
1
1
3
+
+
.
2
2
2
2
1+a
1+b
1+c

1
o
Bajo la sustitucin x = a , y = 1 , z = 1 , la condicin a + b + c = abc, se
o
b
c
traduce en xy + yz + zx = 1 y la desigualdad es equivalente a

x
+
x2 + 1

y
y2 + 1

3
.
2
z2 + 1
z

Pero sta es la tercera desigualdad de esta seccin.


e
o
Otra posible sustitucin para resolver el ejemplo anterior es considerar la sustio
tucin a = tan A, b = tan B, c = tan C, como se tiene que tan A + tan B +
o
tan C = tan A tan B tan C entonces A + B + C = (o bien un mltiplo
u
de ). Pero como 1 + tan2 A = (cos A)2 , la desigualdad es equivalente a
3
a
a
cos A + cos B + cos C 2 que es vlida como se ver en el ejemplo 2.5.2.
Note que la desigualdad de Jensen no es posible aplicarla en este caso ya que
1
la funcin f (x) = 1+x2 no es cncava en R+ .
o
o
En el ejemplo anterior vimos que adems de las sustituciones algebricas hay
a
a
sustituciones trigonmetricas. Esto se muestra tambin en el siguiente ejemplo.
o
e
En las secciones 2.2 y 2.5 del siguiente cap
tulo trabajaremos con sustituciones
geomtricas.
e

50

Desigualdades Numricas
e

Ejemplo 1.7.7 (Rumania, 2002) Si a, b, c son nmeros reales del intervalo


u
(0, 1). Muestre que

abc + (1 a)(1 b)(1 c) < 1.


Al hacer la sustitucin a = cos2 A, b = 2 B, c = cos2 C, con A, B, C en el
o
cos

intervalo (0, ), se tiene que 1 a = 1 cos2 A = sen A, 1 b = sen B


2

y 1 c = senC. Por lo que la desigualdad es equivalente a


cos A cos B cos C + sen A sen B sen C < 1.
Para demostrar esta desigualdad bastar observar que,
a
cos A cos B cos C + sen A sen B sen C < cos A cos B + sen A sen B
= cos(A B) 1.
Ejercicio 1.109 Sean x, y, z nmeros reales positivos, muestre que
u
y3
z3
x3
+ 3
+ 3
1.
x3 + 2y 3 y + 2z 3 z + 2x3

Ejercicio 1.110 (Kazajastn, 2008) Sean x, y, z son nmeros reales positivos


a
u
que cumplen con xyz = 1, muestre que
1
1
1
3
+
+
.
yz + z zx + x xy + y
2

Ejercicio 1.111 (Rusia, 2004) Si n > 3 y x1 , x2 , ..., xn son nmeros reales


u
positivos, con x1 x2 xn = 1, muestre que
1
1
1
+
+ +
> 1.
1 + x1 + x1 x2 1 + x2 + x2 x3
1 + xn + xn x1

Ejercicio 1.112 (Polonia, 2006) Sean a, b, c nmeros reales positivos que


u
satisfacen ab + bc + ca = abc, muestre que
b4 + c4
c4 + a4
a4 + b4
+
+
1.
ab(a3 + b3 ) bc(b3 + c3 ) ca(c3 + a3 )

1.8 Teorema de Muirhead

51

Ejercicio 1.113 (Irlanda, 2007) Sean a, b, c nmeros reales positivos, muestre


u
que
1 bc ca ca
a2 + b2 + c2
a+b+c

+
+
.
3
3
3 a
b
b
Ejercicio 1.114 (Rumania, 2008) Sean a, b, c nmeros reales positivos, con
u
abc = 8, muestre que
a2 b2 c2
+
+
0.
a+1 b+1 c+1

1.8.

Teorema de Muirhead

En 1903, R.F. Muirhead public un art


o
culo acerca de algunos mtodos algebraie
cos usados en el estudio de identidades y desigualdades de funciones simtricas
e
algebraicas de n variables.

El consider expresiones algebraicas de la forma xa1 xa2 xan , luego analiz poo
o
n
1 2
linomios simtricos que conten estas expresiones, para dar un ordenen el ese
an
pacio de n-adas (a1 , a2 , ..., an ) que satisfac la condicin a1 a2 an .
an
o
Vamos a suponer que xi > 0 para toda 1 i n. Denotemos por
!

F (x1 , ..., xn )

la suma de los n! trminos obtenidos de evaluar F (x1 , ..., xn ) en todas las posie
bles permutaciones de (x1 , ..., xn ). Nosotros consideramos slo el caso particular
o
F (x1 , ..., xn ) = xa1 xa2 xan , con xi > 0, ai 0.
n
1 2
Escribimos [a] = [a1 , a2 , ..., an ] =
variables x, y, z > 0 se tiene

1
n!

[1, 1] = xy, [1, 1, 1] = xyz, [2, 1, 0] =

xa1 xa2 xan . Por ejemplo, para las


n
1 2

1 2
[x (y + z) + y 2 (x + z) + z 2 (x + y)].
3!

Es claro que [a] es invariante bajo cualquier permutacin de (a1 , a2 , ..., an ) y


o
entonces los dos conjuntos de a son del mismo tipo si ellos slo dieren por
o
reacomodo. Diremos que un valor de media del tipo [a] es una media simtrica,
e
(n1)!
n
1
por ejemplo, [1, 0, ..., 0] = n! (x1 + x2 + + xn ) = n i=1 xi es la media

52

Desigualdades Numricas
e

1
1
1

1
n!
1 1
n
n
n
aritmtica y [ n , n , ..., n ] = n! (x1 x2 xn ) = n x1 x2 xn es la media
e
geomtrica. Cuando a1 + a2 + + an = 1, [a] es una generalizacin comn
e
o
u
de la media aritmtica y de la media geomtrica.
e
e
Si a1 a2 an , b1 b2 bn , en general [b] no es comparable
con [a], en el sentido que exista una desigualdad entre ellos, vlida para todos
a
los nmeros reales no negativos x1 , x2 , . . ., xn .
u

Muirhead deseaba comparar los valores de los polinomios simtricos [a] y [b]
e
para cualquier valor no negativo de las variables en ambos polinomios.
Denamos de ahora en adelante (a) = (a1 , a2 , ..., an ).
Denicin 1.8.1 Decimos que (a) mayoriza a (b) y escribimos (b) (a), cuano
do (a) y (b) se pueden arreglar de tal forma que satisfacen las siguientes dos
condiciones:
n

ai .

bi =

(1)
i=1

i=1

(2)
i=1

bi

i=1

ai , para toda 1 < n.

Es claro que (a) (a) y que si (c) (b) y (b) (a) garantizan (c) (a).
Teorema 1.8.2 (Teorema de Muirhead) [b] [a] para cualesquiera valores
no negativos de las variables (x1 , x2 , ..., xn ) si y slo si (b) (a). La igualdad
o
slo se tiene cuando (b) y (a) son idnticos o cuando todos los xi son iguales.
o
e
Antes de pasar a la demostracin del teorema, la cual es un poco complicada,
o
veamos algunos ejemplos. Primero, es claro que [2, 0, 0] no puede ser comparado
con [1, 1, 1] ya que la primera condicin en la denicin 1.8.1 no se satisface,
o
o
pero podemos ver que [2, 0, 0] [1, 1, 0], lo cual es equivalente a
x2 + y 2 + z 2 xy + yz + zx.
De la misma forma, podemos ver que:
1. x2 + y 2 2xy [2, 0] [1, 1],
2. x3 + y 3 + z 3 3xyz [3, 0, 0] [1, 1, 1],
3. x5 + y 5 x3 y 2 + x2 y 3 [5, 0] [3, 2],
4. x2 y 2 + y 2 z 2 + z 2 x2 x2 yz + y 2 xz + z 2 xy [2, 2, 0] [2, 1, 1],

1.8 Teorema de Muirhead

53

donde todas las desigualdades son verdaderas si aceptamos el teorema de Muirhead.


Demostracin. (Teorema de Muirhead) Supongamos que [b] [a] para todos
o
los nmeros positivos x1 , x2 , . . ., xn . Tomando xi = x, para todo i, obtenemos
u
x

bi

= [b] [a] = x

ai

Esto slo puede ser verdadero para todo x si


o
bi = ai .
Ahora, sean x1 = x2 = = x = x, x+1 = = xn = 1 para x grande.
Como (b) y (a) estn en orden decreciente, los
a
ndices de las potencias ms
a
grandes de x en [b] y [a] son
b1 + b2 + + b , a1 + a2 + + a ,
respectivamente. Luego, es claro que la primera no puede ser mayor que la
segunda y esto prueba (2) en la denicin 1.8.1.
o
La demostracin en la otra direccin es ms complicada, y para esto necesitao
o
a
remos una nueva denicin y dos lemas.
o
Deniremos un tipo especial de transformacin lineal T de a, como sigue. Suo
ponga que ak > al , y escribimos
ak = + ,

al =

(0 < ).

Si ahora 0 < , entonces la transformacin T se dene como


o
T (ak ) = bk = + =


+
ak +
al
2
2

T (al ) = bl = =


+
ak +
al
2
2

T (a ) = a

( = k, = l).

Si (b) es la imagen de (a) bajo la transformacin T , escribimos b = T a. La


o
denicin no necesariamente implica que (a) o (b) estn en orden decreciente.
o
a
La suciencia de nuestra condicin de comparabilidad ser establecida, si podeo
a
mos demostrar los siguientes dos lemas.

Lema 1.8.3 Si b = T a entonces [b] [a], con igualdad slo cuando todos los
o
xi son iguales.

54

Desigualdades Numricas
e

Demostracin. Podemos reacomodar (a) y (b) de tal forma que k = 1, l = 2.


o
Entonces
[a] [b] = [ + , , x3 , ...] [ + , , x3 , ...]
1
xa3 xan (x+ x + x x+ )
=
n
1
2
1
2
! 3
2n!
1
xa3 xan (x+ x + x x+ )

n
1
2
1
2
! 3
2n!
1

=
(x1 x2 ) xa3 xan (x + x + )(x x2 ) 0
n
1
2
1
3
!
2n!
con igualdad slo cuando todos los xi son iguales.
o
Lema 1.8.4 Si (b) (a), con (b) no idntica a (a), entonces (b) puede ser
e
obtenida de (a) usando sucesivas aplicaciones de un nmero nito de transforu
maciones T .
Demostracin. Al nmero de diferencias a b distintas de cero, le llamao
u
remos la discrepancia entre (a) y (b). Si la discrepancia es cero, los conjuntos
son iguales. Demostremos el lema por induccin, suponiendo que es verdadero
o
cuando la discrepancia es menor que r y demostrando que es verdadero cuando
la discrepancia es r.
Supongamos entonces, que (b) (a) y que la discrepancia es r > 0. Como
n
n
(a b ) = 0, y no todas las diferencias son cero,
i=1 ai =
i=1 bi , y
deben existir diferencias positivas y negativas, y la primera que nos es cero
debe ser positiva por la segunda condicin de (b) (a). Entonces, podemos
o
encontrar k y l tal que
bk < ak , bk+1 = ak+1 , ... , bl1 = al1 , bl > al ,

(1.12)

esto es, al bl es la primera diferencia negativa y ak bk es la ultima diferencia

positiva que la precede.


Sea ak = + , al = , y dena por
= mx {|bk | , |bl |}.
a
Entonces 0 < , ya que ak > al . Tambin, uno o el otro de bl = y
e
bk = , es verdadero, ya que bk bl , y < , adems bk < ak y bl > al .
a
Luego, 0 < .
Ahora escribimos a = +, a = , a = x ( = k, = l). Si bk = ,

k
l
a = bk y si bl = entonces a = bl . Como cada una de las parejas ak ,
k
l

1.8 Teorema de Muirhead

55

bk y al , bl contribuye con una unidad a la discrepancia r entre (b) y (a), la


discrepancia entre (b) y (a ) es menor, la cual es r 1 o r 2.
Enseguida, comparemos la denicin de (a ) con la denicin de T y, observando
o
o
) proviene de (a) por medio de
que 0 < , podemos deducir que (a
una transformacin T .
o
Finalmente, (b) (a ). Para demostrar esto, debemos vericar que las dos
condiciones de son satisfechas y que el orden de (a ) es no creciente. Para la
primera condicin, tenemos
o
+

a
l

i=1

a .
i

ai =

bi =

= 2 = ak + al ,

a
k

i=1

i=1

Para la segunda condicin, necesitamos demostrar que


o
b1 + b2 + + b a + a + + a .
1
2

Ahora, esto es verdadero si < k o l, como se puede ver de la denicin


o
de (a ) y usando la segunda condicin de (b) (a). Esto es cierto para = k,
o
ya que es cierto para = k 1 y bk a , y es verdadero para k < < l, ya
k
que es vlido para = k y los elementos que aparecen en b y a son idnticos.
a
e
Finalmente, podemos observar que
bk + |bk | + = a ,
k
bl |bl | = a
l
y entonces, usando (1.12),

a
k1 = ak1 ak = + > + = ak bk bk+1 = ak+1 = ak+1 ,

a = al1 = bl1 bl a = > = al al+1 = a .


l+1
l
l1

As las desigualdades que afectan a a son las buscadas.

Por lo tanto, hemos demostrado que (b) (a ), un conjunto que proviene de


(a) usando una transformacin T que tiene una discrepancia con (b) menor a
o
r. Esto completa la demostracin del lema y del teorema de Muirhead.
o
La demostracin del teorema de Muirhead nos muestra, por medio de un uso
o
repetido de la transformacin T , como las diferencias entre dos medias compao
rables puede ser descompuesta como una suma de trminos que son obviamente
e
positivos. Podemos deducir, de este ultimo resultado, una nueva demostracin

o
de la desigualdad M G M A.

56

Desigualdades Numricas
e

Ejemplo 1.8.5 (Desigualdad M G M A)


y1 + y2 + + yn

n
.
y1 y2 yn
n
Notemos que la desigualdad M G M A es equivalente a
x1 x2 xn

xn + x n + + x n
n
2
1
n

donde xi = n yi .
Ahora, observemos que
1
n

n
i=1

xn = [n, 0, 0, ..., 0] y x1 x2 xn = [1, 1, ..., 1].


i

Por el teorema de Muirhead, podemos deducir que


[1, 1, ..., 1] [n, 0, 0, ..., 0].
Enseguida, daremos otra demostracin de la desigualdad M G M A, siguiendo
o
las ideas de la demostracin del teorema de Muirhead, para ilustrar como trabaja.
o
1
n

n
i=1

xn (x1 x2 xn ) = [n, 0, 0, ..., 0] [1, 1, ..., 1]


i
= ([n, 0, 0, ..., 0] [n 1, 1, 0, ..., 0]) +

+([n 1, 1, 0, ..., 0] [n 2, 1, 1, 0, ..., 0]) +

+([n 2, 1, 1, 0, ..., 0] [n 3, 1, 1, 1, 0, ..., 0]) +

+... + ([2, 1, 1, ..., 1] [1, 1, ..,1])


1
n1
=
(xn1 x2 )(x1 x2 )+
! 1
2n!
n2
n2
+
(x1 x2 )(x1 x2 )x3 +
!

n3
n3
(x1 x2 )(x1 x2 )x3 x4 + ... .

Como (x x )(xr xs ) > 0, a menos que xr = xs , la desigualdad se sigue.


r
s
Ejemplo 1.8.6 Si a, b son nmeros reales positivos, entonces
u
a2
+
b

b2
a + b.
a

1.8 Teorema de Muirhead


Sea x =

a, y =

57

b, simplicando vemos que se tiene que demostrar


x3 + y 3 xy(x + y).

Por el teorema de Muirhead, tenemos que


1
1
[3, 0] = (x3 + y 3 ) xy(x + y) = [2, 1],
2
2
luego, el resultado se sigue.
Ejemplo 1.8.7 Si a, b, c son nmeros reales no negativos, muestre que
u
a3 + b3 + c3 + abc

1
(a + b + c)3 .
7

No es dif ver que


cil
(a + b + c)3 = 3[3, 0, 0] + 18[2, 1, 0] + 36[1, 1, 1].
Luego, demostraremos que
1
3[3, 0, 0] + 6[1, 1, 1] (3[3, 0, 0] + 18[2, 1, 0] + 36[1, 1, 1]),
7
esto es,

18
36
[3, 0, 0] + 6
7
7

[1, 1, 1]

18
36
([3, 0, 0] [2, 1, 0]) + 6
7
7

18
[2, 1, 0]
7
[1, 1, 1] 0.

Esto se sigue usando las desigualdades [3, 0, 0] [2, 1, 0] y [1, 1, 1] 0.


Ejemplo 1.8.8 Si a, b, c son nmeros reales no negativos, muestre que
u
a+b+c

a3
b3
c3
a2 + b2 b2 + c2 c2 + a2
+
+

+
+ .
2c
2a
2b
bc ca ab

Las desigualdades son equivalentes a


2(a2 bc+ ab2 c+ abc2 ) ab(a2 + b2 )+ bc(b2 + c2 )+ ca(c2 + a2 ) 2(a4 + b4 + c4 ),
que a su vez es equivalente a [2, 1, 1] [3, 1, 0] [4, 0, 0]. Usando el teorema
de Muirhead obtenemos el resultado.

58

Desigualdades Numricas
e

Ejercicio 1.115 Cualesquiera tres nmeros reales a, b y c, satisfacen que


u
a5 + b5 + c5 a3 bc + b3 ca + c3 ab.

Ejercicio 1.116 (IMO, 1961) Sean a, b, c las longitudes de los lados de un


tringulo y sea (ABC) su rea. Muestre que
a
a

4 3(ABC) a2 + b2 + c2 .

Ejercicio 1.117 Sean a, b, c nmeros reales positivos, muestre que


u
a
b
c
9
+
+

.
(a + b)(a + c) (b + c)(b + a) (c + a)(c + b)
4(a + b + c)

Ejercicio 1.118 (IMO, 1964) Sean a, b, c nmeros reales positivos. Muestre


u
que
a3 + b3 + c3 + 3abc ab(a + b) + bc(b + c) + ca(c + a).
Ejercicio 1.119 (Lista corta Iberoamericana, 2003) Sean a, b, c nmeros reales
u
positivos. Muestre que
b3
c3
a3
+ 2
+ 2
a + b + c.
b2 bc + c2 c ca + a2 a ab + b2
Ejercicio 1.120 (Lista corta IMO, 1998) Sea a, b, c nmeros reales positivos
u
tales que abc = 1. Muestre que
b3
c3
3
a3
+
+
.
(1 + b)(1 + c) (1 + c)(1 + a) (1 + a)(1 + b)
4

Cap
tulo 2

Desigualdades Geomtricas
e

2.1.

Dos desigualdades bsicas


a

Las dos desigualdades bsicas en geometr se reeren a los tringulos. Una de


a
a
a
ellas es la desigualdad del tringulo y es la que ms adelante vamos a referir
a
a
como D1, la segunda no es propiamente una desigualdad, es una observacin
o
importante de la geometr de los tringulos, donde se seala que el lado mayor
a
a
n
del tringulo es el opuesto al ngulo mayor y la que denotamos por D2.
a
a
D1. Si A, B y C son puntos del plano, entonces
AB + BC AC.
Adems, la igualdad se alcanza si y slo si B est en el segmento AC.
a
o
a
D2. En un tringulo el lado mayor es el que se opone al ngulo mayor y rec
a
a
procamente.
As si en el tringulo ABC se tiene que A > B, entonces BC > CA.
,
a
Ejercicio 2.1 (i) Si a, b, c son nmeros positivos con a < b + c, b < c + a y
u
c < a + b, entonces se puede encontrar un tringulo con lados de longitud a, b
a
y c.
(ii) Para poder formar un tringulo con lados de longitud a b c, basta que
a
c < a + b.
(iii) Se puede construir un tringulo con segmentos de longitud a, b, c si y slo
a
o
si existen nmeros positivos x, y, z con a = x + y, b = y + z y c = z + x.
u

60

Desigualdades Geomtricas
e

Ejercicio 2.2 (i) Si con segmentos de longitud a < b < c se puede construir

un tringulo, entonces con segmentos de longitud a < b < c se puede


a
armar un tringulo.
a
(ii) El rec
proco de (i) es falso.
(iii) Si con segmentos de longitud a < b < c se puede armar un tringulo,
a
1
1
1
entonces con segmentos de longitud a+b , b+c y c+a se puede formar un
tringulo.
a
Ejercicio 2.3 Considere los segmentos de longitud a, b, c, d y e, de manera
que con cualesquiera tres de ellos se puede formar un tringulo, muestre que
a
hay tres de ellos que forman un tringulo acutngulo.
a
a
Algunas veces la clave para resolver un problema est en identicar ciertas
a
cantidades con magnitudes geomtricas, como sucede en el siguiente ejemplo.
e
Ejemplo 2.1.1 Si a, b, c son nmeros positivos con a2 + b2 ab = c2 , muestre
u
que (a b)(b c) 0.
Como c2 = a2 + b2 ab = a2 + b2 2ab cos 60 , podemos pensar que a, b,
c son las longitudes de los lados de un tringulo tal que la medida del ngulo
a
a
opuesto al lado de longitud c, es igual a 60 . Los ngulos del tringulo ABC
a
a
cumplen, A 60 y B 60 , o bien A 60 y B 60 . Por lo tanto,
por la propiedad D2 tenemos que a c b o bien a c b. En cualquier
caso sucede que (a b)(b c) 0.
Observacin 2.1.2 Tambin se puede resolver el ejemplo anterior sin tener que
o
e
identicar a, b y c con las longitudes de los lados de un tringulo.
a
Supongamos primero que a b, el que a2 + b2 ab = c2 implica que a(a b) =
c2 b2 = (c b)(c + b), por lo que c b 0 y entonces (a b)(b c) 0.
Anlogamente, a b implica c b 0, lo que nos lleva a que
a
(a b)(b c) 0.
Otro ejemplo donde no es inmediato que se trate de una desigualdad geomtrica,
e
o que el uso de la geometr ayude, se muestra a continuacin.
a
o
Ejemplo 2.1.3 Si a, b, c son nmeros positivos, entonces
u
a2 + ac + c2

a2 ab + b2 +

b2 bc + c2 .

2.1 Dos desigualdades bsicas


a

61

Los radicales sugieren usar la ley de los cosenos con ngulos de 120 y de 60
a
como sigue: a2 + ac + c2 = a2 + c2 2ac cos 120 , a2 ab + b2 = a2 + b2
2ab cos 60 y b2 bc + c2 = b2 + c2 2bc cos 60 .
a

60
60

b
B

c
C

Entonces, si consideramos un cuadriltero ABCD, con ADB = BDC =


a
60 y ADC = 120 , de manera que
AD = a, BD = b y CD = c, tene

2 ab + b2 , BC =
mos que AB = a
b2 bc + c2 y CA = a2 + ac + c2 .
La desigualdad que debemos mostrar es la desigualdad del tringulo para el
a
tringulo ABC.
a
Ejercicio 2.4 Sea ABC un tringulo con A > B, muestre que BC > 1 AB.
a
2
Ejercicio 2.5 Sea ABCD un cuadriltero convexo, muestre que:
a
(i) Si AB + BD < AC + CD entonces AB < AC.
1
(ii) Si A > C y D > B entonces BC > 2 AD.

Ejercicio 2.6 Si a1 , a2 , a3 , a4 , a5 son las longitudes de los lados de un


pentgono convexo y si d1 , d2 , d3 , d4 , d5 son las longitudes de sus diagonales,
a
muestre que
a1 + a2 + a3 + a4 + a5
1
<
< 1.
2
d1 + d2 + d3 + d4 + d5
a
Ejercicio 2.7 La longitud ma de la mediana AA de un tringulo ABC cumple
que ma > b+ca .
2
Ejercicio 2.8 Si la longitud de la mediana AA de un tringulo ABC cumple
a
ma > 1 a, muestre que BAC < 90o .
2
Ejercicio 2.9 Si AA es la mediana del tringulo ABC y si AB < AC, entonces
a
> A AC.
BAA

62

Desigualdades Geomtricas
e

Ejercicio 2.10 Si ma , mb y mc son las longitudes de las medianas de un


tringulo con lados de longitudes a, b y c, respectivamente. Muestre que se
a
puede armar un tringulo de lados con longitudes ma , mb y mc , y que
a
3
(a + b + c) < ma + mb + mc < a + b + c.
4
Ejercicio 2.11 (Desigualdad de Ptolomeo) Para un cuadriltero convexo
a
ABCD se cumple que AC BD AB CD + BC DA.
La igualdad se da si y slo si ABCD es un cuadriltero c
o
a
clico.
Ejercicio 2.12 Sea ABCD un cuadriltero c
a
clico. Muestre que AC > BD si
y slo si (AD BC)(AB DC) > 0.
o
Ejercicio 2.13 (Problema de Pompeiu) Sea ABC un tringulo equiltero y
a
a
P un punto del plano que no pertenece al circunc
rculo de ABC. Muestre que
P A, P B y P C son las longitudes de los lados de un tringulo.
a
Ejercicio 2.14 Si ABCD es un paralelogramo, muestre que
AB 2 BC 2 < AC BD.
Ejercicio 2.15 Si a, b y c son las longitudes de los lados de un tringulo, ma ,
a
mb y mc representan las longitudes de las medianas y R el circunradio. Muestre
que:
(i)

a2 + b2 b2 + c2 c2 + a2
12R.
+
+
mc
ma
mb

(ii) ma (bc a2 ) + mb (ca b2 ) + mc (ab c2 ) 0.


Ejercicio 2.16 Sea ABC un tringulo cuyos lados tienen longitudes a, b y c.
a
Supongamos que c > b, muestre que
3
1
(c b) < mb mc < (c b),
2
2
donde mb y mc son las longitudes de las medianas.
Ejercicio 2.17 (Irn, 2005) Sea ABC un tringulo con A = 90 . Sea D la
a
a
interseccin de la bisectriz interna del A con el lado BC y sea Ia el centro de
o
la circunferencia excrita al tringulo ABC opuesta al vrtice A. Muestre que
a
e

AD
2 1.
DIa

2.2 Desigualdades entre los lados de un tringulo


a

2.2.

63

Desigualdades entre los lados de un tringulo


a

Las desigualdades entre las longitudes de los lados de un tringulo aparecen


a
frecuentemente como problemas en los concursos. Un tipo de problemas son
los que piden demostrar una desigualdad que satisfacen las longitudes de los
lados del tringulo y no hay ms elementos geomtricos involucrados, como es
a
a
e
el siguiente.
Ejemplo 2.2.1 Las longitudes de los lados a, b y c de un tringulo cumplen
a
que
a (b + c a) < 2bc.
Como la desigualdad es simtrica en b y c podemos suponer, sin perder generae
lidad, que c b. Estudiaremos la desigualdad en casos:
Caso 1. a b.
Por ser la longitud de los lados de un tringulo tenemos que b < a + c; luego
a
b + c a = b a + c < c + c = 2c

2bc
.
a

Caso 2. a b.
Aqu resulta que b a 0, y como a < b + c 2b, tenemos que

b+ca=c+bac <

2bc
.
a

Otro tipo de problemas para las longitudes de los lados de un tringulo, son
a
aquellos donde se pide demostrar que cierta relacin entre los nmeros a, b y c
o
u
es suciente para construir un tringulo con lados de longitud a, b y c.
a
2

Ejemplo 2.2.2 (i) Si a, b, c son nmeros positivos y satisfacen, a2 + b2 + c2 >


u
2 a4 + b4 + c4 , entonces a, b y c son las longitudes de los lados de un tringulo.
a
(ii) Si a, b, c, d son nmeros positivos y satisfacen
u
a2 + b2 + c2 + d2

> 3 a4 + b4 + c4 + d4 ,

entonces con cualesquiera tres de ellos se puede formar un tringulo.


a
Para la parte (i) basta observar que,
2

a2 + b2 + c2 2 a4 + b4 + c4 = (a+b+c)(a+bc)(ab+c)(a+b+c) > 0
y notar que de estos factores ninguno es negativo, compare con el ejemplo 1.2.5.

64

Desigualdades Geomtricas
e

Para la parte (ii), tenemos que


3 a4 + b4 + c4 + d4 < a2 + b2 + c2 + d2
=

a2 + b2 + c2 a2 + b2 + c2
+
+ d2
2
2
a2 + b2 + c2
2

a2 + b2 + c2
2

+ d4

La segunda desigualdad se sigue de la desigualdad de Cauchy-Schwarz; luego


(a2 +b2 +c2 ) 2
a4 + b4 + c4 < 2
. Por la primera parte tenemos que con a, b y c se
4
puede formar un tringulo. Como el argumento utilizado es simtrico en a, b, c
a
e
y d, tenemos el resultado.
Existe una tcnica que ayuda a transformar una desigualdad entre las longitudes
e
de los lados de un tringulo en una desigualdad entre nmeros positivos (desde
a
u
luego relacionados con los lados), llamada la transformacin de Ravi.
o
Si el inc
rculo (I, r) del tringulo ABC es tangente a los lados BC, CA y
a
AB en los puntos X, Y y Z, respectivamente, tenemos que, x = AZ = Y A,
y = ZB = BX, z = XC = CY .
A
x

x
Y

Z
I

y
B

C
z
X
Es claro que a = y + z, b = z + x, c = x + y, x = s a, y = s b y z = s c,
donde s = a+b+c .
2
y

Veamos como usar la transformacin de Ravi en el siguiente ejemplo.


o
Ejemplo 2.2.3 Las longitudes de los lados a, b y c de un tringulo cumplen
a
que
(b + c a)(c + a b)(a + b c) abc.
Primero tenemos que
(b + c a)(c + a b)(a + b c) = 8(s a)(s b)(s c) = 8xyz,

2.2 Desigualdades entre los lados de un tringulo


a

65

por otro lado


abc = (x + y)(y + z)(z + x).
Luego, la desigualdad es equivalente a
8xyz (x + y)(y + z)(z + x),

(2.1)

que sabemos que es vlida por el ejercicio 1.26.


a
Ejemplo 2.2.4 (APMO, 1996) Sean a, b, c las longitudes de los lados de un


tringulo, muestre que a + b c + b + c a + c + a b a+ b+ c.
a
Como a = y + z, b = z + x, c = x + y, podemos deducir a + b c = 2z,
b + c a = 2x, c + a b = 2y. Luego, la desigualdad es equivalente a

2x + 2y + 2z x + y + y + z + z + x.
Ahora usando la desigualdad entre la media aritmtica y la media cuadrtica
e
a
(vea ejercicio 1.68), obtenemos

2y + 2z
2x + 2y
2z + 2x
2x + 2y + 2z =
+
+
2
2
2
2x + 2y
2y + 2z
2z + 2x
+
+
2
2
2

=
x + y + y + z + z + x.

Adems, la igualdad se tiene si y slo si x = y = z, es decir, si y slo si


a
o
o
a = b = c.
Tambin, es posible expresar el rea de un tringulo ABC, su inradio, su cire
a
a
cunradio y su semiper
metro en trminos de x, y, z. Como a = x + y, b = y + z
e
o
y c = z + x, se obtiene primero que s = a+b+c = x + y + z. Usando la frmula
2
de Hern para el rea de un tringulo se obtiene
o
a
a
(ABC) =

s(s a)(s b)(s c) =

(x + y + z)xyz.

Usando la frmula (ABC) = sr, vemos que


o
r=

(ABC)
=
s

Finalmente, de (ABC) =

abc
4R

R=

(x + y + z)xyz
=
x+y+z

xyz
.
x+y+z

se tiene
(x + y)(y + z)(z + x)
4

(x + y + z)xyz

(2.2)

66

Desigualdades Geomtricas
e

Ejemplo 2.2.5 (India, 2003) Sean a, b, c las longitudes de los lados de un


b
tringulo ABC. Si construimos un tringulo A B C con lados de longitud a+ 2 ,
a
a
a
c
B C ) 9 (ABC).
b + 2 , c + 2 , muestre que (A
4
Como a = y + z, b = z + x y c = x + y, se tiene que las longitudes de los lados
del tringulo A B C son a = x+2y+3z , b = 3x+y+2z , c = 2x+3y+z . Usando
a
2
2
2
la frmula de Hern para el rea de un tringulo se obtiene
o
o
a
a
(A B C ) =

3(x + y + z)(2x + y)(2y + z)(2z + x)


.
16

Aplicando la desigualdad entre la M G M A podemos establecer que 2x + y

3
a
3 3 x2 y, 2y + z 3 3 y 2 z y 2z + x 3 z 2 x, lo que nos ayudar a tener la
siguiente desigualdad
(A B C )

9
3(x + y + z)27(xyz)
= (ABC),
16
4

donde la ultima igualdad es consecuencia directa de la ecuacin (2.2).

o
Ejercicio 2.18 Sean a, b y c las longitudes de los lados de un tringulo, muestre
a
que
3(ab + bc + ca) (a + b + c)2 4(ab + bc + ca).
Ejercicio 2.19 Sean a, b y c las longitudes de los lados de un tringulo, muestre
a
que
ab + bc + ca a2 + b2 + c2 2(ab + bc + ca).
Ejercicio 2.20 Sean a, b y c las longitudes de los lados de un tringulo, muestre
a
que
2 a2 + b2 + c2 (a + b + c)2 .
Ejercicio 2.21 Sean a, b y c las longitudes de los lados de un tringulo, muestre
a
que
a
b
c
3

+
+
< 2.
2
b+c c+a a+b
Ejercicio 2.22 (IMO, 1964) Sean a, b y c las longitudes de los lados de un
tringulo, muestre que
a
a2 (b + c a) + b2 (c + a b) + c2 (a + b c) 3abc.

2.2 Desigualdades entre los lados de un tringulo


a

67

Ejercicio 2.23 Sean a, b y c las longitudes de los lados de un tringulo, muestre


a
que
a b2 + c2 a2 + b c2 + a2 b2 + c a2 + b2 c2 3abc.
Ejercicio 2.24 (IMO, 1983) Sean a, b y c las longitudes de los lados de un
tringulo, muestre que
a
a2 b(a b) + b2 c(b c) + c2 a(c a) 0.
Ejercicio 2.25 Sean a, b y c las longitudes de los lados de un tringulo, muestre
a
que
1
ab bc ca
+
+
< .
a+b b+c c+a
8
Ejercicio 2.26 Las longitudes de los lados de un tringulo a, b y c, satisfacen
a
ab + bc + ca = 3, muestre que

3 a + b + c 2 3.
Ejercicio 2.27 Sean a, b, c las longitudes de los lados de un tringulo y r el
a
inradio del tringulo, muestre que
a

3
1 1 1
+ +
.
a b
c
2r
Ejercicio 2.28 Sean a, b, c las longitudes de los lados de un tringulo y s el
a
semiper
metro, muestre que:
(i) (s a)(s b) < ab,

(ii) (s a)(s b) + (s b)(s c) + (s c)(s a)

ab + bc + ca
.
4

Ejercicio 2.29 Si a, b, c son las longitudes de los lados de un tringulo


a
acutngulo, muestre que
a

c
clica

a2 + b2 c2

a2 b2 + c2 a2 + b2 + c2 ,

68

Desigualdades Geomtricas
e

donde c
clicas, (a, b, c), (b, c, a)
clica signica la suma sobre las permutaciones c
y (c, a, b), de (a, b, c).
Ejercicio 2.30 Si a, b, c son las longitudes de los lados de un tringulo
a
acutngulo, muestre que
a

c
clica

a2 + b2 c2

a2 b2 + c2 ab + bc + ca,

donde c
clicas, (a, b, c), (b, c, a)
clica signica la suma sobre las permutaciones c
y (c, a, b), de (a, b, c).

2.3.

Uso de desigualdades en la geometr del tringua


a
lo

Un problema que ayuda a ejemplicar el uso de las desigualdades en la geometr


a
es el problema 2 que se plante en la Olimpiada Internacional de Matemticas
o
a
en 1961. De ste existen varias pruebas y tambin sus aplicaciones son diversas
e
e
como veremos ms adelante. Por ahora lo presentamos como un ejemplo.
a
Ejemplo 2.3.1 Si a, b y c son las longitudes de los lados de un tringulo de
a

2 + b2 + c2 .
a
rea (ABC), entonces 4 3(ABC) a

Como un tringulo equiltero con lado de longitud a tiene rea igual a 43 a2 ,


a
a
a
la igualdad se alcanza en tal caso; trataremos entonces de comparar lo que
sucede en un tringulo en general con lo que pasa, al respecto, en un tringulo
a
a
equiltero con lado de longitud a.
a
A
c

h
d

Sea BC = a. Si AD es la altura de un tringulo desde A, su longitud h la


a

3
podemos escribir como h = 2 a + y, donde y mide su defecto con respecto a
la longitud de la altura del tringulo equiltero. Tambin escribimos d = a x
a
a
e
2
a
y e = 2 + x, donde x se puede interpretar como el defecto o la diferencia, que

2.3 Uso de desigualdades en la geometr del tringulo


a
a

69

la proyeccin de A en BC tiene, con respecto a la proyeccin de A en BC en el


o
o
tringulo equiltero, que en tal caso es el punto medio del lado BC. Tenemos
a
a
que

a
+x
a2 + b2 + c2 4 3(ABC) = a2 + h2 +
2

+ h2 +

a
x
2

ah
4 3
2

3 2
3
2
2
= a + 2h + 2x 2 3 a
a+y
2
2

3
3 2
= a +2
a + y + 2x2 3a2 2 3ay
2
2

3
3
= a2 + a2 + 2 3ay + 2y 2 + 2x2 3a2 2 3ay
2
2
= 2(x2 + y 2 ) 0.

Adems, la igualdad se da si y slo si x = y = 0, es decir, cuando el tringulo


a
o
a
es equiltero.
a
Veamos otra demostracin del ejemplo anterior. Sea ABC el tringulo con
o
a
lados de longitud a b c, y sea A un punto de manera que A BC sea un
tringulo equiltero con lados de longitud a. Llamemos d = AA , entonces d
a
a
mide de alguna manera el defecto que tiene ABC con respecto al tringulo
a
equiltero.
a

c
B

70

Desigualdades Geomtricas
e

Por la ley de los cosenos, tenemos que


d2 = a2 + c2 2ac cos(B 60 )

= a2 + c2 2ac(cos B cos 60 + sen B sen 60 )


ac sen B
= a2 + c2 ac cos B 2 3
2

a2 + c2 b2
= a2 + c2 ac
2 3(ABC)
2ac
2 + b2 + c2

a
=
2 3(ABC).
2

Pero d2 0, luego tenemos que 4 3(ABC) a2 + b2 + c2 , como desebamos.


a
= A o, equivalentemente, si
Adems, la igualdad se da si d = 0, esto es si A
a
ABC es equiltero.
a
Es comn encontrar entre los problemas de olimpiadas desigualdades que inu
volucran elementos de un tringulo. Algunas de ellas son consecuencia de la
a
siguiente desigualdad, vlida para nmeros positivos a, b, c, (ver ejercicio 1.36
a
u
de la seccin 1.3).
o
1 1 1
+ +
9.
(2.3)
(a + b + c)
a b
c
Adems, recordemos que se da la igualdad si y slo si a = b = c.
a
o
Otra desigualdad que ha resultado muy util para problemas de geometr es

a,

la desigualdad de Nesbitt, (ver ejemplo 1.4.8 de la seccin 1.4). Esta dice que
o
para nmeros positivos a, b, c, siempre se tiene
u
b
c
3
a
+
+
.
b+c c+a a+b
2

(2.4)

La desigualdad anterior se puede demostrar, usando la desigualdad (2.3) de la


siguiente forma
a
b
c
a+b+c a+b+c a+b+c
+
+
=
+
+
3
b+c c+a a+b
b+c
c+a
a+b
1
1
1
+
+
3
= (a + b + c)
b+c c+a a+b
1
1
1
1
= [(a + b) + (b + c) + (c + a)]
+
+
3
2
b+c c+a a+b

9
3
3= .
2
2

2.3 Uso de desigualdades en la geometr del tringulo


a
a

71

La igualdad se da si y slo si a + b = b + c = c + a, o equivalentemente, si


o
a = b = c.
Veamos ahora una serie de ejemplos de desigualdades geomtricas donde tales
e
relaciones se utilizan.
Ejemplo 2.3.2 Sea ABC un tringulo equiltero con lados de longitud a; sea
a
a
M un punto dentro de ABC y D, E, F las proyecciones de M sobre los lados
BC, CA y AB, respectivamente. Muestre que:

1
1
6 3
1
+
+

,
(i)
MD ME
MF
a

1
1
1
3 3
(ii)
+
+

.
MD + ME ME + MF
MF + MD
a
A
F

Sean x = M D, y = M E y z = M F . Si denotamos por (ABC) al rea del


a
tringulo ABC, tenemos que (ABC) (BCM ) + (CAM ) + (ABM ), por lo
a
=
que ah = ax + ay + az, donde h = 23 a es la longitud de la altura de ABC.
Por lo tanto, h = x + y + z (este resultado es conocido como el lema de Viviani,
ver la seccin 2.8). Por la desigualdad (2.3) tenemos que
o
h

1 1 1
+ +
x y z

9, despejando,

9
6 3
1 1 1
+ + =
.
x y z
h
a

Para demostrar la segunda parte, observemos que de la desigualdad (2.3), tenemos que
(x + y + y + z + z + x)
Por lo tanto,

1
x+y

1
y+z

1
z+x

9
2h

1
1
1
+
+
x+y y+z z+x
=

3 3
a .

9.

72

Desigualdades Geomtricas
e

Ejemplo 2.3.3 Si ha , hb y hc son las longitudes de las alturas de un tringulo


a
ABC que tiene inc
rculo de centro I y radio r, tenemos:
(i)

r
r
r
+
+
= 1,
ha hb hc

(ii) ha + hb + hc 9r.
Para demostrar la primera ecuacin, observemos que
o
manera anloga,
a

r
hb

(ICA)
r
(ABC) , hc

r
r
r
+
+
ha hb hc

(IAB)
(ABC) ,

r
ha

sumando

ra
ha a

(IBC)
(ABC) .

De

(ICA)
(IAB)
(IBC)
+
+
(ABC) (ABC) (ABC)
(IBC) + (ICA) + (IAB)
= 1.
(ABC)

=
=

ha

I
r

La desigualdad es consecuencia directa de la desigualdad (2.3), ya que


(ha + hb + hc )

1
1
1
+
+
ha hb hc

r 9r.

Ejemplo 2.3.4 Sea ABC un tringulo con alturas AD, BE, CF y sea H el
a
ortocentro. Muestre que:
(i)
(ii)

BE
CF
AD
+
+
9,
HD HE
HF
HF
3
HD HE
+
+
.
HA HB HC
2

2.3 Uso de desigualdades en la geometr del tringulo


a
a
A

73

F
H

Para demostrar el inciso (i), consideremos S = (ABC), S1 = (HBC), S2 =


(HCA), S3 = (HAB). Como los tringulos ABC y HBC tienen la misma
a
base, la razn de sus reas es igual a la razn de sus alturas, es decir, S1 = HD .
o
a
o
S
AD
Anlogamente, S2 = HE y S3 = HF . Luego, HD + HE + HF = 1.
a
S
BE
S
CF
AD
BE
CF
Usando la desigualdad (2.3), tenemos
BE
CF
AD
+
+
HD HE
HF

HF
HD HE
+
+
AD
BE
CF

9.

Al sustituir aqu la igualdad que calculamos obtenemos (i).

1
Ms an, la igualdad se da si y slo si HD = HE = HF = 3 , o lo que es lo
a u
o
AD
BE
CF
1
mismo, S1 = S2 = S3 = 3 S. Para demostrar la segunda parte observemos que,
S1
S1
S2
S3
HD
HE
HF
HD
a
HA = ADHD = SS1 = S2 +S3 y anlogamente, HB = S3 +S1 , HC = S1 +S2 ,
3
entonces por la desigualdad de Nesbitt tenemos que HD + HE + HF 2 .
HA
HB
HC

Ejemplo 2.3.5 (Corea, 1995) Sean ABC un tringulo y L, M , N puntos sobre


a
BC, CA y AB, respectivamente. Sean P , Q y R los puntos de interseccin
o
de las l
neas AL, BM y CN con el circunc
rculo de ABC, respectivamente.
Muestre que
CN
AL BM
+
+
9.
LP
MQ NR

Sean A el punto medio de BC, P el punto medio del arco BC, D y D las
proyecciones de A y P sobre BC, respectivamente.

74

Desigualdades Geomtricas
e

P
P
AL
AD
AL
AD
Es claro que, LP = P D P A . Luego, el valor m
nimo de LP + BM + CN se
MQ
NR
obtiene cuando P , Q y R son los puntos medios de los arcos BC, CA y AB,
respectivamente. Esto suceder cuando AL, BM y CN sean las bisectrices
a
internas del tringulo ABC. Por lo que, sin perder generalidad, supondremos
a
que AL, BM y CN son las bisectrices internas de ABC. Como AL es bisectriz
interna, se tiene que1

BL =

ba
ca
, LC =
b+c
b+c

a
b+c

AL2 = bc 1

Adems,
a
AL2

AL2

AL
=
=
=
LP
AL LP
BL LC

(bc) 1

a
b+c

a2 bc
(b+c)2

(b + c)2 a2
.
a2

Anlogamente, para las bisectrices BM y CN , se tiene


a
BM
(c + a)2 b2
=
MQ
b2

(a + b)2 c2
CN
=
.
NR
c2

Por lo tanto,
CN
AL BM
+
+
LP
MQ NR

b+c
a

c+a
b

1 b+c c+a a+b


+
+
3
a
b
c
1
(6)2 3 = 9.
3

a+b
c

Ver teorema de la bisectriz y ejercicio 2.8.17 del cap


tulo 2 en [6], pg.74 y 105 o [9],
a
pg.10 y 11.
a

2.3 Uso de desigualdades en la geometr del tringulo


a
a

75

La primera desigualdad se sigue de la convexidad de la funcin f (x) = x2 , y la


o
c
b
segunda desigualdad de relaciones de la forma c + b 2. Observemos que la
igualdad se da si y slo si a = b = c.
o
Otra manera de terminar el problema es como sigue,
b+c
a
=

a2
b2
+ 2
b2
a

c+a
b

b2 c2
+
c2 b2

c2
a2
+ 2
a2
c

a+b
c

3=

+2

ab bc ca
+ 2+ 2
c2
a
b

2 3 + 2 3 3 = 9.
Aqu usamos el hecho que

a2
b2

b
+ a2 2 y que

ab
c2

bc
+ a2 + ca 3 3
b2

(ab)(bc)(ca)
a2 b2 c2

= 3.

Ejemplo 2.3.6 (Lista corta IMO, 1997) Las longitudes de los lados de un
hexgono ABCDEF satisfacen AB = BC, CD = DE y EF = F A. Muestre
a
que
DE
FA
3
BC
+
+
.
BE
DA F C
2
B

A
c

a
b

D
E
Sean a = AC, b = CE y c = EA. La desigualdad de Ptolomeo (ver ejercicio
2.11), aplicada en el cuadriltero ACEF , nos garantiza que AE F C F A
a
CE + AC EF . Como EF = F A, tenemos entonces c F C F A b + F A a.
Por lo tanto,
FA
c

.
FC
a+b
Anlogamente, se puede ver que
a
a
BC

BE
b+c

DE
b

.
DA
c+a

FA
a
b
c
3
Por tanto, BC + DE + F C b+c + c+a + a+b 2 . La ultima desigualdad es

BE
DA
la desigualdad de Nesbitt, que sabemos es cierta.

76

Desigualdades Geomtricas
e

Ejercicio 2.31 Sean a, b, c las longitudes de los lados de un tringulo, muestre


a
que:
(i)
(ii)

b
c
a
+
+
3,
b+ca c+ab a+bc

b+ca c+ab a+bc


+
+
3.
a
b
c

Ejercicio 2.32 Sean AD, BE, CF las alturas de un tringulo ABC y P Q, P R,


a
P S las distancias de un punto P a los lados BC, CA, AB, respectivamente.
Muestre que
AD BE CF
+
+
9.
PQ PR
PS
Ejercicio 2.33 Tres l
neas se trazan por un punto O que se localiza dentro de
un tringulo de rea S de manera que cada lado es cortado por dos de stas.
a
a
e
Las l
neas dividen al tringulo en tres tringulos con vrtice comn O y reas
a
a
e
u
a
S1 , S2 y S3 , adems de tres cuadrilteros. Muestre que:
a
a
(i)
(ii)

1
1
9
1
+
+
,
S1 S2 S3
S
1
1
1
18
+
+
.
S1 S2 S3
S

Ejercicio 2.34 Tres cevianas AL, BM , CN de un tringulo ABC son concua


BP
CP
o
rrentes en un punto P . Muestre que, AP + P M + P N = 6 si y slo si P es el
PL
centroide.
Ejercicio 2.35 Las alturas AD, BE, CF intersectan al circunc
rculo del
tringulo ABC en D , E y F , respectivamente. Muestre que:
a
(i)
(ii)

BE
CF
AD
+
+
9,

DD
EE
FF
BE
CE
9
AD
+
+
.

AD
BE
CF
4

Ejercicio 2.36 Sea ABC un tringulo y sean la , lb , lc las longitudes de las


a
bisectrices internas, s el semiper
metro y r el inradio de tal tringulo. Muestre
a
que:

2.4 La desigualdad de Euler y algunas aplicaciones

77

(i) la lb lc rs2 .

(ii) la lb + lb lc + lc la s2 .

2
2
2
(iii) la + lb + lc s2 .

Ejercicio 2.37 Sea ABC un tringulo y sean M , N , P puntos arbitrarios en los


a
segmentos BC, CA, BA, respectivamente. Denote las longitudes de los lados
del tringulo por a, b, c y el circunradio por R. Muestre que
a
ca
ab
bc
+
+
6R.
AM
BN
CP

Ejercicio 2.38 Sea ABC un tringulo con lados de longitud a, b y c. Sean


a
ma , mb y mc las longitudes de las medianas desde A, B y C, respectivamente.
Muestre que
mx {a ma , b mb , c mc } sR,
a
donde R es el radio del circunc
rculo y s el semiper
metro.

2.4.

La desigualdad de Euler y algunas aplicaciones

Teorema 2.4.1 (Teorema de Euler) Si en un tringulo ABC, O es su cira


cuncentro, I su incentro, R su circunradio y r su inradio, entonces
OI 2 = R2 2Rr.
Demostracin. Veamos una demostracin2 que solamente depende del teorema
o
o
de Pitgoras y del hecho de que el circunc
a
rculo del tringulo BCI tiene centro
a
D, donde D es el punto medio del arco BC 3 . En la demostracin usaremos
o
segmentos dirigidos.
2
3

Otra demostracin puede verse en [6], pg. 122 o [9], pg. 29.
o
a
a
La demostracin puede verse en [6], observacin 3.2.7, pg. 123 o [1], pg. 76.
o
o
a
a

78

Desigualdades Geomtricas
e

Sean M el punto medio de BC y Q la proyeccin ortogonal de I sobre el radio


o
OD. Entonces
OB 2 OI 2 = OB 2 DB 2 + DI 2 OI 2

= OM 2 M D 2 + DQ2 QO2

= (M O + DM ) (M O DM ) + (DQ + QO)(DQ QO)

= DO(M O + M D + DQ + OQ)
= R(2M Q) = 2Rr.
Por lo tanto, OI 2 = R2 2Rr.

Como una consecuencia del teorema anterior tenemos la siguiente desigualdad,


Teorema 2.4.2 (Desigualdad de Euler) R 2r. Adems, R = 2r si y slo
a
o
si el tringulo es equiltero4 .
a
a
Teorema 2.4.3 En un tringulo ABC, con circunradio R, inradio r y semipea
r
metro s, se tiene
R
s
r .
2
3 3
4

Existen demostraciones directas de la desigualdad (esto es, sin utilizar la frmula de Euler).
o
Una de ellas es la siguiente: el c
rculo de los nueve puntos de un tringulo es el circunc
a
rculo
del tringulo medial A B C , como este tringulo es semejante a ABC en razn 2:1, se tiene
a
a
o
que el radio del c
rculo de los nueve puntos es R . Claramente una circunferencia que corta a
2
los tres lados del tringulo debe tener radio mayor que el radio del inc
a
rculo, luego R r.
2

2.4 La desigualdad de Euler y algunas aplicaciones

79

Demostracin. Usaremos que5 , (ABC) = abc = sr. Por la desigualdad M G


o
4R

M A, tenemos que 2s = a + b + c 3 3 abc = 3 3 4Rrs. Luego, 8s3

27(4Rrs) 27(8r 2 s), ya que R 2r. Por lo tanto, s 3 3r.

s
La segunda desigualdad, 33 R , es equivalente a a + b + c 3 3R. Pero
2
sta, utilizando la ley de los senos, es equivalente a sen A + sen B + sen C
e
3 3

o
2 . Observemos que esta ultima desigualdad es verdadera, ya que la funcin
f (x) = sen x es cncava en [0, ], entonces sen A+sen B+sen C sen A+B+C =
o
3
3
sen 60 =

3
2 .

Ejercicio 2.39 Sean a, b y c las longitudes de los lados de un tringulo, muestre


a
que
(a + b c)(b + c a)(c + a b) abc.
Ejercicio 2.40 Sean a, b y c las longitudes de los lados de un tringulo, muestre
a
que
1
1
1
1
+
+
2,
ab bc ca
R
donde R denota el circunradio.
Ejercicio 2.41 Sean A, B y C las medidas de los ngulos en cada uno de los
a
vrtices del tringulo ABC, muestre que
e
a
1
1
1
+
+
4.
sen A sen B sen B sen C
sen C sen A
Ejercicio 2.42 Sean A, B y C las medidas de los ngulos en cada uno de los
a
vrtices del tringulo ABC, muestre que
e
a
sen

A
2

sen

B
2

sen

C
2

1
.
8

Ejercicio 2.43 Sea ABC un tringulo. Llamamos a los ngulos en los vrtices
a
a
e
en A, B, C, de la misma forma, es decir, A, B, C, respectivamente. Sean a, b
y c las longitudes de los lados del tringulo y sea R el radio del circunc
a
rculo.
Muestre que

2A

1
a

2B

Consultar [6], pg. 97 o [9], pg. 13.


a
a

1
b

2C

1
c

2
3

3
R

80

Desigualdades Geomtricas
e

Teorema 2.4.4 (Teorema de Leibniz) En un tringulo ABC con lados de


a
longitud a, b y c, y con circuncentro O, centroide G y circunradio R, se cumple
que
OG2 = R2

1 2
a + b2 + c2 .
9

Demostracin. Usaremos el teorema de Stewart que dice6 que, si L es un


o
punto sobre el lado BC de un tringulo ABC y si AL = l, BL = m, LC = n,
a
entonces a l2 + mn = b2 m + c2 n.
A

R
B

Aplicando el teorema de Stewart al tringulo OAA para encontrar la longitud


a
es el punto medio de BC, obtenemos
de OG, donde A
AA OG2 + AG GA = A O2 AG + AO2 GA .
Como
2
AO = R, AG = AA
3

1
GA = AA ,
3

sustituyendo tenemos
2
1
2
OG2 + (A A)2 = A O2 + R2 .
9
3
3
6

Para la demostracin vase [6], pg. 96 o [9], pg. 6.


o e
a
a

2.4 La desigualdad de Euler y algunas aplicaciones


Por otro lado7 , (A A)2 =
OG2 =

2(b2 +c2 )a2


4

R2

a2
4

y A O2 = R2

2 1 2 2
+ R
3 3
9

a2
4 ,

81
tenemos que

2 b2 + c2 a2
4

a2 2 b2 + c2 a2

6
18
2 + b2 + c2
a
= R2
.
9
= R2

Una consecuencia del teorema anterior es la desigualdad siguiente.


Teorema 2.4.5 (Desigualdad de Leibniz) En un tringulo ABC, con lados
a
de longitud a, b y c, con circunradio R, se cumple que
9R2 a2 + b2 + c2 .
Adems, la igualdad se da si y slo si O = G, es decir, cuando el tringulo es
a
o
a
equiltero.
a
Ejemplo 2.4.6 En un tringulo ABC con lados de longitud a, b y c, y de rea
a
a
(ABC), se tiene que

9abc
.
4 3(ABC)
a+b+c
Usando que 4R(ABC) = abc, tenemos las siguientes equivalencias
a2 b2 c2
3abc
a2 + b2 + c2
4(ABC)

.
2
2 + b2 + c2
16(ABC)
9
a

La desigualdad de Cauchy-Schwarz nos garantiza que a+b+c 3 a2 + b2 + c2 ,
por lo que

9abc
4 3(ABC)
.
a+b+c

9R2 a2 +b2 +c2

Ejercicio 2.44 Sean A, B y C las medidas de los ngulos en cada uno de los
a
vrtices del tringulo ABC, muestre que
e
a
sen2 A + sen2 B + sen2 C
7

Consultar [6], pg. 83 o [9], pg. 10.


a
a

9
.
4

82

Desigualdades Geomtricas
e

Ejercicio 2.45 Sean a, b y c las longitudes de los lados de un tringulo, muestre


a
que

3
4 3(ABC) 3 a2 b2 c2 .

Ejercicio 2.46 Suponga que el inc


rculo de ABC es tangente a los lados BC,
CA, AB, en D, E, F , respectivamente. Muestre que
EF 2 + F D 2 + DE 2

s2
,
3

donde s es el semiper
metro de ABC.
Ejercicio 2.47 Sean a, b, c las longitudes de los lados de un tringulo ABC
a
y sean ha , hb , hc las longitudes de las alturas sobre A, B, C, respectivamente.
Muestre que
b2
c2
a2
+
+
4.
hb hc
hc ha ha hb

2.5.

Funciones simtricas de a, b y c
e

Las longitudes de los lados a, b y c de un tringulo tienen una relacin muy


a
o
estrecha con s, r y R el semiper
metro, el inradio y el circunradio del tringulo,
a
respectivamente. Las relaciones ms usadas son:
a
a + b + c = 2s
2

(2.5)
2

ab + bc + ca = s + r + 4rR

(2.6)

abc = 4Rrs.

(2.7)

La primera es la denicin de s y la tercera se sigue de que el rea de un


o
a
abc
o
o
a
a
tringulo es 4R = rs. Usando la frmula de Hern para el rea de un tringulo
a
tenemos la relacin s(s a)(s b)(s c) = r 2 s2 , por lo que
o
s3 (a + b + c)s2 + (ab + bc + ca)s abc = r 2 s.
Sustituyendo las ecuaciones (2.5) y (2.7) en esta igualdad, y despejando obtenemos que
ab + bc + ca = s2 + r 2 + 4Rr.

2.5 Funciones simtricas de a, b y c


e

83

Ahora bien, como cualquier polinomio simtrico en a, b y c se puede expresar


e
como un polinomio en trminos de (a + b + c), (ab + bc + ca) y (abc), tambin
e
e
lo podemos expresar como un polinomio en s, r y R. Por ejemplo,
a2 + b2 + c2 = (a + b + c)2 2(ab + bc + ca) = 2 s2 r 2 4Rr ,

a3 + b3 + c3 = (a + b + c)3 3(a + b + c)(ab + bc + ca) + 3abc


= 2 s3 3r 2 s 6Rrs .

Estas transformaciones ayudan a resolver diversos problemas como veremos ms


a
adelante.
Lema 2.5.1 Si A, B y C son las medidas de los ngulos en cada uno de los
a
r
vrtices del tringulo ABC, se tiene que cos A + cos B + cos C = R + 1.
e
a
Demostracin.
o
cos A + cos B + cos C =
=
=
=
=
=

b2 + c2 a2 c2 + a2 b2 a2 + b2 c2
+
+
2bc
2ca
2ab
2 + c2 + b c2 + a2 + c a2 + b2 a3 + b3 + c3
a b
2abc
2 + b2 + c2 ) 2(a3 + b3 + c3 )
(a + b + c) (a
2abc
2 r 2 4Rr 4 s3 3r 2 s 6Rrs
4s s
8Rrs
2 r 2 4Rr (s2 3r 2 6Rr)
s
2Rr
2 + 2Rr
2r
r
= + 1.
2Rr
R

Ejemplo 2.5.2 Sean A, B y C los ngulos en cada uno de los vrtices del
a
e
tringulo ABC, muestre que cos A + cos B + cos C 3 .
a
2
El lema 2.5.1 nos garantiza que cos A + cos B + cos C =
desigualdad de Euler, R 2r, tenemos el resultado.

r
R

+ 1, y usando la

Podemos dar otra demostracin directa. Observemos que,


o

a(b2 +c2 a2 )+b(c2 +a2 b2 )+c(a2 +b2 c2 ) = (b+ca)(c+ab)(a+bc)+2abc.

84

Desigualdades Geomtricas
e

Luego,
cos A + cos B + cos C =
=

b2 + c2 a2 c2 + a2 b2 a2 + b2 c2
+
+
2bc
2ca
2ab
(b + c a)(c + a b)(a + b c)
+ 1,
2abc

y como (b + c a)(c + a b)(a + b c) abc, se tiene el resultado.


Ejemplo 2.5.3 (IMO, 1991) Sean ABC un tringulo, I su incentro y L, M ,
a
N las intersecciones de las bisectrices en A, B, C con BC, CA, AB, respectiAI BI CI
8
vamente. Muestre que 1 < AL BM CN 27 .
4
A
M
I
B

C
L
c
El teorema de la bisectriz nos asegura que BL = AB = b y, como BL+LC = a,
LC
CA
ac
ab
se tiene que BL = b+c y LC = b+c . El mismo teorema de la bisectriz aplicado
a
IL
BL
ac
a la bisectriz BI del ngulo ABL, garantiza que AI = AB = (b+c)c = b+c .
a
Por lo que,
AI + IL
IL
a
a+b+c
AL
=
=1+
=1+
=
.
AI
AI
AI
b+c
b+c
AI
b+c
BI
c+a
CI
Luego, AL = a+b+c 8 . Anlogamente, BM = a+b+c y CN =
a
desigualdad que se pide demostrar en trminos de a, b y c, es
e

a+b
a+b+c .

As la
,

(b + c)(c + a)(a + b)
8
1
<
.
3
4
(a + b + c)
27
La desigualdad M G AM nos garantiza que
(b + c)(c + a)(a + b)

(b + c) + (c + a) + (a + b)
3

8
(a + b + c)3 ,
27

por lo que la desigualdad de la derecha que debemos demostrar es ahora evidente.


8

Otra forma de ver la identidad es como sigue: Considere = (ABI), = (BCI) y


r(c+b)
+
AI
c+b
= (CAI), es claro que, AL = ++ = r(a+c+b) = a+c+b .

2.5 Funciones simtricas de a, b y c


e

85

Para demostrar la desigualdad de la izquierda, notemos primero que


(a + b + c)(ab + bc + ca) abc
(b + c)(c + a)(a + b)
=
.
3
(a + b + c)
(a + b + c)3
Sustituyendo las ecuaciones (2.5), (2.6) y (2.7) tenemos
(b + c)(c + a)(a + b)
(a + b + c)3

=
=

2s(s2 + r 2 + 4Rr) 4Rrs


8s3
1 2r 2 + 4Rr
1
2s3 + 2sr 2 + 4Rrs
= +
> .
3
2
8s
4
8s
4

Podemos usar tambin la transformacin de Ravi con a = y + z, b = z + x,


e
o
c = x + y, para concluir de la siguiente manera
(b + c)(c + a)(a + b)
(x + y + z + x)(x + y + z + y)(x + y + z + z)
=
3
(a + b + c)
8(x + y + z)3
=

1
8

1+

x
x+y+z

1
8

1+

xyz
x + y + z xy + yz + zx
+
+
x+y+z
x+y+z
x+y+z

1+

y
x+y+z

1+

z
x+y+z
>

1
.
4

Ejercicio 2.48 Sean A, B y C las medidas de los ngulos en cada uno de los
a
vrtices del tringulo ABC, muestre que
e
a
sen2

B
C
3
A
+ sen2 + sen2 .
2
2
2
4

Ejercicio 2.49 Sean a, b y c las longitudes de los lados de un tringulo y (ABC)


a
el rea del tringulo. Con la herramienta estudiada en esta seccin muestre que
a
a
o

4 3(ABC)

9abc
.
a+b+c

Ejercicio 2.50 Sean a, b y c las longitudes de los lados de un tringulo y (ABC)


a
el rea del tringulo, con la herramienta estudiada en esta seccin, muestre que
a
a
o

3
4 3(ABC) 3 a2 b2 c2 .

86

Desigualdades Geomtricas
e

Ejercicio 2.51 (IMO, 1961) Sean a, b y c las longitudes de los lados de un


tringulo y (ABC) el rea del tringulo, muestre que
a
a
a

4 3(ABC) a2 + b2 + c2 .
Ejercicio 2.52 Sean a, b y c las longitudes de los lados de un tringulo y
a
(ABC) el rea del tringulo, muestre que
a
a

4 3(ABC) a2 + b2 + c2 (a b)2 (b c)2 (c a)2 .


Ejercicio 2.53 Sean a, b y c las longitudes de los lados de un tringulo y
a
(ABC) el rea del tringulo, muestre que
a
a

4 3(ABC) ab + bc + ca.
Ejercicio 2.54 Sean a, b y c las longitudes de los lados de un tringulo y
a
(ABC) el rea del tringulo, muestre que
a
a

3(a + b + c)abc
4 3(ABC)
.
ab + bc + ca
Ejercicio 2.55 Sean a, b y c las longitudes de los lados de un tringulo. Si
a
a + b + c = 1, muestre que
1
a2 + b2 + c2 + 4abc < .
2
Ejercicio 2.56 Sean a, b y c las longitudes de los lados de un tringulo, R el
a
circunradio y r el inradio, muestre que
2r
(b + c a)(c + a b)(a + b c)
= .
abc
R
Ejercicio 2.57 Sean a, b y c las longitudes de los lados de un tringulo y R el
a
circunradio, muestre que

3 3R

a2
b2
c2
+
+
.
b+ca c+ab a+bc

Ejercicio 2.58 Sean a, b y c las longitudes de los lados de un tringulo y


a
b+ca
(ABC) el rea del tringulo. Tomemos como siempre x = 2 , y = c+ab
a
a
2
y z = a+bc . Si 1 = x + y + z, 2 = xy + yz + zx y 3 = xyz, verique las
2
siguientes relaciones:

2.6 Desigualdades con reas y per


a
metros

87

2
(1) (a b)2 + (b c)2 + (c a)2 = (x y)2 + (y z)2 + (z x)2 = 2(1 32 ).

(2) a + b + c = 21 .
2
(3) a2 + b2 + c2 = 21 22 .
2
(4) ab + bc + ca = 1 + 2 .

(5) abc = 1 2 3 .
(6) 16(ABC)2 = 2(a2 b2 + b2 c2 + c2 a2 ) (a4 + b4 + c4 ) = 16r 2 s2 = 161 3 .
(7) R =

(8) r =

1 2 3
.

4 1 3
3
.
1

(9) 1 = s, 2 = r(4R + r), 3 = r 2 s.

2.6.

Desigualdades con reas y per


a
metros

Empezamos esta seccin con los siguientes ejemplos.


o
Ejemplo 2.6.1 (Austria-Polonia, 1985) Si ABCD es un cuadriltero convexo
a
de rea 1, entonces
a
AB + BC + CD + DA + AC + BD 4 +

8.

Sean a = AB, b = BC, c = CD, d = DA, e = AC y f = BD. El rea


a
ef sen
a
del cuadriltero ABCD es (ABCD) = 2 , donde es el ngulo entre las
a
ef
ef sen
diagonales, luego es claro que, 1 = 2 2 .

Como (ABC) = ab sen B ab y (CDA) = cd sen D


2
2
2
1 = (ABCD) ab+cd . Anlogamente, 1 = (ABCD)
a
2
desigualdades nos garantizan que ab + bc + cd + da 4.

cd
2 , tenemos
bc+da
2 . Estas

que
dos

Finalmente, como (e + f )2 = 4ef + (e f )2 4ef 8 y (a + b + c + d)2 =


4(a+c)(b+d)+((a+c)(b+d))2 4(a+c)(b+d) = 4(ab+bc+cd+da) 16,

tenemos que, a + b + c + d + e + f 4 + 8.

88

Desigualdades Geomtricas
e

Ejemplo 2.6.2 (Iberoamericana, 1992) A partir de un tringulo ABC, se consa


truye un hexgono H de vrtices A1 , A2 , B1 , B2 , C1 , C2 como se muestra en
a
e
la gura. Demuestre que el rea del hexgono H es mayor o igual que trece
a
a
veces el rea del tringulo ABC.
a
a
A1

A2
a

a
A

c
b B

B1

b
a

C c
c

C2
C1

B2
Es claro, usando la frmula de rea (ABC) =
o
a

ab sen C
,
2

que

(A1 A2 B1 B2 C1 C2 ) =(A1 BC2 ) + (A2 CB1 ) + (B2 AC1 ) + (AA1 A2 )+


+ (BB1 B2 ) + (CC1 C2 ) 2(ABC)
=

(b + c)2 sen A
(c + a)2 sen B (a + b)2 sen C
+
+
+
2
2
2

a2 sen A b2 sen B c2 sen C


+
+
2(ABC)
2
2
2
(a2 + b2 + c2 )(sen A + sen B + sen C)
+ ca sen B+
=
2
+

+ ab sen C + b c sen A 2(ABC)


=

(a2 + b2 + c2 )(sen A + sen B + sen C)


+ 4(ABC).
2

Por lo tanto, (A1 A2 B1 B2 C1 C2 ) 13(ABC) si y slo si


o
9abc
(a2 + b2 + c2 )(sen A + sen B + sen C)
9(ABC) =
.
2
4R
1
sen A
a = 2R , tenemos
9abc , o lo que es lo
4R

Usando la ley de los senos,


y slo si
o

(a2 +b2 +c2 )(a+b+c)


4R

que la desigualdad es cierta si


mismo

(a2 + b2 + c2 )(a + b + c) 9abc.

2.6 Desigualdades con reas y per


a
metros

89

La ultima desigualdad se deduce ya sea de la desigualdad M G M A, de la

desigualdad del reacomodo o bien de la desigualdad de Tchebychev. Adems,


a
se da la igualdad solamente en el caso en que a = b = c.
Ejemplo 2.6.3 (China, 1988 y 1993) Se tienen dos circunferencias concntricas
e
de radios R y R1 (R1 > R) y un cuadriltero convexo ABCD inscrito en la
a
circunferencia pequea. Las extensiones de AB, BC, CD y DA intersectan a
n
la circunferencia grande en C1 , D1 , A1 y B1 , respectivamente. Muestre que:
(i)

(ii)

R1
per
metro de A1 B1 C1 D1

.
per
metro de ABCD
R
(A1 B1 C1 D1 )

(ABCD)

R1
R

, donde (ABCD) denota el rea del cuadriltero.


a
a
A1

B1

A
D
O
B
C

C1

D1

Para demostrar (i), utilizamos la desigualdad de Ptolomeo (ver ejercicio 2.11)


aplicada a los cuadrilteros OAB1 C1 , OBC1 D1 , OCD1 A1 y ODA1 B1 , sta
a
e
nos asegura que
AC1 R1 B1 C1 R + AB1 R1

BD1 R1 C1 D1 R + BC1 R1

(2.8)

CA1 R1 D1 A1 R + CD1 R1

DB1 R1 A1 B1 R + DA1 R1 .
Luego, al sumar estas desigualdades y descomponer AC1 , BD1 , CA1 y DB1
como AB + BC1 , BC + CD1 , CD + DA1 y DA + AB1 , respectivamente,
tenemos
R1 per
metro (ABCD) + R1 (BC1 + CD1 + DA1 + AB1 )

R per
metro (A1 B1 C1 D1 ) + R1 (AB1 + BC1 + CD1 + DA1 ).

90

Desigualdades Geomtricas
e

Por lo tanto,
R1
per
metro (A1 B1 C1 D1 )

.
per
metro (ABCD)
R
Para demostrar (ii), usaremos el hecho que (ABCD) = ad sen A+bc sen A =
2
sen A
(ad + bc) y tambin que (ABCD) = ab sen B+cd sen B = sen B (ab + cd),
e
2
2
2
donde A = DAB y B = ABC.
A1

B1

w
d
D

a
O

B
b

z
C

D1

C1

Como (AB1 C1 ) = x(a+y)sen (180 A) =


2
a
dad (AB1 C1 ) = x(a+y) . Anlogamente,
ad+bc
(ABCD)
(DA1 B1 )
(ABCD)

w(d+x)
ab+cd .

x(a+y)sen A
, podemos deducir la igual2
(BC1 D1 )
y(b+z) (CD1 A1 )
z(c+w)
(ABCD) = ab+cd , (ABCD) = ad+bc ,

Luego,

x(a + y) + z(w + c) y(b + z) + w(d + x)


(A1 B1 C1 D1 )
=1+
+
.
(ABCD)
ad + bc
ab + cd
La potencia de un punto de la circunferencia grande con respecto a la circunfe2
rencia pequea es igual a R1 R2 . En particular, las potencias de A1 , B1 , C1
n
y D1 son la misma. Por otro lado, tenemos que estas potencias son w(w + c),
x(x + d), y(y + a) y z(z + b), respectivamente.
Sustituyendo en la ecuacin anterior, tenemos que la razn de las reas es
o
o
a
(A1 B1 C1 D1 )
=
(ABCD)
2
= 1 + (R1 R2 )

x
z
y
w
+
+
+
,
y(ad + bc) w(ad + bc) z(ab + cd) x(ab + cd)

La desigualdad M G M A, nos permite deducir que


(A1 B1 C1 D1 )
1+
(ABCD)

2
4(R1 R2 )

(ad + bc)(ab + cd)

2.6 Desigualdades con reas y per


a
metros

91

Ahora bien, como 2 (ad + bc)(ab + cd) ad + bc + ab + cd = (a + c)(b + d)

2R)
+ b + c + d)2 (4 4
= 8R2 , las dos primeras desigualdades se dan
por la desigualdad M G M A, y la ultima se debe a que el cuadrado es de los

cuadrilteros inscritos en una circunferencia, el de mayor per


a
metro. As

1
4 (a

2
4(R1 R2 )
(A1 B1 C1 D1 )
1+
=
(ABCD)
4R2

R1
R

Desde luego, las igualdades en i) y ii) ocurren cuando ABCD es un cuadrado y


unicamente en este caso. Ya que para reducir la desigualdad (2.8) a una igualdad,

se debe tener que los cuatro cuadrilteros OAB1 C1 , OBC1 D1 , OCD1 A1 y


a
ODA1 B1 sean c
clicos. Luego, OA es bisectriz del ngulo BAD, lo mismo
a
sucede con OB, OC y OD.
Existen problemas que a pesar de no ser planteados de forma geomtrica nos
e
invitan a buscar relaciones geomtricas, como en el siguiente ejemplo.
e
Ejemplo 2.6.4 Si a, b, c son nmeros positivos con c < a y c < b, se tiene que
u
c(a c) + c(b c) ab.
Consideremos los tringulos issceles ABC y ACD, los cuales comparten el
a
o

lado AC de longitud 2 c; tomamos el primero con lados iguales, AB = BC,

de longitud igual a a y el segundo que satisfaga que CD = DA y de longitud

igual a b.
D

B
El rea del papalote ABCD es por un lado
a
(ABCD) = (ABC) + (ACD) =
y por otro, (ABCD) = 2(ABD) =

c(a c) +

2 ab sen BAD
.
2

b(b c)

92

Desigualdades Geomtricas
e

Esta ultima forma de calcular el rea muestra claramente que (ABCD)

a
de donde se sigue el resultado buscado.

ab,

Podemos dar otra solucin del ejemplo anterior. Como y BD son perpendio
AC

culares, por el teorema de Pitgoras, tenemos DE = b c y EB = a c.


a
Por la desigualdad de Ptolomeo (ver ejercicio 2.11)



( b c + a c)(2 c) a b + a b,
de donde el resultado se sigue.
Ejercicio 2.59 En cada lado de un cuadrado con lados de longitud uno, se
escoge un punto. Los cuatro puntos forman un cuadriltero con lados de longitud
a
a, b, c y d, muestre que:
(i) 2 a2 + b2 + c2 + d2 4,

(ii) 2 2 a + b + c + d 4.
Ejercicio 2.60 En cada lado de un hexgono regular de lados de longitud 1 se
a
escoge un punto. Los seis puntos forman un hexgono de per
a
metro h. Muestre

que 3 3 h 6.
Ejercicio 2.61 Considere las tangentes al inc
rculo, de un tringulo ABC, que
a

son paralelas a los lados del tringulo. Estas determinan junto con los lados del
a
tringulo un hexgono T . Muestre que
a
a
per
metro de T

2
per
metro de (ABC).
3

Ejercicio 2.62 Encontrar el radio del c


rculo de rea mxima que puede cubrirse
a
a
con tres c
rculos de radio 1.
Ejercicio 2.63 Encontrar el radio del c
rculo de rea mxima que puede cubrirse
a
a
con tres c
rculos de radios r1 , r2 y r3 .
Ejercicio 2.64 Dentro de un cuadrado de lado 1, coloque dos cuadrados ajenos.
Si las longitudes de los lados de los dos cuadrados son a y b, respectivamente,
muestre que a + b 1.
Ejercicio 2.65 Un cuadriltero convexo est inscrito en una circunferencia de
a
a
radio 1, de manera que uno de sus lados es un dimetro y los otros son de
a
longitudes a, b, c. Muestre que abc 1.

2.7 Teorema de Erds-Mordell


o

93

Ejercicio 2.66 Sea ABCDE un pentgono convexo de rea (ABCDE) y tal


a
a
que las reas de los tringulos ABC, BCD, CDE, DEA y EAB son iguales,
a
a
muestre que:
(ABCDE)
(ABCDE)
< (ABC) <
,
4
3

5+ 5
(ii) (ABCDE) =
(ABC).
2

(i)

Ejercicio 2.67 Si AD, BE y CF son las alturas del tringulo ABC, muestre
a
que
per
metro (DEF ) s,
donde s es el semiper
metro de ABC.
Ejercicio 2.68 Las longitudes de las bisectrices internas de un tringulo son
a
menores o iguales a 1, muestre que el rea de dicho tringulo es menor o igual
a
a

a 33 .
Ejercicio 2.69 Si a, b, c, d son las longitudes de los lados de un cuadriltero
a
convexo de rea (ABCD), muestre que:
a
ab + cd
(i) (ABCD)
,
2
ac + bd
,
(ii) (ABCD)
2
a+c
b+d
(iii) (ABCD)
.
2
2

2.7.

Teorema de Erds-Mordell
o

Teorema 2.7.1 (Teorema de Pappus) Sean ABC un tringulo, AA B B y


a
CC A A dos paralelogramos construidos sobre ACy AB, de manera que ambos
estn hacia adentro o ambos hacia afuera del tringulo. Sea P la interseccin
e
a
o
A con C A . Construyamos otro paralelogramo BP P C sobre BC de
de B
manera que BP sea paralela a AP y de la misma longitud. Entonces se tiene
la siguiente relacin entre las reas,
o
a
(BP P C) = (AA B B) + (CC A A).

94

Desigualdades Geomtricas
e

Demostracin.
o

P
A

C
C

B
P

Teorema 2.7.2 (Teorema de Erds-Mordell) Sea P un punto arbitrario deno


tro o sobre la frontera de un tringulo ABC. Si pa , pb , pc son las distancias de
a
P a los lados a, b, c de ABC, respectivamente, entonces
P A + P B + P C 2 (pa + pb + pc ) .
Adems, se da la igualdad si y slo si el tringulo ABC es equiltero y P es el
a
o
a
a
circuncentro.
Demostracin. (Kazarino) Al tringulo ABC, lo reejamos con respecto a
o
a
la bisectriz BL del ngulo B. Sean A y C los reejados de A y C. Al punto P
a
no lo reejamos. Ahora consideramos los paralelogramos determinados por B,
P y A , y por B, P y C .
C
A
P

C
B
A
La suma de las reas de estos paralelogramos es cpa + apc y es igual al rea del
a
a
P P C , donde A P es paralela a BP y de igual longitud. El
paralelogramo A
a
rea de A P P C es menor o igual que b P B. Adems, las reas son iguales
a
a
si BP es perpendicular a A C si y slo si P est sobre BO, donde O es el
o
a
circuncentro de ABC 9 .
BP es perpendicular a A C si y slo si P BA = 90 A , pero A = A y
o
OBC = 90 A, luego P deber de estar sobre BO.
a
9

2.7 Teorema de Erds-Mordell


o

95
P

a
pc P

b
P

pa
B
Luego,

B
cpa + apc bP B.

Por lo tanto,

c
a
P B pa + pc .
b
b

Anlogamente,
a
PA

c
b
pc + pb
a
a

b
a
P C pa + pb .
c
c

Al sumar las desigualdades, tenemos


P A+P B+P C

b c
+
c b

pa +

a b
c a
pb +
+
+
a c
b a

pc 2 (pa + pb + pc ) ,

c
ya que b + b 2. Adems, tenemos la igualdad si y slo si a = b = c y P se
a
o
c
encuentra en AO, BO y CO. Es decir, si el tringulo es equiltero y P = O.
a
a

Ejemplo 2.7.3 Con la notacin del teorema de Erds-Mordell, muestre que


o
o
aP A + bP B + cP C 4(ABC).

Considere los dos paralelogramos que determinan B, C, P y B, A, P , como se


muestra en la gura, y el paralelogramo que se construye siguiendo el teorema
de Pappus. Es claro que
bP B apa + cpc .

96

Desigualdades Geomtricas
e

A
c

pc P

pa
a

De manera anloga, se tienen


a
aP A bpb + cpc ,

cP C apa + bpb .

Por lo tanto,
aP A + bP B + cP C 2(apa + bpb + cpc ) = 4(ABC).
Ejemplo 2.7.4 Con la notacin del teorema de Erds-Mordell, muestre que
o
o
pa P A + pb P B + pc P C 2 (pa pb + pb pc + pc pa ) .
Como en el ejemplo anterior, tenemos que aP A bpb + cpc . Por lo que,
pa P A

c
b
pa pb + pc pa .
a
a

c
a
b
a
pa pb + pb pc , pc P C pc pa + pb pc .
b
b
c
c
Al sumar las tres desigualdades, obtenemos
Anlogamente, tenemos que pb P B
a

a b
b c
+
+
pa pb +
b a
c b
2 (pa pb + pb pc + pc pa ) .

pa P A + pb P B + pc P C

pb pc +

c a
pc pa
+
a c

Ejemplo 2.7.5 Con la notacin del teorema de Erds-Mordell, muestre que


o
o
2

1
1
1
+
+
PA PB PC

1
1
1
+ .
+
pa pb pc

2.7 Teorema de Erds-Mordell


o

97

C1

B1

A
P

C1

B
A
1

A1

Hagamos la inversin en la circunferencia de centro P y radio d = pb . Si A ,


o

B , C son los inversos de A, B, C, respectivamente, y A , B1 , C1 , los inversos


1
de A1 , B1 , C1 , respectivamente, tenemos que
P A P A = P B P B = P C P C = d2

P A1 P A = P B1 P B1 = P C1 P C1 = d2 .
1


Ms an, A , B y C se encuentran sobre B1 C1 , C1 A , y A B1 , respectivaa u
1
1
, P B y P C son perpendiculares a B C , C A y
mente, y los segmentos P A
1 1
1 1

A B1 , respectivamente.
1

Aplicando el teorema de Erds-Mordell al tringulo A B1 C1 , obtenemos que
o
a
1
+ P B + P C 2 (P A + P B + P C ).
P A1
1
1
Pero como,

P A =
1

d2
d2
d2

, P B1 =
, P C1 =
,
P A1
P B1
P C1

P C =

d2
,
PC

P B =

d2
d2
, P A =
,
PB
PA

luego,
d2

1
1
1
+
+
P A1 P B1 P C1

2d2

1
1
1
+
+
PA PB PC

por lo tanto,
2

1
1
1
+
+
PA PB PC

1
1
1
+
+
pa pb pc

98

Desigualdades Geomtricas
e

Ejemplo 2.7.6 Utilizando la notacin del teorema de Erds-Mordell, muestre


o
o
que
R
(pa + pb ) (pb + pc ) (pc + pa ) .
PA PB PC
2r

A
b

pc P

pa
c

C1

Sea C1 sobre BC tal que BC1 = AB. Luego, AC1 = 2c sen B , y el teorema de
2
Pappus nos garantiza que, P B 2c sen B c pa + c pc . Por lo tanto,
2
PB

pa + pc
.
2 sen B
2

Anlogamente,
a
PA

pb + pc
2 sen A
2

PC

pa + pb
.
2 sen C
2

Luego, al multiplicar obtenemos


PA PB PC

1
8 sen A
2

1
sen B
2

sen C
2

(pa + pb ) (pb + pc ) (pc + pa ) ,

La solucin del ejercicio 2.42, nos ayuda a probar que sen A


o
2
r
, de donde el resultado se sigue inmediatamente.
4R

sen B
2

sen C =
2

Ejemplo 2.7.7 (IMO, 1991) Sea P un punto interior del tringulo ABC. Muesa
tre que alguno de los ngulos P AB, P BC, P CA es menor o igual a 30 .
a
Tracemos A1 , B1 y C1 las perpendiculares desde P a los lados BC, CA y AB,
respectivamente. Por el teorema de Erds Mordell, obtenemos que P A + P B +
o
P C 2P A1 + 2P B1 + 2P C1 .

2.7 Teorema de Erds-Mordell


o

99

C1

B1

A1

Luego, alguna de las desigualdades siguientes deber ocurrir,


a
P A 2P C1 , P B 2P A1

P C 2P B1 .

Si por ejemplo, P A 2P C1 , se tiene que 1 P C1 = sen P AB, por lo que


2
PA
P AB 30 o P AB 150 . Pero si, P AB 150 , entonces deber sua
ceder que P BC < 30 y en cualquier caso terminamos.
Ejemplo 2.7.8 (IMO, 1996) Sea ABCDEF un hexgono convexo, donde AB
a
es paralelo a DE, BC paralelo a EF y CD paralelo a F A. Sean RA , RC , RE
los circunradios de los tringulos F AB, BCD y DEF , respectivamente, y sea
a
P el per
metro del hexgono. Muestre que
a
RA + R C + R E

P
.
2

Sean M , N y P puntos dentro del hexgono de manera que M DEF , N F AB y


a
P BCD sean paralelogramos. Sea XY Z el tringulo formado por las rectas que
a
pasan a travs de B, D, F perpendiculares a F A, BC, DE, respectivamente,
e
donde B est sobre Y Z, D sobre ZX y F sobre XY . Observemos que M N P
a
y XY Z son tringulos semejantes.
a
X

E
F

N
P M

D
C

A
B

100

Desigualdades Geomtricas
e

Como los tringulos DEF y DM F son congruentes, stos tienen el mismo


a
e
circunradio; adems, como XM es un dimetro del circunc
a
a
rculo del tringulo
a
DM F , luego XM = 2RE . Anlogamente, Y N = 2RA y ZP = 2RC . As la
a
,
desigualdad que se desea mostrar se puede escribir como
XM + Y N + ZP BN + BP + DP + DM + F M + F N.
El caso M = N = P , es la desigualdad de Erds-Mordell, en la que se basa lo
o
que sigue de la prueba.
X
F
M

Sean Y , Z las respectivas reexiones de Y y Z con respecto a la bisectriz


interna del X. Sean G, H los pies de las perpendiculares de M , X sobre
Y Z , respectivamente. Como (XY Z ) = (XM Z ) + (Z M Y ) + (Y M X),
tenemos
XH Y Z = M F XZ + M G Y Z + M D Y X.
Si hacemos x = Y Z , y = ZX , z = XY , la igualdad es
xXH = xM G + zDM + yF M.
Como XHG = 90o , entonces XH = XG sen XGH XG . Adems, por
a
la desigualdad del tringulo, XG XM + M G, tenemos que
a
z
y
XM XH M G = DM + F M.
x
x
Anlogamente,
a
YN

ZP

z
x
F N + BN ,
y
y
x
y
BP + DP.
z
y

Al sumar estas tres desigualdades, obtenemos


z
y
x
z
y
x
XM + Y N + ZP DM + F M + F N + BN + BP + DP. (2.9)
x
x
y
y
z
z

2.7 Teorema de Erds-Mordell


o

101

Observemos que
y z
+
z y

z
y
BP + BN =
z
y

BP + BN
2

y z

z y

BP BN
2

Como los tringulos XY Z y M N P son semejantes, podemos denir r como


a
r=

BN BP
DP DM
FM FN
=
=
.
XY
YZ
ZX

Al aplicar la desigualdad
y
z
BP + BN
z
y

y
z

z
y

2, obtenemos

y z
+
z y

BP + BN
r

2
2
r yx zx
BP + BN

.
2 z
y

yx zx

z
y

Desigualdades anlogas se tienen para


a
y
x
FN + FM
y
x
x
z
DM + DP
x
z

r xz yz

,
2 y
x
r zy xy
.
DM + DP

2 x
z
FN + FM

Al sumar las desigualdades y sustituirlas en la ecuacin (2.9), tenemos que


o
XM + Y N + ZP BN + BP + DP + DM + F M + F N
como se deseaba.
Ejercicio 2.70 Utilizando la notacin del teorema de Erds-Mordell, muestre
o
o
que
4R
PA PB PC
pa pb pc .
r

Ejercicio 2.71 Utilizando la notacin del teorema de Erds-Mordell, muestre


o
o
que:
(i)
(ii)

P A2 P B 2 P C 2
+
+
12,
pb pc
pc pa
pa pb
PB
PC
PA
3,
+
+
pb + pc pc + pa pa + pb

102

Desigualdades Geomtricas
e

PA
PB
PC
(iii)
6,
+
+
pb pc
pc pa
pa pb
(iv) P A P B + P B P C + P C P A 4(pa pb + pb pc + pc pa ).
Ejercicio 2.72 Sea ABC un tringulo, P un punto arbitrario del plano y sean
a
pa , pb y pc las distancias de P a los lados del tringulo con longitudes a, b y
a
c, respectivamente. Si por ejemplo, P y A estn en distintos lados de la recta
a
BC, entonces pa es negativo, y lo mismo pasa en los otros dos casos. Muestre
que
PA + PB + PC

2.8.

b c
+
c b

pa +

c a
pb +
+
a c

a b
+
b a

pc .

Problemas de optimizacin
o

En esta seccin damos dos ejemplos clsicos conocidos como el problema de


o
a
Fermat-Steiner y el problema de Fagnano.
El problema de Fermat-Steiner. Este problema plantea encontrar un punto
en el interior del tringulo o sobre los lados, de manera que la suma de las
a
distancias del punto a los vrtices del tringulo sea m
e
a
nima. Presentaremos tres
soluciones donde resaltamos los mtodos para resolver el problema.
e
La solucin de Torricelli.
o

Esta se basa en los siguientes dos lemas.

Lema 2.8.1 (Lema de Viviani) La suma de las distancias de un punto interior


a los lados de un tringulo equiltero es igual a la altura del tringulo.
a
a
a
Demostracin. Sea P un punto interior del tringulo ABC. Tracemos el
o
a
tringulo A B C de lados paralelos a los lados de ABC, con P sobre C A
a
y B C sobre la recta por B y C.
A

M
N
P

P
B

P
L C

2.8 Problemas de optimizacin


o

103

Si L, M y N son los pies de las perpendiculares sobre los lados, es claro que
P M = N M , donde M es la interseccin de P N con A B . Adems, P M es
o
a
altura del tringulo equiltero A P P . Si A P es la altura del tringulo A P P
a
a
a
desde A , es claro que P M = A P . Sea L el pie de la altura desde el vrtice
e
A del tringulo A B C . Luego,
a
P L + P M + P N = P L + N M + P N = P L + A P = A P + P L = A L .

Observacin 2.8.2 (i) Hay una prueba del lema de Viviani que se basa en el
o
manejo de las reas. Denotemos por (ABC) el rea del tringulo ABC, entonces
a
a
a
(ABC) = (ABP ) + (BCP ) + (CAP ). Luego, si a es la longitud del lado del
tringulo y h la longitud de su altura, se tiene que ah = aP N + aP L + aP M ,
a
cancelando a obtenemos h = P N + P L + P M .
(ii) Otra demostracin del lema de Viviani se deduce de la siguiente gura
o
A
N

M
P
M

Lema 2.8.3 Si ABC es un tringulo con ngulos menores o iguales a 120 ,


a
a
hay un unico punto P tal que AP B = BP C = CP A = 120 . El punto

P se conoce como el punto de Fermat.

Demostracin. Primero veamos la existencia de P . Sobre los lados AB y CA


o
construimos tringulos equilteros ABC y CAB . Sus circunc
a
a
rculos se intersectan en A y en otro punto que denotamos por P .

104

Desigualdades Geomtricas
e
B
A
C
P
C

Como AP CB es c
clico, tenemos que, CP A = 180o B = 120o . Ana
logamente, por ser AP BC c
clico, AP B = 120o . Finalmente, BP C =
360o AP B CP A = 360o 120o 120o = 120o .
Para la unicidad, supongamos que Q cumple con AQB = BQC = CQA =
120o . Como AQB = 120o , el punto Q deber estar en el circunc
a
rculo de
ABC . Anlogamente, en el circunc
a
rculo de CAB , por lo que Q = P .
Estudiaremos ahora la solucin de Torricelli al problema de Fermat-Steiner. Dado
o
el tringulo ABC con ngulos menores o iguales a 120o , construimos el punto
a
a
de Fermat P , ste satisface que AP B = BP C = CP A = 120o . Ahora
e
por A, B y C tracemos perpendiculares a AP , BP y CP , respectivamente.
E

A A

F
P
Q

C
C

D
Estas perpendiculares determinan un tringulo DEF . Veamos que ste es ea
e
quiltero; como el cuadriltero P BDC es c
a
a
clico por tener en B y C ngulos
a
de 90 y como BP C = 120 , podemos deducir que BDC = 60 . Podemos
repetir este argumento en cada ngulo, luego DEF es equiltero.
a
a
Ahora bien, sabemos que la distancia de P a los vrtices del tringulo ABC es
e
a
igual a la longitud de la altura del tringulo equiltero DEF . Observemos que
a
a
cualquier otro punto Q, dentro del tringulo ABC, satisface que AQ A Q,
a

2.8 Problemas de optimizacin


o

105

donde A Q es la distancia de Q al lado EF , anlogamente, BQ B Q y


a
CQ C Q. Por lo tanto, AQ+BQ+CQ es mayor o igual que la longitud de la
altura de DEF que es AP +BP +CP , que a su vez es igual a A Q+B Q+C Q,
esto ultimo se sigue del lema de Viviani.

La solucin de Hofmann-Gallai. Esta forma de resolver el problema usa la


o
ingeniosa idea de rotar la gura para tratar de colocar los tres segmentos que
necesitamos, uno seguido de otro, para formar una poligonal y luego sumarlos.
Entonces, al unir con un segmento los dos puntos extremos y como el camino
ms corto entre ellos es este segmento de recta, se buscan entonces las condia
ciones bajo las cuales la poligonal est sobre tal recta. La demostracin la da
a
o
J. Hofmann en 1929, pero el mtodo para resolverlo era ya conocido y deber
e
a
ser atribuido al hngaro Tibor Gallai. Estudiemos entonces esta solucin.
u
o
Consideremos un tringulo ABC y un punto P dentro de l; tracemos AP , BP
a
e
y CP . Despus giremos la gura alrededor de B un ngulo de 60 , en sentido
e
a
positivo.
A
C
P
60

P
C

Tenemos varias cosas que sealar. Si C es la imagen de A y P la imagen


n
y BAC son equilteros. Adems, AP = P C y
de P , los tringulos BP P
a
a
a
BP = P B = P P , por lo que, AP + BP + CP = P C + P P + CP . La
trayectoria CP + P P + P C es m
nima cuando C, P , P y C son colineales.
Esta ultima condicin exige que C P B = 120 y BP C = 120 ; pero como

o
C P B = AP B, el punto P debe cumplir que AP B = BP C = 120 (y
entonces tambin CP A = 120 ).
e
Una ventaja de esta solucin es que proporciona otra caracterizacin del punto
o
o
de Fermat y otra forma de localizarlo. Si revisamos la demostracin podemos
o
, donde C es el tercer
resaltar que el punto P se encuentra en el segmento CC
vrtice del tringulo equiltero de lado AB, pero si en lugar de girar con centro
e
a
a
en B, se gira alrededor de C, se obtiene otro tringulo equiltero AB C y se
a
a
. Luego, podemos encontrar P
puede concluir que P se encuentra sobre BB

106

Desigualdades Geomtricas
e

como la interseccin de BB y CC .
o
La solucin de Steiner. Cuando tratamos de resolver problemas de mximos
o
a
y m
nimos nos enfrentamos principalmente a tres preguntas, (i) existe una
solucin?, (ii) es unica la solucin? (iii) qu propiedades caracterizan a la o
o

o
e
las soluciones? La solucin de Torricelli muestra que, de entre todos los puntos
o
del tringulo, este punto particular P , desde el cual se ven los tres lados del
a
tringulo con un ngulo de 120 , corresponde al menor valor de P A + P B +
a
a
P C. De esta forma responde a las tres preguntas que sealamos, hacindolo
n
e
adems de una manera elegante. Sin embargo, la solucin no da indicios de por
a
o
qu Torricelli elige tal punto; Cul fu su primer impulso para tomar ese punto?
e
a e
Probablemente esta pregunta no se podr responder. En lo que sigue veremos
a
una sucesin de ideas que nos llevan a descubrir por qu el punto de Fermat es
o
e
el ptimo, stas son debidas al gemetra suizo Jacob Steiner. Antes veamos los
o
e
o
siguientes dos lemas.
Lema 2.8.4 (Problema de Hern) Encontrar la trayectoria ms corta entre
o
a
dos puntos A y B que estn del mismo lado de una recta d, pasando por la
a
recta.
B
A
d
P
La distancia ms corta entre A y B, pasando por la recta d, la podemos encontrar
a
de la siguiente manera. Reejemos B sobre d para obtener un punto B ; el
segmento AB corta a d en un punto P que hace que AP + P B sea el
m
nimo entre las cantidades AP + P B, con P en d.
B
A
d
P

2.8 Problemas de optimizacin


o

107

Para convencernos basta observar que


AP + P B = AP + P B = AB AP + P B = AP + P B.
Este punto cumple con el siguiente principio de reexin: el ngulo de incidencia
o
a
es igual al ngulo de reexin. Desde luego, el punto con esta propiedad es el
a
o
m
nimo.
Lema 2.8.5 (Problema de Hern con circunferencia) Encontrar la trayeco
toria ms corta entre dos puntos A y B que estn fuera de una circunferencia
a
a
C, tocando a la circunferencia.
A

B
C

Veremos unicamente un esbozo de la solucin.

o
Sea D un punto sobre C, entonces tenemos que el conjunto {P : P A + P B =
DA+ DB} es una elipse ED , con focos en los puntos A y B, y que D pertenece
a ED . En general, Ed = {P : P A + P B = d}, donde d es un nmero positivo,
u
es una elipse con focos en A y B (si d > AB). Ms an, estas elipses tienen la
a u
o
propiedad de que Ed es un subconjunto del interior de Ed si y slo si d < d .
Luego, queremos encontrar el punto Q en C tal que QA + QB sea m
nima. El
punto ptimo Q pertenece a una elipse, precisamente a EQ . Esta elipse EQ no
o
intersecta a C en otro punto, de hecho, si C es otro punto en comn de EQ
u
, en el arco de circunferencia entre Q y C de C,
y C, entonces todo punto C
ser un punto interior de EQ , luego C A + C B < QA + QB y Q no ser el
a
a
o
ptimo, lo que es una contradiccin.
o
Luego, el punto Q que minimiza AQ+QB debe pertenecer a la elipse EQ que es
tangente a C en Q. La recta tangente comn a EQ y C deber ser perpendicular
u
a
al radio CQ, donde C es el centro de C y, por el principio de reexin de la
o
elipse (ngulo de incidencia es igual al ngulo de reexin), deber suceder que
a
a
o
a
CQ es bisectriz interna del ngulo AQB y entonces BQC = CQA.
a

108

Desigualdades Geomtricas
e

Q
A

Ahora volvamos a la solucin de Steiner del problema de Fermat-Steiner. Un


o
punto P que hace que la suma P A + P B + P C sea m
nima puede ser uno de
los vrtices A, B, C o un punto del tringulo distinto a los vrtices. En el primer
e
a
e
caso, si P es un vrtice, entonces un trmino de la suma P A+P B +P C es cero
e
e
y los otros dos son las longitudes de los lados del tringulo ABC que tienen en
a
comn el vrtice elegido. Luego, la suma ser m
u
e
a nima cuando el vrtice elegido
e
sea el opuesto al lado mayor del tringulo.
a
Para analizar el segundo de los casos, Steiner sigue una idea, muy util en los

problemas de optimizacin y que podr caer en la estrategia de divide y veno


a
cers, que es mantener jas algunas de las variables y optimizar las restantes.
a
Las condiciones resultantes en las variables no jas irn restringuiendo el espacio
a
solucin hasta llegar al ptimo global. Concretamente proced as
o
o
a :
Supongamos que P A tiene una longitud ja, es decir, P pertenece a la circunferencia de centro en A y radio P A. Ahora queremos encontrar el punto P que
hace que la suma P B +P C sea m
nima. Observemos que B tiene que estar fuera de dicho c
rculo, de lo contrario P A AB y por la desigualdad del tringulo
a
P B +P C > BC, entonces P A+P B = P C > AB +BC. Por lo tanto, B ser
a
un mejor punto (en lugar de P ). Anlogamente, C debe estar fuera del c
a
rculo.
Ahora bien, como B y C son puntos fuera de la circunferencia C = (A, P A),
el punto ptimo para minimizar P B + P C, con la condicin de que P sea un
o
o
punto sobre la circunferencia C es, por el lema 2.8.5, un punto Q sobre C, de
forma tal que esta circunferencia sea tangente a la elipse con focos B y C en
Q. Este punto Q satisface la condicin de que los ngulos AQB y CQA sean
o
a
iguales. Como los papeles de A, B y C puede intercambiarse, si ahora jamos el
punto B (y P B), entonces el punto ptimo Q satisface que los ngulos AQB
o
a
y BQC son iguales, y entonces AQB = BQC = CQA = 120 . Por lo
tanto, Q tiene que ser el punto de Fermat. Todo lo anterior funciona en este
segundo caso para asegurar que Q est dentro del tringulo ABC, si los ngulos
a
a
a
.
del tringulo no son mayores que 120
a
El problema de Fagnano

Este plantea encontrar, dentro de un tringulo


a

2.8 Problemas de optimizacin


o

109

acutngulo, un tringulo inscrito de per


a
a
metro m
nimo. Damos dos soluciones
clsicas, donde el reejar sobre rectas juega un papel central. Una debida a H.
a
Schwarz y la otra a L. Fejr.
e
La solucin de Schwarz. El matemtico alemn Hermann Schwarz dio la
o
a
a
siguiente solucin del problema basndose en dos observaciones las cuales apao
a
recen en los siguientes dos lemas. Estos lemas nos hacen ver que el tringulo
a
inscrito de menor per
metro es el tringulo formado con los pies de las alturas
a
del tringulo, el cual se conoce como tringulo rtico.
a
a
o
Lema 2.8.6 Sea ABC un tringulo. Sean D, E y F los pies de las alturas
a
sobre BC, CA y AB, desde los vrtices A, B y C, respectivamente. Entonces
e
los tringulos ABC, AEF , DBF y DEC son semejantes.
a
Demostracin. Basta ver que los primeros dos tringulos son semejantes, ya
o
a
que las otras semejanzas se demuestran de manera anloga.
a
A

E
F
H

Como estos dos tringulos tienen en comn al ngulo en A, bastar ver que
a
u
a
a
AEF = ABC. Pero es claro que AEF + F EC = 180 y como el
cuadriltero BCEF es c
a
clico se tiene que ABC + F EC = 180 , por lo
tanto AEF = ABC.

Lema 2.8.7 Con la notacin del lema anterior se tiene que, el reejado de D
o
con respecto a AB es colineal a E y F , y el reejado de D con respecto a CA
es colineal con E y F .
Demostracin. Es inmediata del lema anterior.
o

110

Desigualdades Geomtricas
e
A
D
E
F
D
B

Con estos elementos podemos ahora nalizar la solucin, propuesta por H.


o
Schwarz, al problema de Fagnano.
Demostraremos ahora que el tringulo de per
a
metro m
nimo es el tringulo
a
o
rtico. Denotemos a ste por DEF y consideremos otro tringulo LM N inscrito
e
a
en ABC.
C
E

N
F

C
Reejemos la gura completa sobre el lado BC, despus el tringulo resultante
e
a
lo reejamos sobre CA, luego sobre AB, sobre BC y nalmente en CA.
Tenemos en total seis tringulos congruentes y dentro de cada uno de ellos
a
tenemos su tringulo rtico y el otro tringulo inscrito LM N . El lado AB del
a
o
a
ultimo tringulo es paralelo al lado AB del primero, ya que como resultado de la

a
primera reexin, el lado AB se rota en sentido negativo un ngulo 2B, despus
o
a
e
en sentido negativo un ngulo 2A, en la tercera reexin queda invariante, en
a
o
la cuarta reexin gira un ngulo 2B en sentido positivo y en la quinta tambin
o
a
e
gira en sentido positivo un ngulo 2A. Luego, el ngulo total de rotacin del
a
a
o
segmento AB es cero.
El segmento F F es igual a dos veces el per
metro del tringulo rtico, ya que
a
o
F F se compone de seis pedazos, donde cada lado del rtico est tomado dos
o
a
veces. Tambin la l
e
nea quebrada N N es el doble del per
metro de LM N .

2.8 Problemas de optimizacin


o

111

Adems, N N es una recta paralela a F F y de la misma longitud, luego como


a
la longitud de la quebrada N N es mayor que la longitud del segmento N N ,
tenemos que el per
metro de DEF es menor que el per
metro de LM N .
La solucin de Fejr. La solucin que da el matemtico hngaro L. Fejr se
o
e
o
a
u
e
basa tambin en reexiones. Sea LM N un tringulo inscrito en ABC. Tomemos
e
a
la reexin L del punto L sobre el lado CA y L la reexin de L sobre el
o
o
lado AB, tracemos los segmentos M L y N L , es claro que LM = M L y que
L N = N L. Por lo que el per
metro de LM N satisface que,
LM + M N + N L = L N + N M + M L L L .
A

De lo anterior podemos asegurar que, si el punto L est jo, los puntos M y N


a
que hacen m
nimo el per
metro LM N son las intersecciones de L L con CA
y AB, respectivamente. Ahora veamos cul es la mejor opcin para el punto L.
a
o
Ya sabemos que el per
metro de LM N es L L , as que el punto L deber hacer

a
que esta cantidad sea m
nima. Es evidente que, AL = AL = AL y que AC y
AB son bisectrices de los ngulos LAL y L AL, respectivamente, por lo que
a
L AL = 2BAC = 2, el cual es un ngulo jo.
a
A

L
B

112

Desigualdades Geomtricas
e

La ley de los cosenos aplicada al tringulo AL L nos garantiza que


a
(L L )2 = (AL )2 + (AL )2 2AL AL cos 2
= 2AL2 (1 cos 2).

As L L es m
,
nima cuando AL lo sea, es decir, cuando AL sea la altura10 .
Un anlisis semejante desde los puntos B y C nos lleva a que tambin BM y
a
e
CN debern ser alturas. Luego, el tringulo LM N de per
a
a
metro m
nimo es el
tringulo rtico.
a
o
Ejercicio 2.73 Sea ABCD un cuadriltero c
a
clico convexo. Si O es la interseccin de las diagonales AC y BD, y P , Q, R, S son los pies de las perpendio
culares desde O sobre los lados AB, BC, CD, DA, respectivamente, muestre
que P QRS es el cuadriltero de per
a
metro m
nimo inscrito en ABCD.
Ejercicio 2.74 Sea P un punto dentro del tringulo ABC. Sean D, E y F
a
los puntos de interseccin de AP , BP y CP con los lados BC, CA y AB,
o
respectivamente. Determine P tal que el rea del tringulo DEF sea mxima.
a
a
a
Ejercicio 2.75 (IMO, 1981) Sea P un punto dentro del tringulo ABC. Sean
a
D, E y F los pies de las perpendiculares desde P a los lados BC, CA y AB,
CA
BC
respectivamente. Encuentre el punto P que minimiza a P D + P E + AB .
PF
Ejercicio 2.76 Sean P , D, E y F , como en el ejercicio 2.75. Para qu punto
e
2 + CE 2 + AF 2 es m
P , la suma BD
nima?
Ejercicio 2.77 Sean P , D, E y F como en el ejercicio 2.75. Para qu punto
e
P , el producto P D P E P F es mximo?
a
Ejercicio 2.78 Sea P un punto dentro del tringulo ABC. Para qu punto
a
e
P , la suma P A2 + P B 2 + P C 2 es m
nima?
Ejercicio 2.79 Para cada punto P del circunc
rculo del tringulo ABC, se
a
trazan las perpendiculares P M y P N a los lados AB y CA, respectivamente.
Determine en que punto P , la longitud M N es mxima y cul es su longitud.
a
a
10

Hasta aqu bastar para terminar la demostracin de Fejr al problema de Fagnano, ya

a
o
e
que si AL es altura y L L corta en E y F a los lados CA y AB, respectivamente, entonces
BE y CF son alturas. Veamos porqu. El tringulo AL L es issceles con L AL = 2A,
e
a
o
luego, AL L = 90 A y por simetr ELA = 90 A, por tanto CLE = A.
a
Luego, AELB es un cuadriltero c
a
clico, por lo tantoAEB = ALB = 90 , lo que implica
que BE sea altura. Anlogamente, CF es altura.
a

2.8 Problemas de optimizacin


o

113

Ejercicio 2.80 (Turqu 2000) Sea ABC un tringulo acutngulo con cira,
a
a
cunradio R, las longitudes de las alturas AD, BE y CF son ha , hb y hc ,
respectivamente. Sean ta , tb y tc las longitudes de las tangentes desde A, B y
C, respectivamente, al circunc
rculo de DEF . Muestre que
t2
t2
3
t2
a
+ b + c R.
ha hb hc
2
Ejercicio 2.81 Sean ha , hb , hc las longitudes de las alturas de un tringulo
a
ABC y pa , pb , pc las distancias desde un punto P a los lados BC, CA, AB,
respectivamente, donde P es un punto dentro del tringulo ABC. Muestre que:
a
(i)

ha hb hc
+
+
9.
pa
pb
pc

(ii) ha hb hc 27pa pb pc .
(iii) (ha pa )(hb pb )(hc pc ) 8pa pb pc .
Ejercicio 2.82 Si h es la longitud de la altura ms grande de un tringulo
a
a
acutngulo entonces r + R h.
a
Ejercicio 2.83 De los tringulos que tienen una base comn y un mismo
a
u
per
metro, el issceles es el que tiene mayor rea.
o
a
Ejercicio 2.84 De todos los tringulos con un per
a
metro dado, el tringulo
a
equiltero es el que tiene mayor rea.
a
a
Ejercicio 2.85 De todos los tringulos inscritos en un c
a
rculo dado, el tringulo
a
equiltero es el que tiene mayor per
a
metro.
Ejercicio 2.86 Si P es un punto dentro del tringulo ABC, l = P A, m = P B
a
y n = P C, muestre que
(lm + mn + nl)(l + m + n) a2 l + b2 m + c2 n.
Ejercicio 2.87 (IMO, 1961) Sean a, b y c las longitudes de los lados de un
tringulo ABC y sea (ABC) su area, muestre que
a

4 3(ABC) a2 + b2 + c2 .

114

Desigualdades Geomtricas
e

Ejercicio 2.88 Sea (ABC) el rea del tringulo ABC y sea F el punto de
a
a
Fermat del tringulo. Muestre que
a

4 3(ABC) (AF + BF + CF )2 .
Ejercicio 2.89 Sea P un punto dentro del tringulo ABC, muestre que
a
P A + P B + P C 6r.
Ejercicio 2.90 (El rea del tringulo pedal). Para un tringulo ABC y un
a
a
a
punto P del plano, se dene el tringulo pedal de P con respecto a ABC
a
como el tringulo A1 B1 C1 , donde A1 , B1 , C1 son los pies de las perpendiculares
a
desde P sobre BC, CA, AB, respectivamente. Muestre que el rea del tringulo
a
a
A1 B1 C1 satisface
(A1 B1 C1 ) =

(R2 OP 2 )(ABC)
,
4R2

donde O es el circuncentro y (ABC) es el rea del tringulo ABC. Concluya


a
a
que el tringulo pedal de rea mxima es el tringulo medial.
a
a
a
a

Cap
tulo 3

Problemas Recientes de
Desigualdades

Problema 3.1 (Bulgaria, 1995) Sean SA , SB y SC las reas de los heptgoa


a
nos regulares A1 A2 A3 A4 A5 A6 A7 , B1 B2 B3 B4 B5 B6 B7 y C1 C2 C3 C4 C5 C6 C7 ,
respectivamente. Suponga que A1 A2 = B1 B3 = C1 C4 , muestre que

SB + SC
1
<
< 2 2.
2
SA
Problema 3.2 (Repblicas Checa y Eslovaca, 1995) Sea ABCD un tetraedro
u
con
BAC + CAD + DAB = ABC + CBD + DBA = 180 .
Muestre que CD AB.
Problema 3.3 (Estonia, 1995) Sean a, b, c las longitudes de los lados de un
tringulo y , , los ngulos opuestos a los lados, respectivamente. Muestre
a
a
que si el inradio del tringulo es r, entonces
a
a sen + b sen + c sen 9r.

116

Problemas Recientes de Desigualdades

Problema 3.4 (Francia, 1995) Tres c


rculos con el mismo radio tienen un punto
comn. Si S es el rea del conjunto de puntos que son interiores al menos a dos
u
a
c
rculos, cmo debern colocarse los c
o
a
rculos para que S sea m
nima?
Problema 3.5 (Alemania, 1995) Sea ABC un tringulo con D y E puntos
a
sobre BC y CA, respectivamente, de manera que DE pasa por el incentro de
ABC. Si S = rea(CDE) y r es el inradio, muestre que S 2r 2 .
a
Problema 3.6 (Irlanda, 1995) Sean A, X, D puntos sobre una recta con X
entre A y D. Sea B un punto tal que ABX = 120 y sea C un punto entre

B y X. Muestre que 2AD 3 (AB + BC + CD).


Problema 3.7 (Corea, 1995) Un nmero nito de puntos del plano, tienen la
u
propiedad de que cada tres de ellos forman un tringulo de rea menor o igual a
a
a
1. Muestre que todos los puntos estn dentro o sobre los lados de un tringulo
a
a
de rea menor o igual a 4.
a
Problema 3.8 (Polonia, 1995) Para un entero positivo n jo, encuentre el
valor m
nimo de la suma
x1 +

xn
x2 x3
2
+ 3 + + n,
2
3
n

donde x1 , x2 , . . . , xn son nmeros positivos que satisfacen que la suma de sus


u
rec
procos es n.
Problema 3.9 (IMO, 1995) Sea ABCDEF un hexgono convexo con AB =
a
BC = CD y DE = EF = F A, tal que BCD = EF A = . Sean G y H
3
puntos en el interior del hexgono tales que AGB = DHE = 2 . Muestre
a
3
que
AG + GB + GH + DH + HE CF.
Problema 3.10 (Balcnica, 1996) Sean O el circuncentro y G el centroide de
a
un tringulo ABC. Sean R y r el circunradio y el inradio del tringulo. Muestre
a
a
que OG R(R 2r).
Problema 3.11 (China, 1996) Supngase que x0 = 0, xi > 0 para i =
o
1, 2, ..., n, y n xi = 1. Muestre que
i=1
n

i=1

xi

< .
1 + x0 + + xi1 xi + + xn
2

Problemas Recientes de Desigualdades

117

Problema 3.12 (Polonia, 1996) Sean n 2 y a1 , a2 , ..., an R+ con


1. Muestre que para x1 , x2 , ..., xn R+ con n xi = 1, se tiene
i=1
2
i<j

xi xj

n2
+
n1

n
i=1

n
i=1 ai

ai x2
i
.
1 ai

Problema 3.13 (Rumania, 1996) Sean x1 , x2 , ..., xn , xn+1 nmeros reales


u
positivos con x1 + x2 + + xn = xn+1 . Muestre que
n

n
i=1

xi (xn+1 xi )

i=1

xn+1 (xn+1 xi ).

Problema 3.14 (San Petesburgo, 1996) Sean M la interseccin de las diagoo


nales de un cuadriltero c
a
clico, N la interseccin de los segmentos que unen los
o
puntos medios de lados opuestos y O el circuncentro. Muestre que OM ON .
Problema 3.15 (Austria-Polonia, 1996) Si w, x, y y z son nmeros reales que
u
satisfacen w + x + y + z = 0 y w2 + x2 + y 2 + z 2 = 1. Muestre que
1 wx + xy + yz + zw 0.

Problema 3.16 (Taiwan, 1997) Sean a1 , ..., an nmeros positivos, tales que
u
ai1 +ai+1
es entero para toda i = 1, ..., n, a0 = an , an+1 = a1 y n 3. Muestre
ai
que
an1 + a1
an + a2 a1 + a3 a2 + a4
+
+
+ +
3n.
2n
a1
a2
a3
an
Problema 3.17 (Taiwan, 1997) Sea ABC un tringulo acutngulo, con cira
a
cuncentro O y circunradio R. Muestre que si AO corta al circunc
rculo de OBC
en D, BO corta al circunc
rculo de OCA en E y CO corta al circunc
rculo de
OAB en F , entonces OD OE OF 8R3 .
Problema 3.18 (APMO, 1997) Sea ABC un tringulo. La bisectriz interna
a
del ngulo A intersecta al segmento BC en X y al circunc
a
rculo en Y . Sea
AX
a
la = AY . Anlogamente, dena lb y lc . Muestre que
la
lb
lc
+
+
3
2A
2B
sen
sen
sen2 C

118

Problemas Recientes de Desigualdades

con igualdad si y slo si el tringulo es equiltero.


o
a
a
Problema 3.19 (IMO, 1997) Sean x1 , ..., xn nmeros reales con |x1 + + xn |
u
n+1
o
= 1 y |xi | 2 , para i = 1, ..., n. Muestre que existe una permutacin y1 , ...,
yn de x1 , ..., xn tal que
|y1 + 2y2 + + nyn |

n+1
.
2

Problema 3.20 (Repblicas Checa y Eslovaca, 1998) Sean a, b, c nmeros


u
u
reales positivos. Existe un tringulo con lados de longitud a, b, c si y slo si
a
o
existen nmeros x, y, z, tales que
u
a
y z
+ = ,
z y
x

z
x
b
+ = ,
x z
y

x y
c
+ = .
y
x
z

Problema 3.21 (Hungr 1998) Sea ABCDEF un hexgono centralmente


a,
a
simtrico y P , Q, R puntos sobre los lados AB, CD, EF , respectivamente.
e
Muestre que el rea del tringulo P QR es a lo ms un medio del rea del
a
a
a
a
hexgono.
a
Problema 3.22 (Irn, 1998) Sean x1 , x2 , x3 y x4 nmeros reales positivos que
a
u
cumplen x1 x2 x3 x4 = 1. Muestre que
a
x3 + x3 + x3 + x3 mx x1 + x2 + x3 + x4 ,
4
3
1
2

1
1
1
1
+
+
+
x1 x2 x3 x4

Problema 3.23 (Irn, 1998) Sean x, y, z nmeros mayores que 1 y tales que
a
u
1
1
1
+ y + z = 2. Muestre que
x

x+y+z

x1+

y1+

z 1.

Problema 3.24 (Mediterrnea, 1998) Sea ABCD un cuadrado inscrito en una


a
circunferencia. Si M es un punto en el arco AB, muestre que

M C M D 3 3M A M B.

Problemas Recientes de Desigualdades

119

Problema 3.25 (Nrdica, 1998) Sea P un punto dentro de un tringulo


o
a
equiltero ABC con lados de longitud a. Si las rectas AP , BP y CP ina
tersectan los lados BC, CA y AB del tringulo en L, M y N , respectivamente,
a
muestre que
P L + P M + P N < a.

Problema 3.26 (Espaa, 1998) Una recta que contiene al centroide G del
n
tringulo ABC intersecta el lado AB en P y el lado CA en Q. Muestre que
a
1
P B QC

.
P A QA
4

Problema 3.27 (Armenia, 1999) Sea O el centro del circunc


rculo del tringulo
a
acutngulo ABC. Las rectas CO, AO y BO intersectan, por segunda vez, al
a
circunc
rculo de los tringulos AOB, BOC y AOC en C1 , A1 y B1 , respectia
vamente. Muestre que
BB1 CC1
9
AA1
+
+
.
OA1
OB1 OC1
2

Problema 3.28 (Balcnica, 1999) Sea ABC un tringulo acutngulo y sean


a
a
a
L, M , N los pies de las perpendiculares desde G, el centroide de ABC, a los
lados BC, CA, AB, respectivamente. Muestre que
4
(LM N )
1
<
.
27
(ABC)
4
Problema 3.29 (Bielorusia, 1999) Sean a, b, c nmeros reales positivos tales
u
que a2 + b2 + c2 = 3. Muestre que
1
1
3
1
+
+
.
1 + ab 1 + bc 1 + ca
2
Problema 3.30 (Repblicas Checa y Eslovaca, 1999) Para nmeros positivos
u
u
a, b y c, muestre que
b
c
a
+
+
1.
b + 2c c + 2a a + 2b

120

Problemas Recientes de Desigualdades

Problema 3.31 (Irlanda, 1999) Sean a, b, c, d nmeros reales positivos con


u
a + b + c + d = 1. Muestre que
b2
c2
d2
1
a2
+
+
+
.
a+b b+c c+d d+a
2
Problema 3.32 (Italia, 1999) Sean D y E puntos sobre los lados AB y CA
de un tringulo ABC, respectivamente, de manera que DE es paralela a BC
a
y DE es tangente al inc
rculo de ABC. Muestre que
DE

AB + BC + CA
.
8

Problema 3.33 (Polonia, 1999) Sea D un punto sobre el lado BC del tringulo
a
ABC tal que AD > BC. Considere un punto E sobre el lado CA de manera
AE
BD
que EC = ADBC . Muestre que AD > BE.
Problema 3.34 (Rumania, 1999) Sean a, b, c nmeros reales positivos tales
u
que ab + bc + ca 3abc. Muestre que a + b + c a3 + b3 + c3 .
Problema 3.35 (Rumania, 1999) Sean x1 , x2 , ..., xn nmeros reales positivos
u
tales que x1 x2 xn = 1. Muestre que
1
1
1
+
+ +
1.
n 1 + x1 n 1 + x2
n 1 + xn
Problema 3.36 (Rumania, 1999) Sea n 2 un entero positivo y sean x1 ,
y1 , x2 , y2 , ..., xn , yn nmeros reales positivos tales que x1 + x2 + + xn
u
x1 y1 + x2 y2 + + xn yn . Muestre que
x1 + x2 + + x n

x1 x2
xn
+
+ +
.
y1
y2
yn

Problema 3.37 (Rusia, 1999) Sean a, b y c nmeros reales positivos con


u
1
abc = 1. Muestre que si a+b+c a + 1 + 1 entonces an +bn +cn a1 + b1 + c1 ,
n
n
n
b c
para todo n entero positivo.
Problema 3.38 (Rusia, 1999) Si {x} = x [x] es la parte fraccionaria del
nmero x. Muestre que, para cada entero positivo n,
u
n2

j
j=1

n2 1
.
2

Problemas Recientes de Desigualdades

121

Problema 3.39 (Rusia, 1999) Los nmeros reales positivos x, y satisfacen


u
x2 + y 3 x3 + y 4 . Muestre que
x3 + y 3 2.

Problema 3.40 (San Petesburgo, 1999) Sean x0 > x1 > > xn nmeros
u
reales. Muestre que
x0 +

1
1
1
+
+ +
xn + 2n.
x0 x1 x1 x2
xn1 xn

Problema 3.41 (Turqu 1999) Muestre que (a+3b)(b+4c)(c+2a) 60abc,


a,
para todos los nmeros reales que satisfacen 0 a b c.
u
Problema 3.42 (Reino Unido, 1999) Tres nmeros reales no negativos a, b y
u
c satisfacen a + b + c = 1. Muestre que
7(ab + bc + ca) 2 + 9abc.

Problema 3.43 (E.U., 1999) Sea ABCD un cuadriltero convexo. Muestre


a
que
|AB CD| + |AD BC| 2 |AC BD| .

Problema 3.44 (APMO, 1999) Sea {an } una sucesin de nmeros reales que
o
u
satisfacen, ai+j ai + aj , para toda i, j = 1, 2, . . .. Muestre que
a1 +

an
a2
+ +
an , para toda n N.
2
n

Problema 3.45 (IMO, 1999) Sea n 2 un entero dado.

122

Problemas Recientes de Desigualdades

(a) Determine la menor constante C para la cual se verica la desigualdad

4
1i<jn

xi xj (x2 + x2 ) C
i
j

1in

xi ,

para todos los nmeros reales no negativos x1 , ..., xn .


u

(b) Para esta constante C determine cuando ocurre la igualdad.


Problema 3.46 (Repblicas Checa y Eslovaca, 2000) Muestre que para todos
u
los nmeros reales positivos a y b,
u
3

a
+
b

2(a + b)

1 1
+
.
a b

Problema 3.47 (Corea, 2000) Los nmeros reales a, b, c, x, y, z satisfacen


u
a b c > 0 y x y z > 0. Muestre que
b2 y 2
c2 z 2
3
a2 x2
+
+
.
(by + cz)(bz + cy) (cz + ax)(cx + az) (ax + by)(ay + bx)
4

Problema 3.48 (Mediterranea, 2000) Sean P , Q, R, S los puntos medios de los


lados BC, CD, DA, AB, respectivamente, del cuadriltero convexo ABCD.
a
Muestre que
4(AP 2 + BQ2 + CR2 + DS 2 ) 5(AB 2 + BC 2 + CD2 + DA2 ).

Problema 3.49 (Austria-Polonia, 2000) Sean x, y, z nmeros reales no negau


tivos tales que x + y + z = 1. Muestre que
2 (1 x2 )2 + (1 y 2 )2 + (1 z 2 )2 (1 + x)(1 + y)(1 + z).

Problema 3.50 (IMO, 2000) Sean a, b, c nmeros reales positivos con abc = 1.
u
Muestre que
a1+

1
b

b1+

1
c

c1+

1
a

1.

Problemas Recientes de Desigualdades

123

Problema 3.51 (Balcnica, 2001) Sea a, b, c nmeros reales positivos tales


a
u
que abc a + b + c su suma. Muestre que

a2 + b2 + c2 3 abc.

Problema 3.52 (Brasil, 2001) Muestre que (a + b)(a + c) 2


para todos los nmeros reales positivos a, b, c.
u

abc(a + b + c)

Problema 3.53 (Polonia, 2001) Muestre que la desigualdad


n
i=1

ixi

n
2

xi
i

+
i=1

se tiene para cada entero n 2 y para todos los nmeros reales no negativos
u
x1 , x2 , ..., xn .
Problema 3.54 (Austria-Polonia, 2001) Muestre que
2<

a + b b + c c + a a3 + b3 + c3
+
+

3,
c
a
b
abc

donde a, b, c son las longitudes de los lados de un tringulo.


a
Problema 3.55 (IMO, 2001) Muestre que para cualesquiera nmeros reales
u
positivos a, b y c se cumple

a2

a
b
c
+
+
1.
2 + 8ca
2 + 8ab
+ 8bc
b
c

Problema 3.56 (Lista corta IMO, 2001) Muestre que para todos los nmeros
u
reales x1 , x2 , . . . , xn ,

x2
xn
x1
n.
2 + 1 + x 2 + x2 + + 1 + x 2 + + x 2 <
1 + x1
n
1
1
2
Problema 3.57 (Austria, 2002) Sean a, b, c nmeros reales tales que existen
u
, , {1, 1}, con a + b + c = 0. Determine el valor positivo ms
a
pequeo de
n

a3 +b3 +c3
abc

124

Problemas Recientes de Desigualdades

Problema 3.58 (Balcnica, 2002) Muestre que


a
2
2
27
2
+
+

,
b(a + b) c(b + c) a(c + a)
(a + b + c)2
para cualesquiera nmeros reales positivos a, b, c.
u
Problema 3.59 (Canad, 2002) Muestre que para cualesquiera nmeros reales
a
u
positivos a, b, c,
a3
b3
c3
+
+
a + b + c,
bc ca ab
y determine cundo ocurre la igualdad.
a
Problema 3.60 (Irlanda, 2002) Muestre que para cualesquiera nmeros reales
u
positivos x, y, z menores que 1, se tiene que

3 3 xyz
y
z
x
+
+

1x 1y 1z
1 3 xyz
Problema 3.61 (Rioplatense, 2002) Sean a, b, c nmeros reales positivos.
u
Muestre que
a
1
+
b+c 2

1
b
+
c+a 2

1
c
+
a+b 2

1.

Problema 3.62 (Rioplatense, 2002) Sean a, b, c nmeros reales positivos.


u
Muestre que
9
1 1 1
a+b b+c c+a
+ 2 + 2
+ + + .
2
c
a
b
a+b+c a b
c

Problema 3.63 (Rusia, 2002) Muestre que x + y + z xy + yz + zx,


para nmeros reales positivos x, y, z tales que x + y + z = 3.
u
Problema 3.64 (APMO, 2002) Los nmeros reales positivos a, b, c satisfacen
u
1
+ 1 + 1 = 1. Muestre que
a
b
c

a + bc + b + ca + c + ab abc + a + b + c.

Problemas Recientes de Desigualdades

125

Problema 3.65 (Irlanda, 2003) Las longitudes a, b, c de los lados de un


tringulo cumplen que a + b + c = 2. Muestre que
a
1 ab + bc + ca abc 1 +

1
.
27

Problema 3.66 (Rumania, 2003) Muestre que en cualquier tringulo ABC la


a
siguiente desigualdad es verdadera

1
1
1
3
+
+

,
mb mc mc ma ma mb
S
donde S es el rea del tringulo y ma , mb , mc son las longitudes de las mediaa
a
nas.
Problema 3.67 (Rumania, 2003) Sean a, b, c, d nmeros reales positivos con
u
abcd = 1. Muestre que
1 + ab 1 + bc 1 + cd 1 + da
+
+
+
4.
1+a
1+b
1+c
1+d

Problema 3.68 (Rumania, 2003) En un tringulo ABC, sean la , lb , lc las


a
longitudes de las bisectrices, y sea s el semiper
metro. Muestre que

la + lb + lc 3s.

Problema 3.69 (Rusia, 2003) Sean a, b, c nmeros reales positivos con


u
a + b + c = 1. Muestre que
1
1
1
2
2
2
+
+

+
+
.
1a 1b 1c
1+a 1+b 1+c
Problema 3.70 (APMO, 2003) Muestre que
1

(an + bn ) n + (bn + cn ) n + (cn + an ) n < 1 +

2n
,
2

126

Problemas Recientes de Desigualdades

donde n > 1 es un entero y a, b, c son las longitudes de los lados de un tringulo


a
con per
metro uno.
Problema 3.71 (IMO, 2003) Dada n > 2 y nmeros reales x1 x2
u
xn , muestre que

i,j

2
|xi xj | (n2 1)
3

i,j

(xi xj )2 ,

donde la igualdad se da si y slo si x1 , x2 , ... , xn forman una progresin


o
o
aritmtica.
e
Problema 3.72 (Lista corta Iberoamericana, 2004) Si los nmeros positivos
u
x1 , x2 ,..., xn satisfacen que x1 + x2 + . . . + xn = 1, muestre que
x1
x2
xn
n2
+
+ +

.
x2 (x1 + x2 + x3 ) x3 (x2 + x3 + x4 )
x1 (xn + x1 + x2 )
3

Problema 3.73 (Repblicas Checa y Eslovaca, 2004) Sea P (x) = ax2 + bx + c


u
un polinomio cuadrtico con coecientes reales no negativos. Muestre que, para
a
cualquier nmero positivo x,
u
P (x)P

1
x

(P (1))2 .

Problema 3.74 (Croacia, 2004) Muestre que la desigualdad


a2
b2
c2
3
+
+

(a + b)(a + c) (b + c)(b + a) (c + a)(c + b)


4
es vlida para todos los nmeros reales positivos a, b, c.
a
u
Problema 3.75 (Estonia, 2004) Sean a, b, c nmeros reales positivos tales que
u
a2 + b2 + c2 = 3. Muestre que
1
1
1
+
+
1.
1 + 2ab 1 + 2bc 1 + 2ca

Problemas Recientes de Desigualdades

127

Problema 3.76 (Irn, 2004) Sean x, y, z nmeros reales para los cuales
a
u
xyz = 1. Muestre que
x4 + y 4 + z 4 + 3(x + y + z)

x2 x2 y 2 y 2 z 2 z 2
+
+
+
+
+ .
y
z
x
z
x
y

Problema 3.77 (Corea, 2004) Sean R y r el circunradio y el inradio del


tringulo acutngulo ABC, respectivamente. Suponga que A es el ngulo
a
a
a
mayor del tringulo. Sea M el punto medio de BC y sea X la interseccin de
a
o
las tangentes al circunc
rculo de ABC en B y C. Muestre que
AM
r

.
R
AX

Problema 3.78 (Moldovia, 2004) Muestre que para cualesquiera nmeros


u
reales a, b, c 0, la siguiente desigualdad se cumple

a3 + b3 + c3 a2 bc + b2 ca + c2 ab.

Problema 3.79 (Ucrania, 2004) Sean x, y, z nmeros reales positivos tales


u
que x + y + z = 1. Muestre que

xy + z + yz + x + zx + y 1 + xy + yz + zx.

Problema 3.80 (Ucrania, 2004) Sean a, b, c nmeros reales positivos tales


u
que abc 1. Muestre que
a3 + b3 + c3 ab + bc + ca.

Problema 3.81 (Rumania, 2004) Encuentre todos los nmeros reales positivos
u
a, b, c que satisfacen las desigualdades
4(ab + bc + ca) 1 a2 + b2 + c2 3(a3 + b3 + c3 ).

128

Problemas Recientes de Desigualdades

Problema 3.82 (Rumania, 2004) Los nmeros reales a, b, c satisfacen a2 +


u
b2 + c2 = 3. Muestre que
|a| + |b| + |c| abc 4.

Problema 3.83 (Rumania, 2004) Considere el tringulo ABC y sea O un punto


a
interior de ABC. Las rectas OA, OB, OC intersectan los lados del tringulo en
a
A1 , B1 , C1 , respectivamente. Sean R1 , R2 , R3 los radios de los circunc
rculos
de los tringulos OBC, OCA, OAB, respectivamente, y sea R el radio del
a
circunc
rculo del tringulo ABC. Muestre que
a
OB1
OC1
OA1
R1 +
R2 +
R3 R.
AA1
BB1
CC1

Problema 3.84 (Rumania, 2004) Sea n 2 un nmero entero y sean a1 , a2 ,


u
..., an nmeros reales. Muestre que para cualquier subconjunto no vac S de
u
o
{1, 2, ..., n}, la siguiente desigualdad se cumple
2

ai
iS

1ijn

(ai + + aj )2 .

Problema 3.85 (APMO, 2004) Sean a, b, c nmeros reales positivos. Muestre


u
que
(a2 + 2)(b2 + 2)(c2 + 2) 9(ab + bc + ca).
Problema 3.86 (Lista corta IMO, 2004) Sean a, b y c nmeros reales positivos
u
tales que ab + bc + ca = 1. Muestre que

3
3
1
3
+ 6(a + b + c)
.
3
abc
abc
Problema 3.87 (IMO, 2004) Sea n 3 un nmero entero. Sean t1 , t2 , . . . ,
u
tn nmeros reales positivos tales que
u
n2 + 1 > (t1 + t2 + + tn )

1
1
1
+ + +
t1 t2
tn

Problemas Recientes de Desigualdades

129

Muestre que ti , tj , tk son las longitudes de los lados de un tringulo, para todo
a
i, j, k, con 1 i < j < k n.
Problema 3.88 (Japn, 2005) Sean a, b y c nmeros reales positivos, tales
o
u
que a + b + c = 1. Muestre que

3
3
a 1 + b c + b 3 1 + c a + a 1 + a b 1.

Problema 3.89 (Rusia, 2005) Sean x1 , x2 , . . . , x6 nmeros reales tales que


u
1
x2 + x2 + + x2 = 6 y x1 + x2 + + x6 = 0. Muestre que x1 x2 x6 2 .
6
2
1
Problema 3.90 (Reino Unido, 2005) Sean a, b, c nmeros reales positivos.
u
Muestre que
a b c
+ +
b c a

1 1 1
+ +
a b
c

(a + b + c)

Problema 3.91 (APMO, 2005) Sean a, b y c nmeros reales positivos tales


u
que abc = 8. Muestre que
a2
(1 +

a3 )(1

b3 )

b2
(1 +

b3 )(1

c3 )

c2
(1 +

c3 )(1

a3 )

4
.
3

Problema 3.92 (IMO, 2005) Sean x, y, z nmeros reales positivos tales que
u
xyz 1. Muestre que
x5

y5 y2
z5 z2
x5 x2
+ 5
+ 5
0.
2 + z2
2 + x2
+y
y +z
z + x2 + y 2

Problema 3.93 (Balcanes, 2006) Sean a, b, c nmeros reales positivos, muestre


u
que
1
1
3
1
+
+

.
a(b + 1) b(c + 1) c(a + 1)
1 + abc
Problema 3.94 (Estonia, 2006) Sea O el circuncentro de un tringulo acutngua
a
, B y C los circuncentros de los tringulos BCO, CAO y
lo ABC y sea A
a

130

Problemas Recientes de Desigualdades

ABO, respectivamente. Muestre que el rea del tringulo ABC es menor o


a
a
igual que el rea del tringulo A B C .
a
a
Problema 3.95 (Lituania, 2006) Sean a, b, c nmeros reales positivos, muestre
u
que
1
1
1 1
1
1
1
+
+

+
+
.
a2 + bc b2 + ca c2 + ab
2 ab bc ca

Problema 3.96 (Turqu 2006) Sean a1 , a2 , . . . , an nmeros reales positivos


a,
u
tales que
a1 + a2 + + an = a2 + a2 + + a2 = A.
1
2
n
Muestre que
i=j

(n 1)2 A
ai
.

aj
A1

Problema 3.97 (Iberoamericana, 2006) Considere n nmeros reales a1 , a2 ,


u
. . . , an , no necesariamente distintos. Sea d la diferencia entre el mximo y el
a
m
nimo valor de los nmeros y sea s = i<j |ai aj |. Muestre que
u
(n 1)d s

n2 d
.
4

Determine las condiciones en los n nmeros para que se cumpla la igualdad.


u
Problema 3.98 (IMO, 2006) Determine el m
nimo nmero real M tal que la
u
desigualdad
ab(a2 b2 ) + bc(b2 c2 ) + ca(c2 a2 ) M (a2 + b2 + c2 )2 ,
se cumpla, para cualesquiera nmeros reales a, b, c.
u
Problema 3.99 (Bulgaria, 2007) Encuentre todos los enteros positivos n tal
que si a, b, c son nmeros reales no negativos con a + b + c = 3, entonces
u
abc(an + bn + cn ) 3.

Problemas Recientes de Desigualdades

131

Problema 3.100 (Bulgaria, 2007) Si a, b, c son nmeros reales positivos,


u
muestre que
(a + 1)(b + 1)2 (b + 1)(c + 1)2 (c + 1)(a + 1)2

+
+
a + b + c + 3.
3
3
3
3 c2 a2 + 1
3 a2 b2 + 1
3 b2 c2 + 1
Problema 3.101 (China, 2007) Si a, b, c son las longitudes de los lados de un
tringulo con a + b + c = 3, encuentre el m
a
nimo de
a2 + b2 + c2 +

4abc
.
3

Problema 3.102 (Grecia, 2007) Si a, b, c son las longitudes de los lados de


un tringulo, muestre que
a
(a + b c)4 (b + c a)4
(c + a b)4
+
+
ab + bc + ca.
b(b + c a) c(c + a b) a(a + b c)
Problema 3.103 (Irn, 2007) Si a, b, c son tres nmeros reales positivos,
a
u
muestre que
a+b b+c c+a
> 1.
+
+
ab bc ca
Problema 3.104 (Mediterranea, 2007) Sean x, y, z nmeros reales tales que
u
1
xy + yz + zx = 1. Muestre que xz < 2 . Es posible mejorar la cota de 1 ?
2
Problema 3.105 (Mediterranea, 2007) Sea x > 1 un nmero real positivo que
u
no sea un entero. Muestre que
[x]
x + {x}

[x]
x + {x}

x + [x]
{x}

{x}
x + [x]

>

9
,
2

donde [x] y {x} representan la parte entera y la parte fraccionaria de x, respectivamente.


Problema 3.106 (Per, 2007) Sean a, b, c nmeros reales positivos tales que
u
u
1
a + b + c a + 1 + 1 . Muestre que
b
c
a+b+c

2
3
+
.
a + b + c abc

132

Problemas Recientes de Desigualdades

Problema 3.107 (Rumania, 2007) Sean a, b, c nmeros reales positivos tales


u
que
1
1
1
+
+
1.
a+b+1 b+c+1 c+a+1
Muestre que
a + b + c ab + bc + ca.

Problema 3.108 (Rumania, 2007) Sea ABC un tringulo acutngulo con


a
a
AB = AC. Para todo punto P en el interior del tringulo ABC, considere la
a
circunferencia con centro en A y radio AP ; sean M y N las intersecciones de los
lados AB y AC con la circunferencia, respectivamente. Determine la posicin
o
de P de tal forma que la suma M N + BP + CP sea m
nima.
Problema 3.109 (Rumania, 2007) Los puntos M , N , P en los lados BC, CA,
AB del tringulo ABC, respectivamente, son tales que el tringulo M N P es
a
a
acutngulo. Sea x la longitud de la altura menor del tringulo ABC y X la
a
a
longitud de la altura mayor del tringulo M N P . Muestre que x 2X.
a
Problema 3.110 (APMO, 2007) Sean x, y, z nmeros reales positivos tales
u

que x + y + z = 1. Muestre que


x2 + yz
2x2 (y + z)

y 2 + zx
2y 2 (z + x)

z 2 + xy
2z 2 (x + y)

1.

Problema 3.111 (Bltica, 2008) Si los nmeros reales positivos a, b, c satisa


u
facen que a2 + b2 + c2 = 3, muestre que
b2
c2
(a + b + c)2
a2
+
+

.
2 + b + c2 2 + c + a2 2 + a + b2
12
Bajo que circunstancias se tiene la igualdad?
Problema 3.112 (Canad, 2008) Sean a, b, c nmeros reales positivos tales
a
u
que a + b + c = 1. Muestre que
3
a bc b ca c ab
+
+
.
a + bc b + ca c + ab
2

Problemas Recientes de Desigualdades

133

Problema 3.113 (Irn, 2008) Encuentre el menor nmero real K tal que, para
a
u
cualesquiera nmeros reales positivos x, y, z, se cumple la siguiente desigualdad
u

x y + y z + z x K (x + y)(y + z)(z + x).

Problema 3.114 (Irlanda, 2008) Si los nmeros reales positivos a, b, c, d


u
2 + b2 + c2 + d2 = 1. Muestre que
satisfacen que a
a2 b2 cd + ab2 c2 d + abc2 d2 + a2 bcd2 + a2 bc2 d + ab2 cd2

3
.
32

Problema 3.115 (Irlanda, 2008) Sean x, y, z nmeros reales positivos, tales


u
que xyz 1. Muestre que:
(a) 27 (1 + x + y)2 + (1 + y + z)2 + (1 + z + x)2 ,
(b) (1 + x + y)2 + (1 + y + z)2 + (1 + z + x)2 3(x + y + z)2 .
La igualdad es cierta si y slo si x = y = z = 1.
o
Problema 3.116 (Rumania, 2008) Si a, b, c son nmeros reales positivos, tales
u
que ab + bc + ca = 3, muestre que
1
1+

a2 (b

+ c)

1
1+

b2 (c

+ a)

1
1

+ c2 (a

+ b)

1
.
abc

Problema 3.117 (Rumania, 2008) Determine el mximo valor para el nmero


a
u
real k si
1
1
1
+
+
k k,
(a + b + c)
a+b b+c c+a
para cualesquiera nmeros reales a, b, c 0, que cumplen adems que a+b+c =
u
a
ab + bc + ca.
Problema 3.118 (Serbia, 2008) Sean a, b, c nmeros reales positivos tales que
u
a + b + c = 1. Muestre que
a2 + b2 + c2 + 3abc

4
.
9

134

Problemas Recientes de Desigualdades

Problema 3.119 (Vietnam, 2008) Sean x, y, z nmeros reales distintos y


u
no-negativos. Muestre que
1
1
4
1
.
+
+

2
2
2
(x y)
(y z)
(z x)
xy + yz + zx
Cundo se da la igualdad?
a
Problema 3.120 (IMO, 2008) (i) Si x, y, z son tres nmeros reales distintos
u
a 1 y tales que xyz = 1. Muestre que
y2
z2
x2
+
+
1.
(x 1)2 (y 1)2
(z 1)2
(ii) Muestre que la igualdad es cierta para un nmero innito de nmeros
u
u
racionales x, y, z.

Cap
tulo 4

Soluciones a los Ejercicios y


Problemas

En este cap
tulo presentamos las soluciones o sugerencias de los ejercicios y problemas que aparecen en este libro. En las secciones 1 y 2 damos las soluciones a
los ejercicios de los cap
tulos 1 y 2, respectivamente. En la seccin 3 las solucioo
nes a los problemas del cap
tulo 3. Le recomendamos al lector que no consulte
este cap
tulo sin antes haber intentado resolver los ejercicios y problemas l
e
mismo.

4.1.

Soluciones a los ejercicios del cap


tulo 1

Solucin 1.1 Se sigue de la denicin de a < b y la propiedad 1.1.1 para el


o
o
nmero a b.
u
Solucin 1.2 (i) Si a < 0, entonces a > 0. Use tambin que (a)(b) = ab.
o
e
(ii) (a)b > 0. (iii) a < b b a > 0, use ahora la propiedad 1.1.2. (iv) Use
1
la propiedad 1.1.2. (v) Si a < 0, entonces a > 0. (vi) a a = 1 > 0. (vii) Si
a < 0, entonces a > 0. (viii) Use (vi) y la propiedad 1.1.3. (ix) Muestre que
ac < bc y que bc < bd. (x) Use la propiedad 1.1.3 con a 1 > 0 y a > 0. (xi)
Use la propiedad 1.1.3 con 1 a > 0 y a > 0.
Solucin 1.3 (i) a2 < b2 b2 a2 = (b + a)(b a) > 0. (ii) Si b > 0,
o
entonces 1 > 0, ahora use el ejemplo 1.1.4 (ii).
b

136

Soluciones a los Ejercicios y Problemas

Solucin 1.4 Para (i), (ii) y (iii) use la denicin, y para (iv) y (v) recuerde
o
o
que |a|2 = a2 .
Solucin 1.5 (i) x |x| y x |x|. (ii) Considere |a| = |a b + b| y
o
|b| = |b a + a| y aplique la desigualdad del tringulo. (iii) (x2 + xy + y 2 )(x
a
y) = x3 y 3 . (iv) (x2 xy + y 2 )(x + y) = x3 + y 3 .
Solucin 1.6 Si a, b o c son cero, tenemos la igualdad. Entonces, suponga que
o
|a| |b| |c| > 0, ya que la desigualdad es simtrica en a, b y c. Dividiendo
e
entre |a|, la desigualdad es equivalente a
1+

c
c
c
c
b
b
b
b
+

1+
0.
+
1+
+ 1+ +
a
a
a
a a
a
a a

c
b
Como a 1 y a 1, se tiene que 1 +
Entonces, es suciente probar que

b
b
c
b
c
c
+
1+ +
+

a
a
a a
a a

b
a

b
a

= 1+

+ 1+

y 1+

c
a

c
= 1 + a.

c
b
0.
+
a a

Ahora, use la desigualdad del tringulo y el ejercicio 1.5.


a
Solucin 1.7 (i) Use que 0 b 1 y 1 + a > 0, para ver que
o
0 b(1 + a) 1 + a

0 b a 1 ab

ba
1.
1 ab

(ii) La desigualdad de la izquierda es clara. Como 1 + a 1 + b, se tiene que


1
1
1+b 1+a , ahora vea que,
b
a
b
a+b
a
+

+
=
1.
1+b 1+a
1+a 1+a
1+a
(iii) Para la desigualdad izquierda use que ab2 ba2 = ab(b a) es producto de
nmeros reales no negativos. Para la desigualdad derecha note que, b 1
u
b2 b b b2 , ahora termine as
,
ab2 ba2 ab2 b2 a2 = b2 (a a2 ) a a2 =
Solucin 1.8 Muestre, ms generalmente, que x <
o
a

1
que x > 2 1 + 1+x < 2.

1
1
1
( a)2 .
4
2
4

2 1+

Solucin 1.9 ax + by ay + bx (a b)(x y) 0.


o

1
1+x

>

2y

4.1 Soluciones a los ejercicios del cap


tulo 1

137

Solucin 1.10 Suponga que x y. Luego, utilice el ejercicio anterior con


o
1
1
y 2 , y y x .

x2 ,

Solucin 1.11 Observe que


o
(a b)(c d) + (a c)(b d) + (d a)(b c) = 2(a b)(c d) = 2(a b)2 0.
Solucin 1.12 Se sigue de
o
f (a, c, b, d) f (a, b, c, d) = (a c)2 (a b)2 + (b d)2 (c d)2
= (b c)(2a b c) + (b c)(b + c 2d)

= 2(b c)(a d) > 0.

f (a, b, c, d) f (a, b, d, c) = (b c)2 (b d)2 + (d a)2 (c a)2

= (d c)(2b c d) + (d c)(c + d 2a)

= 2(d c)(b a) > 0.

Solucin 1.13 Para que las expresiones estn bien denidas es necesario que
o
e

2
x 1 y x = 0. Multiplique el numerador y el denominador por 1 + 1 + 2x .
2

Realice simplicaciones, para obtener 2 2x + 1 < 7; ahora resuelva para x.


Solucin 1.14 Como 4n2 < 4n2 + n < 4n2 + 4n + 1, se tiene que, 2n <
o

4n2 + n < 2n + 1. Luego, su parte entera es 2n y lo que hay que mostrar es

1
entonces 4n2 + n < 2n + 4 , que es inmediato al tomar cuadrados.
Solucin 1.15 Como (a3 b3 )(a2 b2 ) 0, se tiene que a5 +b5 a2 b2 (a+b),
o
luego
ab
ab
abc2
c
2 2
= 2 2 2
.
=
a5 + b5 + ab
a b (a + b) + ab
a b c (a + b) + abc2
a+b+c
Anlogamente,
a
a5

bc
b5 +c5 +bc

a
a+b+c

ca
c5 +a5 +ca

b
a+b+c .

Por lo que,

ab
bc
ca
c
a
b
+
+

+
+
,
5 + ab b5 + c5 + bc c5 + a5 + ca
+b
a+b+c a+b+c a+b+c

pero,

c
a+b+c

a
a+b+c

b
a+b+c

c+a+b
a+b+c

= 1.

Solucin 1.16 Considere p(x) = ax2 + bx + c, por hiptesis p(1) = a + b + c y


o
o
p(1) = a b + c son no negativos. Como a > 0, el m
nimo de p se logra en b
2a

138

Soluciones a los Ejercicios y Problemas


2

b
ces, se tiene que a = (x1 + x2 )
y su valor es 4acb < 0. Si x1 , x2 son las ra
4a
c
a+b+c
ab+c
y a = x1 x2 , por lo que a = (1 x1 )(1 x2 ), a = (1 + x1 )(1 + x2 )
y ac = 1 x1 x2 . Observe que, (1 x1 )(1 x2 ) 0, (1 + x1 )(1 + x2 ) 0 y
a
1 x1 x2 0, garantizan que 1 x1 ,x2 1.

Solucin 1.17 Si las tres desigualdades ocurren, debe suceder que a, b y c son
o
1
menores que 1, y que a(1b)b(1c)c(1a) > 64 . Por otro lado, para 0 x 1
1
1
siempre sucede que x(1 x) 4 , se tiene que a(1 b)b(1 c)c(1 a) 64 .
Solucin 1.18 Use la desigualdad M G M A con a = 1, b = x.
o
1
Solucin 1.19 Use la desigualdad M G M A, con a = x y b = x .
o

Solucin 1.20 Use la desigualdad M G M A, con a = x2 y b = y 2 .


o
Solucin 1.21 En el ejercicio anterior sume x2 + y 2 de ambos lados.
o
Solucin 1.22 Use la desigualdad M G M A con a = x+y , b = x+y y use
o
x
y
tambin la M G M A, para x y y. O bien, reduzca al ejercicio 1.20.
e
b
Solucin 1.23 Use la desigualdad M G M A, con ax y x .
o

Solucin 1.24 Use la desigualdad M G M A, con


o
Solucin 1.25
o
0 < b a.

a+b
2

( a b)
ab =
=
2

(ab)2

,
2( a+ b)2

Solucin 1.26 x + y 2 xy.


o
Solucin 1.27 x2 + y 2 2xy.
o

Solucin 1.28 xy + zx 2x yz.


o
Solucin 1.29 Vea el ejercicio 1.27.
o
1
y

2 .
xy

Solucin 1.30
o

1
x

Solucin 1.31
o

xy
z

Solucin 1.32
o

x2 +(y 2 +z 2 )
2

yz
x

xy 2 z
zx

= 2y.

x y2 + z2 .

a
b

b
y a.

simplique y acote usando

4.1 Soluciones a los ejercicios del cap


tulo 1

139

Solucin 1.33 x4 + y 4 + 8 = x4 + y 4 + 4 + 4 4 4 x4 y 4 16 = 8xy.


o

Solucin 1.34 (a + b + c + d) 4 4 a b c d,
o

Solucin 1.35
o

a
b

b
+c+

c
d

d
a

abcd
bcda

44

1
a

a1
b1

a2
b2

+ +

an
bn

Solucin 1.38 an 1 > n a


o
na

n1
2

(a 1)

1+a++an1
n

Solucin 1.39 1 =
o

1+a
2

nn

n+1
2

>a

n1
2

1+b
2

a1 an
b1 bn

n1
2

, pero

1+c
2

1
c

1
d

44

1
x1

1
xn

+ +

a3
b

b3
c

bc = 3ab. Anlogamente,
a

Por lo que,

3
2( ab

b3
c

c3
a)+

b3
c

nn

1
x1 xn .

= n.

(a 1) an1 + + 1 >
1+a++an1
n

>

(n1)n
2

=a

n1
2


a b c = abc.

Solucin 1.40 Aplicando la desigualdad M G M A, se tiene,


o
3

1
abcd.

= 4.

Solucin 1.36 (x1 + + xn ) n n x1 xn ,


o
Solucin 1.37
o

1
b

c3
a

+ ca 3bc y

c3
a

a3
b
a3
b

(ab + bc + ca) 3(ab + bc + ca).

b3
c

+ bc

+ ab 3ca.

Segunda Solucin. Tambin se puede resolver aplicando el ejercicio 1.107.


o
e
Solucin 1.41 Si abc = 0, el resultado es claro. Si abc > 0, se tiene que
o
ab bc ca
1
+
+
=
c
a
b
2

b c
+
c b

+b

c a
+c
+
a c

a b
+
b a

1
(2a + 2b + 2c)
2
y el resultado se sigue.
Solucin 1.42 Aplique la desigualdad M G M A dos veces, a2 b + b2 c + c2 a
o
3abc, ab2 + bc2 + ca2 3abc.
Solucin 1.43
o

1+ab
1+a

abc+ab
1+a

= ab

1 + ab 1 + bc 1 + ca
+
+
= ab
1+a
1+b
1+c
3

1+c
1+a

1+c
1+a

+ bc

(abc)2 = 3.

1+a
1+b

+ ca

1+b
1+c

140

Soluciones a los Ejercicios y Problemas


1
a+b

Solucin 1.44
o
que

1
b+c

1
c+a

(a + b + c) 9 , es equivalente a mostrar
2

1
1
1
+
+
a+b b+c c+a

(a + b + b + c + c + a) 9,

lo cual se sigue del ejercicio 1.36. Para la otra desigualdad use


Vea el ejercicio 1.22.

1
a

1
b

4
a+b .

Solucin 1.45 Note que,


o
1
(1 + 1) + (1 + 1 ) + + (1 + n )
n + Hn
2
=
.
n
n

Ahora aplique la desigualdad M G M A.


1
1
Solucin 1.46 Dena, yi = 1+xi xi = yi 1 = 1yi . Ahora, obsero
yi
ve que y1 + . . . + yn = 1 implica que 1 yi = j=i yj , luego, j=i yj

(n 1)

j=i yj

1 yi
yi

xi =
i

1
n1

j=i yj

yi

(n 1)n

j=i yj

i
i

Solucin 1.47 Dena an+1 = 1 (a1 + + an ) y xi =


o
1, ..., n + 1. Aplique directamente el ejercicio 1.46.

n
i=1

n
1
i=1 1+ai
n

=
i,j

n
i=1

ai aj
=
(1 + aj ) ai

i>j

i=1

n
ai
i=1 1+ai

=1

1
=
ai

ai (n 1)

= (n1)n .

yi

Solucin 1.48
o

1
n1

1ai
ai ,

para i =

= n 1. Observe que

1
1
ai
ai

ai
1 + ai
1 + ai
i=1
i=1
i=1

( ai aj 1)( ai aj )2 ( ai + aj )
.

(1 + ai )(1 + aj ) ai aj

2+a +a

i
j
1
1
Como 1 1+ai + 1+aj = 1+ai +aj +ai aj , tenemos que ai aj 1. Por lo tanto,
los trminos de la ultima suma son positivos.
e

a2
n
i
i=1 ai +bi

Solucin 1.49 Sean Sa =


o
n

Sa Sb =

i=1

y Sb =

a2 b2
i
i
=
ai + bi

b2
n
i
i=1 ai +bi .
n

n
i=1

ai

bi = 0,
i=1

4.1 Soluciones a los ejercicios del cap


tulo 1

141

luego Sa = Sb = S. Se tiene entonces,


n

2S =
i=1

1
a2 + b2
i
i

ai + bi
2

n
i=1

(ai + bi )2
=
ai + bi

ai ,
i=1

de donde la desigualdad se sigue despus de usar el ejercicio 1.21.


e
Solucin 1.50 Como la desigualdad es homognea1 se puede suponer, sin
o
e
prdida de generalidad, que abc = 1. Haciendo x = a3 , y = b3 y z = c3 , la
e
desigualdad es equivalente a,
1
1
1
+
+
1.
x+y+1 y+z+1 z+x+1
Sean A = x + y + 1, B = y + z + 1 y C = z + x + 1, luego
1
1
1
+ + 1 (A 1)(B 1)(C 1) (A + B + C) + 1 0
A B C
(x + y)(y + z)(z + x) 2(x + y + z) 2
(x + y + z)(xy + yz + zx 2) 3.
Ahora, use que
1
2
x+y+z
xy + yz + zx
(xzy) 3 y
(xyz) 3 .
3
3
Segunda Solucin. Use las ideas de la solucin del ejercicio 1.15. Inicie de que
o
o
(a2 b2 )(a b) 0 para garantizar que a3 + b3 + abc ab(a + b + c), luego

a3

c
1

.
3 + abc
+b
abc(a + b + c)

Solucin 1.51 Note que abc


o
1
+1
a

1
+1
b

1
+1
c

a+b+c 3
3

1
27 .

1 1 1
1
1
1
1
+ + +
+
+
+
a b
c ab bc ca abc
3
3
1
1+
+
+
3
abc
3
2
abc
(abc)

= 1+

=
1

1
1+
3
abc

43 .

Una funcin f (a, b, . . .) es homognea si para t R se tiene que f (ta, tb, . . .) =


o
e
tf (a, b, . . .). Luego, una desigualdad de la forma f (a, b, . . .) 0, para el caso de una funcin homognea es equivalente a f (ta, tb, . . .) 0 para cualquier t > 0.
o
e

142

Soluciones a los Ejercicios y Problemas

Solucin 1.52 La desigualdad es equivalente a b+c b+c b+c 8. Use


o
a
b
c
ahora, la desigualdad M G M A para cada trmino del producto, y la desiguale
dad se resuelve inmediatamente.
Solucin 1.53 Note que,
o
b
c
(a + 1)(b + 1)(c + 1) 2
a
+
+
=
(a + 1)(b + 1) (b + 1)(c + 1) (c + 1)(a + 1)
(a + 1)(b + 1)(c + 1)
2
3
=1
,
(a + 1)(b + 1)(c + 1)
4
que es equivalente a (a + 1)(b + 1)(c + 1) 8, y esta ultima desigualdad es


b+1
c+1
a+1
a b c = 1.
inmediata de la desigualdad 2
2
2
Solucin 1.54 Vea que es similar al ejercicio 1.52.
o
Solucin 1.55 Aplique la desigualdad entre la media aritmtica y la media
o
e
armnica para obtener
o
2ab
=
a+b

1
a

2
+

1
b

a+b
.
2

Puede concluir tambin que si la igualdad se alcanza entonces a = b = c.


e
Solucin 1.56 Utilice primero que (a + b)2 4ab, luego tome en cuenta que
o
n
i=1

1
4
ai bi

n
i=1

1
.
(ai + bi )2

Ahora, use el ejercicio 1.36, para mostrar que


n

(ai + bi )2
i=1

i=1

1
n2 .
(ai + bi )2

Solucin 1.57 Por la desigualdad M G M A, se tiene que xy + yz 2y xz.


o

Sumando desigualdades similares se obtiene 2(xy+yz+zx) 2(x yz+y zx+

z xy). Nuevamente, por M G M A, se tiene que x2 + x2 + y 2 + z 2 4x yz.


Sumando los resultados similares, una vez ms, se llega a que x2 + y 2 + z 2
a

x yz + y zx + z xy. Ahora, sumando ambas desigualdades, se obtiene el


2

resultado (x+y+z) x yz + y zx + z xy.


3

4.1 Soluciones a los ejercicios del cap


tulo 1

143

Solucin 1.58 La desigualdad M GM A lleva a x4 +y 4 2x2 y 2 . Usando


o
que
M GM A nuevamente se tiene que 2x2 y 2 +z 2 8xyz. O bien, directamente
se tiene que
4 4 4

z2 z2
4 x y z
x4 + y 4 +
+
4
= 8xyz.
2
2
4
Solucin 1.59 Por M G M A, se tiene
o
x2
y2
+
2
y1 x1
La ultima desigualdad se deduce de

xy
(x 1)(y 1)

x
x1

8.

2, ya que (x 2)2 0.

Segunda Solucin. Sean a = x1, b = y1, que son nmeros positivos, entono
u
2
2
ces la desigualdad que se quiere demostrar es equivalente a (a+1) + (b+1) 8.
b
a
Ahora bien, por M G M A, tenemos que (a + 1)2 4a y (b + 1)2 4b. Luego,
2
(a+1)2
b
+ (b+1) 4 a + a 8. La ultima desigualdad es consecuencia del

b
a
b
ejercicio 1.24.
Solucin 1.60 Observe que (a, b, c) y (a2 , b2 , c2 ) se ordenan de la misma forma,
o
use la desigualdad (1.2).
Solucin 1.61 Por el ejercicio anterior
o
a3 + b3 + c3 a2 b + b2 c + c2 a.
1
1
Observe que ( a , 1 , 1 ) y ( a2 , b1 , c1 ) se ordenan de la misma forma. Entonces,
2
2
b c
usando la desigualdad (1.2), se obtiene

1
1
1
+
+
a3 b3 c3
1 1
1 1
1 1
+ 2 + 2

2 c
a
b a c b
a b c
=
+ +
c a b
= a2 b + b2 c + c2 a.

(ab)3 + (bc)3 + (ca)3 =

Sumando las dos desigualdades, se obtiene el resultado.


Solucin 1.62 Use la desigualdad (1.2) con (a1 , a2 , a3 ) = (b1 , b2 , b3 ) =
o
a b c
b c a

b , c , a y (a1 , a2 , a3 ) = c , a , b .

144

Soluciones a los Ejercicios y Problemas

Solucin 1.63 Use la desigualdad (1.2) con (a1 , a2 , a3 ) = (b1 , b2 , b3 ) =


o
1
1 1 1
, b , c y (a , a , a ) = 1 , 1 , a .
1 2 3
a
b c
Solucin 1.64 Suponga que a b c, y considere (a1 , a2 , a3 ) = (a, b, c).
o
Use la desigualdad del reacomodo (1.2) dos veces con (a , a , a ) = (b, c, a) y
1 2 3
(c, a, b), respectivamente. Desde luego, se est utilizando
a
(b1 , b2 , b3 ) =

1
1
1
,
,
b+ca c+ab a+bc

Solucin 1.65 Utilice la misma idea que en el ejercicio anterior, pero con n
o
variables.
Solucin 1.66 Use el ejercicio anterior y el hecho que
o

s
sa1

=1+

a1
sa1 .

Solucin 1.67 Aplique el ejercicio 1.65 a la sucesin a1 , . . ., an , a1 , . . ., an .


o
o
Solucin 1.68 Aplique el ejemplo 1.4.11.
o
Solucin 1.69 Note que 1 = (a2 + b2 + c2 ) + 2(ab + bc + ca), y use el ejercicio
o
anterior como sigue
a+b+c
1
=

3
3
por lo tanto,
evidente.

1
3

a2 + b2 + c2
3

2
a2 + b2 + c2 . Luego, 2(ab + bc + ca) 3 , y el resultado es

Segunda Solucin. La desigualdad es equivalente a 3(ab+bc+ca) (a+b+c)2


o
pero sta se reduce a ab + bc + ca a2 + b2 + c2 .
e

Solucin 1.70 Sea G = n x1 x2 xn la media geomtrica de los nmeros


o
e
u
2
1
dados y (a1 , a2 , ..., an ) = x1 , xGx2 , , x1 xGxn .
n
2
G
Utilice el corolario 1.4.2, se tiene que
n

an1 an
G
G
G
G
a1 a2
+
+ +
+
=
+
+ +
+ ,
a2 a3
an
a1
x2 x3
xn x1

luego,
1
x1

1
x2

n
+ +

1
xn

G.

4.1 Soluciones a los ejercicios del cap


tulo 1

145

Tambin, por el corolario 1.4.2,


e
n

a2
an
x1 x2
xn
a1
+
+ +
=
+
+ +
,
an a1
an1
G
G
G

entonces

x1 + x2 + + xn
.
n
Las igualdades ocurren si y slo si a1 = a2 = = an , es decir, si y slo si
o
o
x1 = x2 = = xn .
G

Solucin 1.71 La desigualdad es equivalente a


o
n1
n1
n1
a1 + a2 + + an

a1 an
a1 an a1 an
+
+ +
,
a1
a2
an

la cual se verica usando la desigualdad del reacomodo varias veces.


n
1
i=1 1ai

ai
Solucin 1.72 Primero note que n 1a =
o
i=1
i
Por la desigualdad M G M A se obtiene

1
n

n
i=1

1 ai

n
n

i=1

1
=
1 ai
1

1
n

n
i=1 (1

ai )

n
i=1 1

ai .

1
n

n
i=1 (1

ai )

n
.
n1

Ms an, la desigualdad de Cauchy-Schwarz sirve para mostrar que


a u
n
i=1

1 ai

n
i=1

(1 ai ) n =

n(n 1)

y
i=1

ai

n.

Solucin 1.73 (i) 4a + 1 < 4a+1+1 = 2a 1. (ii) Use la desigualdad de


o
+
2

Cauchy-Schwarz con u = ( 4a + 1, 4b + 1, 4c + 1) y v = (1, 1, 1).


Solucin 1.74 Suponga que a b c d (los otros casos son anlogos).
o
a
Entonces, si A = b + c + d, B = a + c + d, C = a + b + d y D = a + b + c, se
1
1
1
1
tiene que A B C D . Aplique la desigualdad de Tchebyshev dos veces
para mostrar que
a3 b3 c3 d3
1
+
+
+
(a3 + b3 + c3 + d3 )
A
B
C
D
4

1
1
1
1
+ + +
A B C
D

146

Soluciones a los Ejercicios y Problemas

1 2
(a + b2 + c2 + d2 )
16

1
1
1
1
+ + +
A B C
D

1 2
(a + b2 + c2 + d2 )(a + b + c + d)
16

A+B+C +D
3

1
1
1
1
+ + +
A B C
D

Ahora, use la desigualdad de Cauchy-Schwarz para ver que


a2 + b2 + c2 + d2 ab + bc + cd + da = 1
1
y use la desigualdad (A + B + C + D)( A +

1
B

1
C

1
D)

Solucin 1.75 Use la desigualdad del reacomodo con


o

a 3 b 3 c
a
3
, (b1 , b2 , b3 ) =
,
(a1 , a2 , a3 ) = 3 ,
b
c
a
b
y permutacin (a , a , a ) =
o
1 2 3

b
c,

a b c
+ +
b c a

c
a,

a
b

a2
+
bc

16.

b
c

c
a

, para obtener
b2
+
ca

c2
.
ab

Finalmente use que abc = 1.


Segunda Solucin. Aplique la desigualdad entre M G M A de la siguiente
o
manera:
2

1 a a b
3
3 a
3 aab
+ +
=
= a3 = a.

3 b
b c
bbc
bc
Anlogamente,
a
desigualdades.

1
3

b
c

+b+
c

c
a

by

1
3

c
a

c
a

a
b

c. Ahora sume las tres

Solucin 1.76 Por hiptesis, para toda k, se tiene que s 2xk > 0. Por la
o
o
desigualdad de Cauchy-Schwarz
n
k=1

x2
k
s 2xk

k=1

(s 2xk )

xk
k=1

= s2 .

4.1 Soluciones a los ejercicios del cap


tulo 1
Pero 0 <

n
k=1 (s

147

2xk ) = ns 2s, por lo que


n

k=1

x2
s
k

.
s 2xk
n2

Solucin 1.77 La funcin f (x) = x +


o
o

1 2
x

es convexa en R+ .

a
b
c
Solucin 1.78 La funcin f (a, b, c) = b+c+1 + a+c+1 + a+b+1 + (1 a)(1
o
o
b)(1 c) es convexa en cada una de las variables, luego su mximo se alcanza
a
en los extremos.

Solucin 1.79 Si x = 0, entonces la desigualdad se reduce a 1 + 1


o

1+y 2

2,

que es verdadera ya que y 0. Por simetr la desigualdad es cierta para y = 0.


a,
Suponga ahora que 0 < x 1 y 0 < y 1. Sea u 0 y v 0 tales que
x = eu y y = ev , entonces la desigualdad se convierte en
1
1

+
1 + e2v
1 + e2u
esto es,

f (u) + f (v)
f
2

2
1 + e(u+v)

u+v
2

2x

1
12e
donde f (x) = 1+e2x . Como f (x) = e4x (1+e2x )5/2 , la funcin es cncava en
o
o
el intervalo [0, ). Por lo tanto, la desigualdad anterior es verdadera.

Solucin 1.80 Encuentre f (x).


o
Solucin 1.81 Utilice log(sen x) o bien
o
sen A sen B = sen

A+B AB
+
2
2

sen

A+B AB

2
2

Solucin 1.82 (i) Si 1+nx 0, la desigualdad es evidente ya que (1+x)n 0.


o
Suponga que (1+nx) > 0. Aplique M GM A a los nmeros (1, 1, . . . , 1, 1+nx)
u
con (n 1) unos.
(ii) Sean a1 , . . ., an nmeros positivos y dena, para cada j = 1, . . . n, j =
u
a1 +...+aj
. Aplique la desigualdad
j
j
j j1 (j 1), lo que implica
j
j
j j1 j

j
(j 1)
j1

de Bernoulli para mostrar que

j
j1

j1
j1
= j1 (jj (j 1)j1 ) = aj j1 .

148

Soluciones a los Ejercicios y Problemas

n1
n2
n
Luego, n an n1 an an1 n2 an an1 a1 .

Solucin 1.83 Si x y z, se tiene que xn (x y)(x z) y n (x y)(y z)


o
y z n (z x)(z y) 0.
Solucin 1.84 Note que x(xz)2 +y(y z)2 (xz)(y z)(x+y z) 0 si y
o
slo si x(xz)(xy)+y(yz)(yx)+z(xz)(yz) 0. La desigualdad ahora
o
se sigue de la desigualdad de Schr. O bien, la ultima expresin es simtrica en
u

o
e
x, y y z, por lo que se puede suponer x z y, ahora regrese a la desigualdad
original, donde claramente
x(x z)2 + y(y z)2 0 (x z)(y z)(x + y z).
Solucin 1.85 La desigualdad es homognea, por lo que se puede suponer que
o
e
x
a + b + c = 1. Ahora los sumandos de la izquierda son de la forma (1x)2 ,
x
4+2x
es convexa, ya que f (x) = (1x)4
(1x)2
b
c
a
Jensen, (1a)2 + (1b)2 + (1c)2 3f a+b+c = 3f
3

y la funcin f (x) =
o
desigualdad de

> 0. Por la
1
3

3 2
2 .

Solucin 1.86 Como (a + b + c)2 3(ab + bc + ca), se puede deducir que


o
3
9
1 + ab+bc+ca 1 + (a+b+c)2 . Entonces, la desigualdad ser verdadera si
a
1+

6
9

.
(a + b + c)2
(a + b + c)

Pero esta ultima desigualdad se sigue de que 1

3
a+b+c
z = 1;
c

0.

1
se sigue inmediataAhora bien, si abc = 1, considere x = a , y = 1 y
b
mente que xyz = 1. Luego, la desigualdad es equivalente a

1+

3
6

xy + yz + zx
x+y+z

que es la primera parte del ejercicio.


Solucin 1.87 Utilice la convexidad de la funcin f (x) = xr , para r 1
o
o
(su segunda derivada es r(r 1)xr2 ). Primero suponga que r > s > 0. La
r
desigualdad de Jensen para funciones convexas f (x) = x s aplicada a xs , . . . ,
1
xs dice que
n
r
w1 xr + + wn xr (w1 xs + + wn xs ) s
1
n
1
n

y tomando la 1 -sima potencia de ambos lados de la desigualdad se obtiene la


r e
desigualdad deseada.

4.1 Soluciones a los ejercicios del cap


tulo 1

149
r

Ahora, suponga que 0 > r > s. Entonces f (x) = x s es cncava, es decir,


o
la desigualdad de Jensen es vlida en el otro sentido. Sin embargo, si toma la
a
potencia 1 -sima la desigualdad se invierte nuevamente.
r e
r
Finalmente, en el caso r > 0 > s, f (x) = x s es nuevamente convexa, y
e
tomando la 1 -sima potencia en ambos lados de la desigualdad, sta se preserva.
r e
Solucin 1.88 (i) Aplique la desigualdad de Hlder a los nmeros xc , . . . , xc ,
o
o
u
n
1
c , . . . , y c con a = a and b = b .
y1
n
c
c
(ii) Para probar esto siga la demostracin del ejemplo 1.5.9. Lo unico que tiene
o

1 b
1 c
1 a
que probar adems es que xi yi zi a xi + b yi + c zi , pero esto se sigue de la
a
parte (i) del mismo ejemplo.
Solucin 1.89 Por la simetr de las variable en la desigualdad se puede suponer
o
a
que a b c. Se tienen dos casos, (i) b a+b+c y (ii) b a+b+c .
3
3
Caso (i) b a+b+c .
3
Se tiene que a+b+c a+c c, y luego es cierto que a+b+c b+c c.
3
2
3
2
Entonces, existen , [0, 1] tales que
c+a
= c + (1 )
2

a+b+c
3

and

b+c
= c + (1 )
2

a+b+c
3

Sumando estas igualdades, se obtiene que


a + b + 2c
= (+)c+(2)
2

a+b+c
3

= (2)

Luego,
a + b 2c
= (2 )
2

a + b 2c
3

1
Por lo tanto, 2 ( + ) = 3 y ( + ) = 2 .
2
Ahora bien, como f es una funcin convexa, se tiene que
o

f
f
f

a+b
2
b+c
2
c+a
2

1
(f (a) + f (b))
2

f (c) + (1 )f
f (c) + (1 )f

a+b+c
3
a+b+c
3

a + b 2c
+2c.
3

150

Soluciones a los Ejercicios y Problemas

entonces, sumando estas desigualdades se obtiene que


f

a+b
2

+f

b+c
2

+f

c+a
2

1
(f (a) + f (b) + f (c))
2
3
a+b+c
+ f
.
2
3

Caso (ii) b a+b+c .


3
Es anlogo al caso (i), utilizando el hecho que a
a
a+b+c
3 .

a+c
2

a+b+c
3

ya

a+b
2

Solucin 1.90 Si alguna de las variables a, b o c es cero, la desigualdad


o
es evidente. Aplicando la desigualdad de Popovicius, al ejercicio anterior, y
utilizando la funcin f : R R+ denida por f (x) = exp(2x), que es convexa
o
ya que f (x) = 4 exp(2x) > 0, se obtiene que
2(x + y + z)
)
3
2 [exp(x + y) + exp(y + z) + exp(z + x)] =

exp(2x) + exp(2y)+ exp(2z) + 3 exp(

=2 [exp(x) exp(y) + exp(y) exp(z) + exp(z) exp(x)] .


Deniendo a = exp(x), b = exp(y), c = exp(z), se puede reescribir la desigualdad anterior como

3
a2 + b2 + c2 + 3 a2 b2 c2 2(ab + bc + ca).
Para la segunda parte aplique la desigualdad AM GM de la siguiente forma

3
2abc + 1 = abc + abc + 1 3 a2 b2 c2 .
Solucin 1.91 Aplicando la desigualdad de Popoviciu a la funcin convexa
o
o
1
9
4
4
4
1
f (x) = x + x se obtiene la desigualdad, a + 1 + 1 + a+b+c b+c + c+a + a+b .
b
c
Ahora, multiplique ambos lados de la desigualdad por (a + b + c) para terminar
la demostracin.
o
Solucin 1.92 Observe que por la ecuacin (1.8), se obtiene
o
o
1
1
1
x2 + y 2 + z 2 |x||y| |y||z| |z||x| = (|x| |y|)2 + (|y| |z|)2 + (|z| |x|)2
2
2
2
lo cual es claramente mayor o igual a cero. Por lo tanto,
|xy + yz + zx| |x||y| + |y||z| + |z||x| x2 + y 2 + z 2 .

4.1 Soluciones a los ejercicios del cap


tulo 1

151

Segunda Solucin. Aplique la desigualdad de Cauchy-Schwarz con (x, y, z) y


o
(y, z, x).
Solucin 1.93 La desigualdad es equivalente a tener ab+bc+ca a2 +b2 +c2 ,
o
la cual se sabe que es verdadera. Vea el ejercicio 1.27.
Solucin 1.94 Observe que, si a + b + c = 0, entonces se sigue de la ecuacin
o
o
3 + b3 + c3 = 3abc. Como (x y) + (y z) + (z x) = 0, se obtiene
(1.7) que a
la factorizacin
o
(x y)3 + (y z)3 + (z x)3 = 3(x y)(y z)(z x).
Solucin 1.95 Suponga, sin prdida de generalidad, que a b c. Entonces,
o
e
necesita demostrar que
a3 + b3 + c3 + 3abc 0.
Como
a3 + b3 + c3 + 3abc = (a)3 + b3 + c3 3(a)bc,
factorice, la ultima expresin, como

o
1
(a + b + c)((a + b)2 + (a + c)2 + (b c)2 ).
2
La conclusin se sigue de la desigualdad del tringulo, b + c > a.
o
a
Solucin 1.96 Sea p = |(x y)(y z)(z x)|. Utilizando la desigualdad
o
M G M A del lado derecho de la identidad (1.8), se tiene que
x2 + y 2 + z 2 xy yz zx

3
2

p2 .

(4.1)

Ahora bien, como |x y| x + y, |y z| y + z, |z x| z + x, se tiene que


2(x + y + z) |x y| + |y z| + |z x|.
Aplicando nuevamente la desigualdad M G M A, se obtiene que

2(x + y + z) 3 3 p,

y el resultado se sigue de las desigualdades (4.1) y (4.2).

(4.2)

152

Soluciones a los Ejercicios y Problemas

Solucin 1.97 Utilice la identidad (1.7), factorice, la condicin x3 + y 3 + z 3


o
o
3xyz = 1, y se obtiene
(x + y + z)(x2 + y 2 + z 2 xy yz zx) = 1.

(4.3)

Sean A = x2 + y 2 + z 2 y B = x+ y + z. Observe que B 2 A = 2(xy + yz + zx).


Por la identidad (1.8), se tiene que B > 0. La ecuacin 4.3 se transforma en
o
B A
luego 3A = B 2 +
obtener

2
B.

B2 A
2

= 1,

Como B > 0, aplique la desigualdad M G M A, para

1
1
2
= B2 + +
3,
B
B B
es decir, A 1. El m
nimo A = 1 se alcanza, por ejemplo, con (x, y, z) =
(1, 0, 0).
3A = B 2 +

Solucin 1.98 Por la desigualdad (1.11), se tiene que


o
1 1 4 16
(1 + 1 + 2 + 4)2
64
+ + +

=
.
a b
c
d
a+b+c+d
a+b+c+d
Solucin 1.99 Utilice la desigualdad (1.11) dos veces para obtener
o
2

( (a+b) )2
(a2 + b2 )2
(a + b)4
a4 b4
2
+

=
.
a +b =
1
1
2
2
8
4

Solucin 1.100 Escriba el lado izquierdo como


o

( 2)2 ( 2)2 ( 2)2


+
+
x+y
y+z
z+x
y use la desigualdad (1.11).
Solucin 1.101 Escriba el lado izquierdo como
o
x2
y2
z2
+
+
axy + bzx ayz + bxy azx + byz
y ahora aplicando la desigualdad (1.11), se tiene
y2
z2
(x + y + z)2
3
x2
+
+

,
axy + bzx ayz + bxy azx + byz
(a + b)(xy + zx + yz)
a+b

4.1 Soluciones a los ejercicios del cap


tulo 1

153

donde la ultima desigualdad se sigue de la ecuacin (1.8).

o
Solucin 1.102 Escriba el lado izquierdo como
o
b2
c2
b2
c2
a2
a2
+
+
+
+
+
,
a+b b+c c+a a+b b+c c+a
luego use la desigualdad (1.11).
Solucin 1.103 (i) Escriba el lado izquierdo como
o
x2
y2
z2
+ 2
+ 2
x2 + 2xy + 3zx y + 2yz + 3xy z + 2zx + 3yz
y use la desigualdad (1.11) para obtener
x
y
z
(x + y + z)2
+
+
2
.
x + 2y + 3z y + 2z + 3x z + 2x + 3y
x + y 2 + z 2 + 5(xy + zx + yz)
Ahora, slo falta probar la desigualdad
o
1
(x + y + z)2
,
x2 + y 2 + z 2 + 5(xy + zx + yz)
2
pero sta es equivalente a x2 + y 2 + z 2 xy + zx + yz.
e
(ii) Como en la parte (i), escriba el lado izquierdo como

w2
x2
y2
z2
+
+
+
xw + 2yw + 3zw xy + 2xz + 3xw yz + 2yw + 3xy zw + 2xz + 3yz
entonces, use la desigualdad (1.11) para obtener
x
y
z
w
+
+
+

x + 2y + 3z y + 2z + 3w z + 2w + 3x w + 2x + 3y
(w + x + y + z)2

.
4(wx + xy + yz + zw + wy + xz)
Luego, la desigualdad que tiene que probar es
(w + x + y + z)2
2
,
4(wx + xy + yz + zw + wy + xz)
3
que es equivalente a 3(w2 + x2 + y 2 + z 2 ) 2(wx + xy + yz + zw + wy + xz).
Pero sta se deduce de la desigualdad M G M A aplicada seis veces en la
e
forma x2 + y 2 2xy.

154

Soluciones a los Ejercicios y Problemas

Solucin 1.104 Use la desigualdad (1.11) para obtener


o
y2
z2
x2
+
+

(x + y)(x + z) (y + z)(y + x) (z + x)(z + y)

(x + y + z)2
.
x2 + y 2 + z 2 + 3(xy + yz + zx)

Tambin, la desigualdad
e
3
(x + y + z)2

2 + y 2 + z 2 + 3(xy + yz + zx)
x
4
es equivalente a
x2 + y 2 + z 2 xy + yz + zx.
Solucin 1.105 Escriba el lado izquierdo como
o
a2
b2
c2
d2
+
+
+
a(b + c) b(c + d) c(d + a) d(a + b)
y use la desigualdad (1.11) para obtener
a2
b2
c2
d2
(a + b + c + d)2
+
+
+

a(b + c) b(c + d) c(d + a) d(a + b)


a(b + 2c + d) + b(c + d) + d(b + c)
Observe, por otro lado, que
(a + b + c + d)2
=
(ac + bd) + (ab + ac + ad + bc + bd + cd)
=

a2 + b2 + c2 + d2 + 2ab + 2ac + 2ad + 2bc + 2bd + 2cd


(ac + bd) + (ab + ac + ad + bc + bd + cd)

Demostrar que la ultima expresin es mayor que 2, es equivalente a demostrar

o
2 + c2 2ac y b2 + d2 2bd, las cuales son inmediatas por la desigualdad
que a
M G M A.
Solucin 1.106 Escriba el lado izquierdo como
o
a2
b2
c2
d2
e2
+
+
+
+
ab + ac bc + bd cd + ce de + ad ae + be
y use la desigualdad (1.11) para obtener
a2
b2
c2
d2
e2
(a + b + c + d + e)2
+
+
+
+

.
ab + ac bc + bd cd + ce de + ad ae + be
ab

4.1 Soluciones a los ejercicios del cap


tulo 1

155

Como
(a + b + c + d + e)2 =

a2 + 2

ab,

muestre que
a2 + 4

ab 5

ab,

lo cual es equivalente a
2
La ultima desigualdad se sigue de

a2

ab.

a2

ab.

Solucin 1.107 (i) Use la desigualdad de Tchebyshev con los nmeros (a


o
u
b2
c2
a2
b c) y ( x y z ) para tener que
1
3

a3 b3 c3
+
+
x
y
z

a2
x

b2
y

c2
z

a+b+c
,
3

entonces, por la desigualdad (1.11), se tiene que


(a + b + c)2
a2 b2 c2
+
+

.
x
y
z
x+y+z
Por lo tanto,

(a + b + c)2 a + b + c
a3 b3 c3
+
+

.
x
y
z
x+y+z
3

(ii) Por el ejercicio 1.88, se tiene que


a3 b3 c3
+
+
x
y
z

1
3

(1 + 1 + 1) 3 (x + y + z) 3 a + b + c.

Elevando al cubo ambos lados de la desigualdad y dividiendo ambos lados por


3(x + y + z) se obtiene el resultado.
Solucin 1.108 Use la desigualdad util (1.11) y obtendr que
o

a
x2 + + x 2
x2
x2
n
n
1
1
=
+ +
x1 + + x n x1 + + x n
x1 + + xn

x1 + + x n
(x1 + + xn )2
=
.
n(x1 + + xn )
n

Luego, es suciente probar que


x1 + + xn
n

kn
t

x1 x n .

156

Soluciones a los Ejercicios y Problemas

Como k = mx {x1 , . . . , xn } min {x1 , . . . , xn } = t, se tiene que kn n


a
t
y como x1 ++xn 1, ya que todos los xi son enteros positivos, es suciente
n
probar que
x1 + + x n n
x1 x n ,
n
que es equivalente a la desigualdad M G M A.
Como todas las desigualdades intermedias que se han usado son desigualdades
vlidas cuando x1 = = xn se concluye que esto sucede en este caso.
a
Solucin 1.109 Con la sustitucin a = x , b =
o
o
y
la desigualdad como,

y
z

z
y c = x , se puede reescribir

b3
c3
a3
+ 3
+ 3
1,
a3 + 2 b + 2 c + 2
con la condicin extra que, abc = 1.
o
Para probar esta ultima desigualdad se puede usar la condicin extra como sigue

o
a3
b3
c3
+ 3
+ 3
+2 b +2 c +2

a3

=
=

a3
b3
c3
+ 3
+ 3
+ 2abc b + 2abc c + 2abc
a2
b2
c2
+ 2
+ 2
a2 + 2bc b + 2ca c + 2ab
(a + b + c)2
= 1.
a2 + b2 + c2 + 2bc + 2ca + 2ab
a3

Y la desigualdad anterior se deduce de la desigualdad (1.11).


c
Solucin 1.110 Con la sustitucin x = a , y = b , z = a , la desigualdad toma
o
o
b
c
la forma
b
c
3
a
+
+
.
b+c c+a a+b
2

Y esta ultima desigualdad es la desigualdad de Nesbitt, ejemplo 1.4.8.

a1
a2
Solucin 1.111 Use la sustitucin x1 = a1 , x2 = a3 , . . ., xn = an . Como
o
o
a2
a1
1
1
a
1+x1 +x1 x2 = 1+ a2 + a2 a3 = a1 +a2 +a3 y anlogamente para los otros sumandos
a1

a1 a2

del lado izquierdo de la desigualdad. Por lo que se tiene que la desigualdad es


ahora equivalente a
a2
an
a1
+
+ +
> 1.
a1 + a2 + a3 a2 + a3 + a4
an + a1 + a2

4.1 Soluciones a los ejercicios del cap


tulo 1

157

Pero esta desigualdad es ahora evidente. Unicamente observe que para toda
i = 1, ..., n se cumple que
ai + ai+1 + ai+2 < a1 + a2 + + an .
1
o
Solucin 1.112 Con la sustitucin x = a , y = 1 y z = 1 , la condicin
o
o
b
c
ab + bc + ca = abc, se transforma en x + y + z = 1 y la desigualdad original es
equivalente a

x4 + y 4 y 4 + z 4 z 4 + x4
+
+
1 = x + y + z.
x3 + y 3 y 3 + z 3 z 3 + x3
Por la desigualdad de Tchebyshev, se garantiza que
x4 + y 4
x3 + y 3 x + y

,
2
2
2
por lo que,
x4 + y 4 y 4 + z 4 z 4 + x4
x+y y+z z+x
+
+
.
+ 3
+ 3

3 + y3
3
3
x
y +z
z +x
2
2
2

Solucin 1.113 La desigualdad de la izquierda se sigue de aplicar la desigualdad


o
(1.11). Para la desigualdad de la derecha, la sustitucin, x = bc , y = ca , z = ab
o
a
b
c
nos permite reescribir la desigualdad como
x+y+z
yz + zx + xy

.
3
3
Elevando al cuadrado ambos lados y se obtiene que 3(xy + yz + zx) (x +
y + z)2 , la cual es vlida si y slo si (xy + yz + zx) x2 + y 2 + z 2 , pero esta
a
o
desigualdad es ya conocida.
Solucin 1.114 Note que
o
1
1
1
+
+
a+1 b+1 c+1
1
1
1
+
+
.
1
a+1 b+1 c+1

a2 b2 c2
+
+
0 33
a+1 b+1 c+1

158

Soluciones a los Ejercicios y Problemas

Utilizando la sustitucin, a =
o

2x
y ,

1
1
1
+
+
=
a+1 b+1 c+1

b=

2y
z ,

c=

2z
x,

se obtiene

1
1
1
+ 2y
+ 2z
+1
x +1
z +1
z
x
y
+
+
=
2x + y 2y + z 2z + x
z2
x2
y2
+
+
=
2
2
2xy + y
2yz + z
2zx + x2
(x + y + z)2

= 1.
2xy + y 2 + 2yz + z 2 + 2zx + x2
2x
y

Para la unica desigualdad que aparece en este clculo, se aplic la desigualdad

a
o
(1.11).
Solucin 1.115 Observe que
o
2
2
[5, 0, 0] = (a5 + b5 + c5 ) (a3 bc + b3 ca + c3 ab) = [3, 1, 1],
6
6
donde se us el teorema de Muirhead.
o
Solucin 1.116 Utilizando la frmula de Hern para el rea de un tringulo,
o
o
o
a
a
podemos reescribir la desigualdad como sigue

a2 + b2 + c2 4 3

(a + b + c) (a + b c) (a + c b) (b + c a)
.
2
2
2
2

Pero, sta es equivalente a


e
(a2 + b2 + c2 )2 3[((a + b)2 c2 )(c2 (b a)2 )]

= 3(2c2 a2 + 2c2 b2 + 2a2 b2 (a4 + b4 + c4 )),

es decir, a4 + b4 + c4 a2 b2 + b2 c2 + c2 a2 , que en trminos del teorema de


e
Muirhead es equivalente a probar que [4, 0, 0] [2, 2, 0].
Segunda Solucin. Utilizando la sustitucin
o
o
x = a + b c, y = a b + c, z = a + b + c,
obtenemos que x + y + z = a + b + c; entonces, utilizando la frmula de Hern,
o
o
tenemos
4(ABC) =

(a + b + c)(xyz)

(a + b + c)

(a + b + c)2
(x + y + z)3

.
=
27
3 3

4.1 Soluciones a los ejercicios del cap


tulo 1

159

Ahora, slo se tiene que probar que (a + b + c)2 3(a2 + b2 + c2 ). Aplicando,


o
el teorema de Muirhead, se deduce esta desigualdad, ya que [1, 1, 0] [2, 0, 0].
Solucin 1.117 Note que,
o
b
c
9
a
+
+

(a + b)(a + c) (b + c)(b + a) (c + a)(c + b)


4(a + b + c)
8(ab + bc + ca)(a + b + c) 9(a + b)(b + c)(c + a)
24abc + 8

(a2 b + ab2 ) 9

(a2 b + ab2 ) + 18abc

6abc a2 b + ab2 + b2 c + bc2 + c2 a + ca2


[1, 1, 1] [2, 1, 0]
Solucin 1.118 La desigualdad es equivalente a
o
a3 + b3 + c3 ab(a + b c) + bc(b + c a) + ca(c + a b).
Dena x = a + b c, y = b + c a, z = a + c b, para obtener a =
b = x+y , c = y+z . Entonces, la desigualdad que se tiene que probar es
2
2

x+z
2 ,

1
1
((x+y)3 +(y+z)3 +(z+x)3 ) ((x+y)(x+z)x+(x+y)(y+z)y+(x+z)(y+z)z),
8
4
que es equivalente a
3(x2 y+y 2 x+y 2 z+z 2 y+z 2 x+x2 z) 2(x2 y+y 2 x+y 2 z+z 2 y+z 2 x+x2 z)+6xyz
o
x2 y + y 2 x + y 2 z + z 2 y + z 2 x + x2 z 6xyz
y aplicando el teorema de Muirhead se obtiene el resultado cuando x, y, z son
no-negativos. Si uno de ellos es negativo (y no puede ser ms de uno al mismo
a
tiempo), se tiene que
x2 (y + z) + y 2 (x + z) + z 2 (x + y) = x2 2c + y 2 2a + z 2 2b 0
pero 6xyz es negativo, lo cual concluye la prueba.
Solucin 1.119 Observe que
o
b3
c3
a3
+ 2
+ 2
a+b+c
b2 bc + c2 c ca + a2 a ab + b2

160

Soluciones a los Ejercicios y Problemas

que es equivalente a la desigualdad


a3 (b + c) b3 (c + a) c3 (a + b)
+ 3
+ 3
a + b + c,
b3 + c3
c + a3
a + b3
que a su vez es equivalente a
a3 (b+c)(a3 +c3 )(a3 +b3 )+b3 (c+a)(b3 +c3 )(a3 +b3 )+c3 (a+b)(a3 +c3 )(b3 +c3 )
(a + b + c)(a3 + b3 )(b3 + c3 )(c3 + a3 ).
Se puede reescribir esta ultima desigualdad en trminos del teorema de Muirhead

e
como
[9, 1, 0] + [6, 4, 0] + [6, 3, 1] + [4, 3, 3]

1
1
[1, 0, 0]
[6, 3, 0] + [3, 3, 3]
2
3
[7, 3, 0] + [6, 4, 0] + [6, 3, 1] + [4, 3, 3]

[9, 1, 0] [7, 3, 0]
lo cual es evidente utilizando Muirhead.
Solucin 1.120 Suponga que a b c, luego
o
1
1
1

.
(1 + b) (1 + c)
(1 + c) (1 + a)
(1 + a) (1 + b)
Use Tchebyshev para ver que
b3
c3
a3
+
+

(1 + b) (1 + c) (1 + a) (1 + c) (1 + a) (1 + b)
1
1
1
1
(a3 + b3 + c3 )
+
+
3
(1 + b)(1 + c) (1 + a)(1 + c) (1 + a)(1 + b)
1
3 + (a + b + c)
= (a3 + b3 + c3 )
.
3
(1 + a)(1 + b)(1 + c)
1
Finalmente, use que 3 (a3 + b3 + c3 ) ( a+b+c )3 ,
3

c)

3+a+b+c 3
3

a+b+c
3

para ver que

1 3
3 + (a + b + c)
(a + b3 + c3 )

3
(1 + a)(1 + b)(1 + c)
Para la ultima desigualdad, observe que

a+b+c
3
a+b+c
3
+ a+b+c
3

1 y (1 + a)(1 + b)(1 +
3

1
2.

6
(1 +

a+b+c 3
3 )

6
.
8

4.2 Soluciones a los ejercicios del cap


tulo 2

161

Segunda Solucin. Multiplicando por el denominador comn y desarrollando


o
u
ambos lados, la desigualdad deseada es equivalente a
a(a4 + b4 + c4 + a3 + b3 + c3 ) 3(1 + a + b + c + ab + bc + ca + abc).
Como 4(a4 + b4 + c4 + a3 + b3 + c3 ) = 4(3[4, 0, 0] + 3[3, 0, 0]) y 3(1 + a + b + c +
ab + bc + ca + abc) = 3([0, 0, 0] + 3[1, 0, 0] + 3[1, 1, 0] + [1, 1, 1]), la desigualdad
es equivalente a
4[4, 0, 0] + 4[3, 0, 0] [0, 0, 0] + 3[1, 0, 0] + 3[1, 1, 0] + [1, 1, 1].
Ahora, observemos que
[4, 0, 0]

4 4 4
4 4 4
, ,
= a 3 b 3 c 3 = 1 = [0, 0, 0],
3 3 3

donde tenemos que abc = 1. Tambin


e
1
1
3[4, 0, 0] 3[2, 1, 1] = 3 (a2 bc + b2 ca + c2 ab) = 3 (a + b + c) = 3[1, 0, 0]
3
3
y
3[3, 0, 0] 3

4 4 1
, ,
3 3 3

4 4 1
4 4 1
1 4 4 1
= 3 (a 3 b 3 c 3 + b 3 c 3 a 3 + c 3 a 3 b 3
3
1
= 3 (ab + bc + ca) = 3[1, 0, 0].
3

Finalmente, [3, 0, 0] [1, 1, 1]. Sumando estas desigualdades, obtenemos la


desigualdad deseada.

4.2.

Soluciones a los ejercicios del cap


tulo 2

Solucin 2.1 (i) Trace un segmento BC de longitud a, con centro en B un


o
c
rculo de radio c y con centro en C un c
rculo de radio b, Bajo qu circunse
tancias se intersectan?
(ii) Se sigue de (i).
(iii) a = x + y, b = y + z, c = z + x x = a+cb , y = a+bc , z = b+ca .
2
2
2


Solucin 2.2 (i) c < a+b c < a+b+2 ab = ( a+ b)2 c < a+ b.
o
(ii) Con 2, 3 y 4 se puede construir un tringulo y con 4, 9 y 16 no es posible
a
construirlo.

162

Soluciones a los Ejercicios y Problemas

1
1
1
(iii) a < b < c a + b < a + c < b + c b+c < c+a < a+b , por lo
1
1
1
que ser suciente que ver que a+b < b+c + c+a , pero es ms fcil ver que
a
a a
1
1
1
c < b+c + c+a .

Solucin 2.3 Utilice el hecho de que si a, b, c son las longitudes de los lados de
o
un tringulo, el ngulo que se opone al lado de longitud c es o bien recto, agudo
a
a
u obtuso dependiendo si c2 es igual, menor o mayor a a2 + b2 , respectivamente.
Suponga entonces que a b c d e, y que las ternas de segmentos
de longitud (a, b, c) y (c, d, e) no forman un tringulo acutngulo. Luego, como
a
a
c2 a2 + b2 y e2 c2 + d2 , se puede concluir que e2 a2 + b2 + d2
a2 + b2 + c2 a2 + b2 + a2 + b2 = (a + b)2 + (a b)2 (a + b)2 , por lo cual
a + b e, lo que nos lleva a una contradiccin.
o
Solucin 2.4 Como A > B entonces BC > CA. Utilizando la desigualdad
o
del tringulo AB < BC + CA y la armacin anterior se tiene que AB < 2BC.
a
o
Solucin 2.5 (i) Sea O el punto de interseccin de las diagonales AC y BD.
o
o
Aplique la desigualdad del tringulo a los tringulos ABO y CDO. Sumando
a
a
las desigualdades, tenemos AB + CD < AC + BD. Por otro lado, por hiptesis
o
se tiene que AB + BD < AC + CD. Sumando estas dos ultimas desigualdades

obtiene que AB < AC.


(ii) Sea DE paralela a BC; entonces EDA < BCD < A, luego DE >
1
1
2 AD. Por lo tanto, 2 AD < DE < BC, vea el ejercicio anterior.
Solucin 2.6 Cada di es menor que la suma de la longitud de dos lados. Use
o
tambin el hecho que, en un cuadriltero convexo la suma de la longitud de dos
e
a
lados opuestos es menor que la suma de la longitud de las diagonales.
Solucin 2.7 Aplique la desigualdad del tringulo a los tringulos ABA y
o
a
a
1
AA C para mostrar que c < ma + 1 a y b < ma + 2 a.
2
Solucin 2.8 Si , , son los ngulos del tringulo en A, B y C, respectivao
a
a
mente, y si 1 = BAA y 2 = A AC entonces, por D2, > 1 y > 2 .
Luego, 180o = + + > 1 + 2 + = 2. O bien, si trazamos un c
rculo
de dimetro BC, A deber estar fuera del c
a
a
rculo y entonces BAC < 90 .
Solucin 2.9 Construya un paralelogramo ABDC, con una diagonal BC y
o
otra AD la cual es igual al doble de AA y use la desigualdad D2, en el tringulo
a
ABD.
Solucin 2.10 Complete un paralelogramo como en la solucin anterior para
o
o
c+a
a+b
b+c
a
mostrar que ma < 2 . Anlogamente, mb < 2 y mc < 2 . Para probar la

4.2 Soluciones a los ejercicios del cap


tulo 2

163

desigualdad de la izquierda, sean A , B , C los puntos medios de los lados BC,


CA y AB, respectivamente.
A
B

Prolongue el segmento C B hasta un punto A de manera que C A = BC.


Aplique el resultado anterior al tringulo AA A cuyos lados tienen longitud
a
ma , mb y mc .
Solucin 2.11 Considere el cuadriltero ABCD y sea O un punto en el exterior
o
a
del cuadriltero tal que AOB sea semejante a ACD, entonces tambin OAC
a
e
y BAD son semejantes. Si O, B y C son colineales se tiene la igualdad, si no
son colineales se tiene una desigualdad2 .
Solucin 2.12 Sean a = AB, b = BC, c = CD, d = DA, m = AC y
o
n = BD. Sea R el radio del circunc
rculo de ABCD. Se tiene que3
m(ab + cd)
,
4R
n(bc + ad)
(ABCD) = (BCD) + (DAB) =
,
4R
(ABCD) = (ABC) + (CDA) =

donde (ABCD) denota el rea del cuadriltero ABCD. Por lo tanto,


a
a
bc + ad
m
=
>1
n
ab + cd

bc + ad > ab + cd

(d b)(a c) > 0.

Solucin 2.13 Con centro en A haga una rotacin de 60 del tringulo ABP.
o
o
a
El punto B se transforma en C y sea P el transformado de P. El tringulo
a
P P C tiene lados P P = P A, P C = P B y P C, como el que se desea.
2
3

Ver [6], pg. 136 o [1], pg. 128.


a
a
Ver [6], pg. 97 o [9], pg. 13.
a
a

164

Soluciones a los Ejercicios y Problemas


A

A
P

P
B

Segunda Solucin. Use la desigualdad de Ptolomeo (ejercicio 2.11) en los


o
cuadrilteros ABCP , ABP C y AP BC; despus de cancelar factores comua
e
nes tendr que: P B < P C + P A, P A < P C + P B y P C < P A + P B,
a
respectivamente. Esto garantiza la existencia del tringulo.
a
Tercera Solucin. Otra solucin para el caso en que P este dentro de ABC.
o
o
Sea P el punto donde AP corta al lado BC. Use ahora que, AP < AP <
AB = BC < P B + P C. Anlogamente obtenga las desigualdades P B <
a
P C + P A y P C < P A + P B.

Solucin 2.14 Sean a = AB, b = BC, x = AC, y = BD. Recuerde que


o
en un paralelogramo se cumple la identidad 2(a2 + b2 ) = x2 + y 2 . Suponga
adems, sin perder generalidad, que a b. Es claro que 2b < (x + y), por lo
a
que (2b)2 < (x + y)2 = x2 + y 2 + 2xy = 2(a2 + b2 ) + 2xy. Luego, reduciendo
se obtiene que 2(b2 a2 ) < 2xy.
Solucin 2.15 (i) Prolongue las medianas AA , BB y CC hasta que corten
o
al circunc
rculo en A1 , B1 y C1 , respectivamente. Use la potencia de A para
a2
e
establecer que A A1 = 4ma . Tambin, use el hecho de que ma + A A1 2R y
2

que la longitud de la mediana satisface m2 = 2(b +c )a , es decir, 4m2 + a2 =


a
a
4
2(b2 + c2 ). Hay expresiones anlogas para mb y mc .
a
(ii) Use la desigualdad de Ptolomeo en los cuadrilteros AC GB , BA GC y
a
GA , donde G denota el centroide. Por ejemplo, para el primer cuadriltero
CB
a
2
b
c
se obtiene que 3 ma a 2 mc + 2 mb , entonces 2ma a2 ab mc + ac mb .
2
3
3
Solucin 2.16 Use la frmula 4m2 + b2 = 2(c2 + a2 ), para ver que m2
o
o
b
b
3
a
m2 = 4 (c2 b2 ). Ahora bien, usando la desigualdad del tringulo pruebe que
c
mb + mc < 3 (b + c). De aqu puede deducir la desigualdad de la izquierda.

4.2 Soluciones a los ejercicios del cap


tulo 2

165

La desigualdad de la derecha se puede deducir de la primera cuando se aplica


al tringulo de lados4 de longitud ma , mb y mc .
a
Solucin 2.17 Sean a, b y c la longitud de los lados del tringulo ABC. Si E
o
a
y F son las proyecciones de Ia sobre las rectas AB y CA, respectivamente, es
claro que, si ra es el radio del exc
rculo, se tiene que ra = Ia E = EA = AF =
F Ia = s, donde s es el semiper
metro de ABC. Adems, si ha es la altura del
a
AD
a
tringulo ABC desde A, entonces DIa = ha . Como aha = bc, se tiene que
a
r
AD
ha
bc
=
=
=
DIa
ra
as

abc
4R

4Rr
a2

1
rs

4Rr
,
a2

donde r y R son el inradio y circunradio de ABC, respectivamente.


AD
a
Como 2R = a y 2r = b + c a, entonces DIa = b+ca = b+c 1. Bastar que
a
a

b+c
2 , pero bc =
2 c2
b
vea que a 2 o, equivalentemente, que 2bc a
b2 +c2
a2
= 2.
2
Solucin 2.18 Simplicando y utilizando el ejercicio 1.27, la primera desigualo
dad es equivalente a ab + bc + ca a2 + b2 + c2 . Para la segunda desarrolle
(a+b+c)2 y utilizando la desigualdad del tringulo se obtiene que a2 < a(b+c).
a
Solucin 2.19 Utilice la sugerencia anterior.
o
Solucin 2.20 Si desarrolla la expresin regresar al ejercicio anterior.
o
o
a
Solucin 2.21 La primera desigualdad es la desigualdad de Nesbitt, ejemplo
o
1.4.8. Para la segunda desigualdad use el hecho de que a + b > a+b+c , entonces
2
2c
c
< a+b+c .
a+b
Solucin 2.22 Observe que a2 (b + c a) + b2 (c + a b) + c2 (a + b c)
o
2abc = (b + c a) (c + a b) (a + b c); ahora vea el ejemplo 2.2.3.
Solucin 2.23 Observe que
o
a b2 + c2 a2 +b c2 + a2 b2 + c a2 + b2 c2 =

a2 (b + c a) + b2 (c + a b) + c2 (a + b c) ,

vea ahora el ejercicio 2.22.


4

Ver la solucin del ejercicio 2.10.


o

166

Soluciones a los Ejercicios y Problemas

Solucin 2.24 Use la transformacin de Ravi con a = y + z, b = z + x,


o
o
c = x + y para ver primero que
a2 b(ab)+b2 c(bc)+c2 a(ca) = 2(xy 3 +yz 3 +zx3 )2(xy 2 z+x2 yz+xyz 2 ).
Luego, la desigualdad es equivalente a
desigualdad (1.11).

x2
y

y2
z

+ zx x + y + z. Ahora, use la

Solucin 2.25
o
ab bc ca
+
+
a+b b+c c+a

=
<

ab bc ca

a+b b+c c+a


cab
1
,
(a + b)(b + c)(c + a)
8

para la ultima desigualdad, vea la solucin del ejemplo 2.2.3.

o
Solucin 2.26 Por el ejercicio 2.18
o
3(ab + bc + ca) (a + b + c)2 4(ab + bc + ca).
Entonces, como ab + bc + ca = 3, se sigue que 9 (a + b + c)2 12, de donde
se tiene el resultado.
Solucin 2.27 Use la transformacin de Ravi, a = y + z, b = z + x, c = x + y.
o
o
Por la desigualdad M G M A, y la desigualdad de Cauchy-Schwarz,
1 1 1
+ +
a b
c

1
1
1
+
+
y+z z+x x+y

1
1
1
1
+ +
2
yz
xy
zx

x+ y+ z
=

2 xyz

3 x+y+z

2 xyz

3
3 x+y+z
=
.
=
2
xyz
2r

Para la ultima identidad vea el nal de la demostracin del ejemplo 2.2.4.

4.2 Soluciones a los ejercicios del cap


tulo 2

167

Solucin 2.28 (i) Se sigue de las equivalencias siguientes,


o
(s a)(s b) < ab s2 s(a + b) < 0
a + b + c < 2(a + b)
c < a + b.
(ii) Use la transformacin de Ravi, a = y + z, b = z + x, c = x + y, para ver
o
que la desigualdad es equivalente a,
4(xy + yz + zx) (y + z)(z + x) + (z + x)(x + y) + (x + y)(y + z),
y para justicar la ultima desigualdad basta ver que, xy +yz +zx x2 +y 2 +z 2 ,

que se sabe del ejercicio 1.27.


Otra manera de obtener (ii).
La desigualdad es equivalente a,
3s2 2s(a + b + c) + (ab + bc + ca)

ab + bc + ca
.
4

Y sta a su vez es equivalente a 3(ab + bc + ca) 4s2 , que se reescribe como


e
3(ab + bc + ca) (a + b + c)2 , la cual se sigue tambin del ejercicio 1.27.
e
Solucin 2.29 La ley de los cosenos ayuda a ver que
o

2ab cos C 2ac cos B


a2 + b2 c2 a2 b2 + c2 =
= 2a

(b cos C)(c cos B)


b cos C + c cos B
= a2 .
2a
2

Solucin 2.30 Por la desigualdad de Cauchy-Schwarz, para cualesquiera x, y,


o
z, w 0, se tiene que

xy + zw (x + z)(y + w).
De donde,

c
clica

a2 + b2 c2

a2 b2 + c2 =

1
2

1
2

c
clica

a2 + b2 c2

c2 + a2 b2

a2 b2 + c2

c2 a2 + b2

(2a2 )(2c2 ) =
c
clica

ac.
c
clica

168

Soluciones a los Ejercicios y Problemas

Solucin 2.31 Considere nmeros positivos x, y, z, con a = y + z, b = z + x


o
u
y c = x + y. Las desigualdades son equivalentes a demostrar que
y+z z+x x+y
+
+
3
2x
2y
2z
Para la primera desigualdad use que
desigualdad de Nesbitt.

y
y
x

2x
2y
2z
+
+
3.
y+z z+x x+y
+

x
y

2 y para la segunda use la

Solucin 2.32 Como en los tringulos con la misma base la razn de sus alturas
o
a
o
es igual a la razn de sus reas, se tiene que,
o
a
PS
(P BC) (P CA) (P AB)
(ABC)
PQ PR
+
+
=
+
+
=
= 1.
AD BE CF
(ABC) (ABC) (ABC)
(ABC)
Utilice la desigualdad (2.3) de la seccin 2.3.
o
1
Solucin 2.33 (i) Recuerde que (S1 + S2 + S3 )( S1 +
o

1
S2

1
S3 )

9.

(ii) Los vrtices de los tringulos forman un hexgono que se divide en 6 tringue
a
a
a
los de reas S1 , S2 , S3 , T1 , T2 , T3 , donde los tringulos de reas Si y Ti tienen
a
a
a
un ngulo comn. Utilice la frmula de rea que involucra al seno del ngulo
a
u
o
a
a
para demostrar que S1 S2 S3 = T1 T2 T3 . Use despus la desigualdad M G M A,
e
como sigue
S

1
1
1
+
+
S1 S2 S3

(S1 + S2 + S3 + T1 + T2 + T3 )

18 6 S1 S2 S3 T1 T2 T3

= 18.
3
S1 S2 S3

1
1
1
+
+
S1 S2 S3

La igualdad se da cuando el punto O es el centroide del tringulo y las l


a
neas
que pasan por O son las medianas del tringulo, es decir, en el caso en que
a
S1 = S2 = S3 = T1 = T2 = T3 = 1 S.
6
Solucin 2.34 Si P = G es el centroide, la igualdad es clara, ya que AG =
o
GL
BG
CG
= GN = 2.
GM
BP
CP
AL
Por otro lado, si AP + P M + P N = 6, se tiene que P L + BM + CN = 9.
PL
PM
PN
(P CA)
PM
PN
No es dif ver que P L = (P BC) , BM = (ABC) y CN = (P AB) , por lo que
cil
AL
(ABC)
(ABC)
PL
AL

PM
BM

PN
CN

= 1. Lo que lleva a que

AL BM
CN
+
+
PL PM
PN

PL PM
PN
+
+
AL BM
CN

= 9.

4.2 Soluciones a los ejercicios del cap


tulo 2

169

Por la desigualdad (2.3), se sabe que esta igualdad ocurre solamente cuando
BM
CN
AL
P L = P M = P N = 3, lo que garantiza que P sea el centroide.
Solucin 2.35 (i) Como HD = DD , HE = EE y HF = F F , donde H es
o
el ortocentro5 . Luego, la solucin se sigue de la parte (i) del ejemplo 2.3.4.
o

e
o
(ii) Como AD = AD+DD = 1 + HD , se tiene tambin, al ver la solucin del
AD
AD
AD
BE
CF
HD
HE
HF
AD
ejemplo 2.3.4, que AD + BE + CF = 1 + AD + 1 + BE + 1 + CF = 4.
Como

AD
AD

BE
BE

CF
CF

AD
AD

BE
BE

CF
CF

9 se sigue el resultado.

Solucin 2.36 Como se seal en el ejemplo 2.3.5, la longitud de la bisectriz


o
n o
interna del ngulo A cumple
a
2
la = bc 1

a
b+c

4bc
(s(s a)).
(b + c)2

2
Como 4bc (b + c)2 , se tiene que la s(s a), y la lb s (s a)(s b)
(sa)+(sb)
c
s
= s 2 . Por lo tanto, la lb lc s s(s a)(s b)(s c) = s(sr),
2
2
2
2
la lb +lb lc +lc la s a+b+c = s2 , y la +lb +lc s(sa)+s(sb)+s(sc) = s2 .
2

Solucin 2.37 Sean = AM B, = BN A, = AP C, y sea (ABC) el


o
a
rea. Se tiene que
abc
1
.
(ABC) = a AM sen =
2
4R
De donde,
Luego,

bc
AM

= 2R sen . Anlogamente,
a

ca
BN

= 2R sen y

ab
CP

= 2R sen .

bc
ca
ab
+
+
= 2R(sen + sen + sen ) 6R.
AM
BN
CP
La igualdad se alcanza si M , N y P son los pies de las alturas.
Solucin 2.38 Sean A1 , B1 , C1 los puntos medios de los lados BC, CA, AB,
o
respectivamente, y sean B2 , C2 las reecciones de A1 con respecto a AB y CA,
respectivamente. Tambin considere a D la interseccin de AB con A1 B2 y E
e
o
la interseccin de CA con A1 C2 . Entonces,
o
2DE = B2 C2 C2 B1 + B1 C1 + C1 B2 = A1 B1 + B1 C1 + C1 A1 = s.
Use que A1 DAE es un cuadriltero inscrito en una circunferencia de dimetro
a
a
AA1 y use la ley de senos en ADE, para deducir que DE = AA1 sen A =
5

Consultar [6], pg. 85 o [9], pg. 37.


a
a

170

Soluciones a los Ejercicios y Problemas

a
ma sen A. Entonces, s 2DE = 2ma sen A = 2ma 2R = ama , es decir,
R
ama sR. Anlogamente, puede vericar que bmb sR y cmc sR.
a

Solucin 2.39 La desigualdad es equivalente a 8(s a)(s b)(s c) abc,


o
donde s es el semiper
metro.
Como (ABC) = sr = abc = s(s a)(s b)(s c), donde R y r denotan el
4R
circunradio y el inradio, respectivamente, se tiene que demostrar solamente que
8sr 2 abc. Es decir, que 8sr 2 4Rrs, que es equivalente a 2r R.
Solucin 2.40 El rea del tringulo ABC, satisface que (ABC) = abc =
o
a
a
4R
(a+b+c)r
1
1
1
1
1
, luego, ab + bc + ca = 2Rr R2 , donde R y r denotan el circunradio
2
y el inradio, respectivamente.
Solucin 2.41 Use el ejercicio 2.40 y la ley de los senos.
o
Solucin 2.42 Use que6 sen A = (sb)(sc) , donde s es el semiper
o
metro.
2
bc
B
C
Expresiones semejantes para sen 2 y sen 2 , para ver que
sen

B
C
(s a)(s b)(s c)
sr 2
r
1
A
sen sen =
=
=
,
2
2
2
abc
abc
4R
8

donde R y r denotan el circunradio y el inradio, respectivamente.


Solucin 2.43 Sabe por la desigualdad (2.3), que
o
(a + b + c)

1 1 1
+ +
a b
c

9.

Como a + b + c 3 3R, se tiene que

1 1 1
3
+ +
.
a b
c
R

(4.4)

Usando nuevamente la desigualdad (2.3), se obtiene


1

+
+
3 2A 2B 2C

Sea f (x) = log 2x , como f (x) =


de Jensen,

1
x2

sen2

A
2

1cos A
2

2
2
2
1 b +c a
2bc
2

3
3
= .
2
+ B + C)

(4.5)

> 0, f es convexa. Usando la desigualdad

1
log
+ log
+ log
3
2A
2B
2C
6

2
(A

log

a2 (bc)2
4bc

+
+
3 2A 2B 2C

(sb)(sc)
.
bc

4.2 Soluciones a los ejercicios del cap


tulo 2

171

Usando (4.5) y el hecho que log x es estrictamente creciente, se tiene que

1
log
+ log
+ log
3
2A
2B
2C

3
log .
2

1
Suponga que a b c, lo cual implica A B C. Entonces a

log 2A log 2B log 2C . Usando la desigualdad de Tchebyshev,

1
1

1
log
+ log
+ log

a
2A b
2B c
2C

1 1 1
+ +
a b
c

log

2A

(4.6)
1
b

1
c

+ log 2B + log 2C
3

Entonces, por (4.4) y (4.6), se tiene

3
1
3
log
+ log
+ log

log .
a
2A b
2B
c
2C
R
2
Elevando a las potencias adecuadas y tomando rec
procos, obtiene la desigualdad deseada. En todas las desigualdades anteriores, la igualdad se tiene si y slo
o
si a = b = c (esto es, la igualdad se tiene slo para tringulos equilteros).
o
a
a
Solucin 2.44 Por la ley de los senos, se tiene que
o
sen B
sen C
1
sen A
=
=
=
,
a
b
c
2R
donde a, b, c son las longitudes de los lados del tringulo y R es el circunradio.
a
Entonces,
sen2 A + sen2 B + sen2 C =
=

b2
c2
a2
+
+
4R2 4R2 4R2
1
(a2 + b2 + c2 )
4R2
1
9
9R2 = ,
2
4R
4

donde la desigualdad se sigue de la desigualdad de Leibniz.


Solucin 2.45 Use la desigualdad de Leibniz y el hecho de que el rea de un
o
a
abc
tringulo est dada por (ABC) = 4R .
a
a
Solucin 2.46 Note que el inc
o
rculo de ABC es el circunc
rculo de DEF .
Aplicando la desigualdad de Leibniz a DEF , se obtiene
EF 2 + F D 2 + DE 2 9r 2 ,

172

Soluciones a los Ejercicios y Problemas

donde r es el inradio de ABC. Por otra parte, usando el teorema 2.4.3 se obtiene
que s2 27r 2 , de donde
EF 2 + F D 2 + DE 2

s2
.
3

Solucin 2.47
o
a2
b2
c2
+
+
hb hc hc ha ha hb

a2 bc + b2 ca + c2 ab
abc(a + b + c)
=
2
4(ABC)
4(ABC)2
2R
abc(a + b + c)
=
4.
abc (a+b+c)r
r
4

=
=

4R

Solucin 2.48 Recuerde que sen2


o
3
cos C 2 (vea el ejemplo 2.5.2).

A
2

1cos A
2

y use que cos A + cos B +

Solucin 2.49 Observe que,


o

4 3(ABC)

9abc
9 4 Rrs
2s
4 3rs
2 3s 9R R.
a+b+c
2s
3 3

La ultima desigualdad la se demostr en el teorema 2.4.3.

o
Solucin 2.50 Use el ejercicio anterior y la desigualdad entre la media armnica
o
o
y la media geomtrica
e
3
1
ab

1
bc

1
ca

a2 b2 c2 .

Solucin 2.51 Use el ejercicio anterior y la desigualdad M G M A


o

a2 b2 c2

a2 + b2 + c2
.
3

Solucin 2.52 Primero observe que si s =


o

a+b+c
2

entonces

a2 + b2 + c2 (a b)2 (b c)2 (c a)2 =

= a2 (b c)2 + b2 (c a)2 + c2 (a b)2

= 4{(s b)(s c) + (s c)(s a) + (s a)(s b)}.

4.2 Soluciones a los ejercicios del cap


tulo 2

173

Por lo que, si x = s a, y = s b, z = s c, entonces la desigualdad es


equivalente a

3 xyz(x + y + z) xy + yz + zx.
Elevando al cuadrado y reduciendo trminos, se tiene que la desigualdad anterior
e
es equivalente a
xyz(x + y + z) x2 y 2 + y 2 z 2 + z 2 x2 .
Deduzca sta ultima de la desigualdad de Cauchy-Schwarz con (xy, yz, zx) y
e

(zx, xy, yz).


Solucin 2.53 Use el ejercicio 2.50 y la desigualdad 3 3 (ab)(bc)(ca) ab +
o
bc + ca.
Solucin 2.54 Note que
o
3(a + b + c)abc
9abc

ab + bc + ca
a+b+c

(a + b + c)2 3(ab + bc + ca)


a2 + b2 + c2 ab + bc + ca,

ahora use el ejercicio 2.49.


Solucin 2.55 Utilice (2.5), (2.6) y (2.7), observe que a2 + b2 + c2 + 4abc =
o
1
2
2 2r .
Solucin 2.56 Utilice las relaciones usadas en la solucin al ejercicio 2.39
o
o
(b + c a)(c + a b)(a + b c)
abc

=
=
=

8(s a)(s b)(s c)


abc
8s(s a)(s b)(s c)
4Rs( abc )
4R
2r
8(rs)2
= .
4Rs(rs)
R

174

Soluciones a los Ejercicios y Problemas

Solucin 2.57 Observe que


o
b2
c2
1
a2
+
+
=
b+ca c+ab a+bc
2

b2
c2
a2
+
+
sa sb sc

sb
sc
sa
a+
b+
c
sa
sb
sc

1
2

s
2

s (a + b + c)s2 2(ab + bc + ca)s + 3abc


s
2
(s a)(s b)(s c)

b
c
a
+
+
sa sb sc

s 2s3 2s(s2 + r 2 + 4rR) + 3(4Rrs)


s
2
r2s

2s(R R )
2s(R r)
3 3rR
2
=

= 3 3R,
r
r
r
=

las dos ultimas desigualdades se derivan del hecho que R 2r (lo que implica

que r R ) y que s 3 3r, respectivamente.


2
Solucin 2.58 Debe partir del lado de las ecuaciones donde se encuentra la
o
relacin entre las s y realice las operaciones.
o
Solucin 2.59 Si x1 , 1 x1 , x2 , 1 x2 , . . ., son las longitudes en que queda
o
dividido cada lado por el punto correspondiente, se tiene que a2 + b2 + c2 + d2 =
1
1
(x2 + (1 xi )2 ). Muestre que 2 2(xi 1 )2 + 2 = x2 + (1 xi )2 1.
i
i
2
Para la parte (ii), la desigualdad de la derecha se sigue de la desigualdad del
tringulo. Para la desigualdad de la izquierda use reexiones en los lados, como
a
en la siguiente gura.

d
b

4.2 Soluciones a los ejercicios del cap


tulo 2

175

Solucin 2.60 Esta es igual a (ii) del problema anterior.


o
Solucin 2.61 Si ABC es el tringulo y DEF GHI es el hexgono con DE,
o
a
a
F G, HI paralelos a BC, AB, CA, respectivamente, se tiene que el per
metro
del hexgono es 2(DE + F G + HI). Sean X, Y , Z los puntos de tangencia
a
del inc
rculo con los lados BC, CA, AB, respectivamente, y sea p = a + b + c
el per
metro del tringulo ABC. Dena x = AZ = AY , y = BZ = BX y
a
z = CX = CY , se tiene entonces la relacin
o
AE + ED + DA
2x
DE
=
=
.
a
p
p
Anlogamente, se tienen las otras dos relaciones
a
2z
FG
=
,
c
p

HI
2y
=
.
b
p

Luego,
p(DEF GHI) =

4(xa + yb + zc)
p

4(a(s a) + b(s b) + c(s c))


2s
4((a + b + c)s (a2 + b2 + c2 ))
=
2s
(a2 + b2 + c2 )
= 2(a + b + c) 4
(a + b + c)
=

1
pero, a2 + b2 + c2 3 (a + b + c)(a + b + c) por la desigualdad de Tchebyshev.
Por lo tanto, p(DEF GHI) 2(a + b + c) 4 (a + b + c) = 2 (a + b + c).
3
3

Solucin 2.62 Considere el circunc


o
rculo del tringulo equiltero con lados
a
a
de longitud 2. Los c
rculos con centro en los puntos medios de los lados del
tringulo y radio 1 cubren al c
a
rculo de radio 2. Si un c
rculo de radio mayor que

2 3
rculos de radio 1, entonces uno de los tres c
rculos
3 es cubierto por tres c
cubre una cuerda de longitud mayor a 2.
Solucin 2.63 Tome el tringulo acutngulo de lados de longitud 2r1 , 2r2 y
o
a
a
2r3 , si existe. Su circunradio es el buscado. Si no existe el tringulo, la solucin
a
o
es el mayor radio entre r1 , r2 y r3 .
Solucin 2.64 Lema 1. Si un cuadrado de lado de longitud a est dentro del
o
a
rectngulo de lados de longitud c y d, entonces a m {c, d}.
a
n
Demostracin.
o

176

Soluciones a los Ejercicios y Problemas

d
a
c
Por los vrtices del cuadrado dibujamos rectas paralelas a los lados del rectngulo
e
a
de tal manera que encierren al cuadrado como se muestra en la gura. Como las
rectas paralelas forman un cuadrado dentro del rectngulo se tiene el resultado.
a
Lema 2. La longitud de la diagonal del cuadrado inscrito en un tringulo es
a
menor o igual a la longitud de la bisectriz interna del ngulo recto.
a
Demostracin. Sea ABC un tringulo rectngulo con hipotenusa CA y sea
o
a
a
P QRS el cuadrado inscrito.
A

S
T
P

Podemos suponer que los vrtices P y Q pertenecen a los catetos del tringue
a
lo rectngulo (en caso de no ser as transladamos el cuadrado) y sea O la
a
,
intereseccin de las diagonales P R y QS.
o
Como BQOP es c
clico (B = O = 90 ), se sigue que QBO = QP O =
, entonces O pertenece a la bisectriz interna del ngulo B. Sea T la in45
a
terseccin de BO con RS, entonces QBT = QST = 45 , luego BQT S
o
es c
clico y el centro O del circunc
rculo de BQT S es la interseccin de las
o
mediatrices de los segmentos SQ y BT , pero la mediatriz de SQ es P R, por lo
tanto el punto O pertenece a P R y si V es el punto medio de BT , se tiene que
V OO es un tringulo rectngulo. Como O O > O V , se tiene que las cuerdas
a
a
SQ y BT satisfacen SQ < BT , que es lo que se desea.

4.2 Soluciones a los ejercicios del cap


tulo 2

177

Termine ahora la demostracin del ejercicio. Sea ABCD el cuadrado de lados


o
de longitud 1 y sea l la recta que separa los dos cuadrados.
D F

B
G

Si l es paralela a uno de los lados del ABCD, entonces aplique el lema 1.


En caso contrario, l intersecta cada recta que determina uno de los lados del
cuadrado ABCD. Suponga que A es el vrtice ms lejano a l.
e
a
Si l corta a los lados de ABCD en E, F , G, H como en la gura, se tiene por
el lema 2 que la suma de las diagonales de los cuadrados pequeos es menor o
n

igual a AC, esto es 2(a + b) 2, luego el resultado.


Solucin 2.65 Si , , son los ngulos centrales que abren las cuerdas de
o
a
longitudes a, b, c, respectivamente, se tiene que a = 2 sen , b = 2 sen y
2
2
c = 2 sen . Luego,
2
abc = 8 sen

sen sen 8 sen3


2
2
2

++
6

= 8 sen3 (30 ) = 1.

La desigualdad se sigue del ejercicio 1.81.


Solucin 2.66 Una primera observacin es ver que las diagonales son paralelas
o
o
a los lados. Sea X el punto de interseccin de las diagonales AD y CE. Ahora,
o
se puede dividir el pentgono como
a
(ABCDE) = (ABC) + (ACX) + (CDE) + (EAX).
Como ABCX es un paralelogramo, se tiene que (ABC) = (CXA) = (CDE).
Sean a = (CDX) = (EAX) y b = (DEX), entonces se obtiene que a =
b

(CXA)
= (CDX) = a+b , de donde
a
para encontrar (ABCDE).
AX
XD

a
b

1+ 5
2 .

Ahora, ya cuenta con los elementos

178

Soluciones a los Ejercicios y Problemas

Solucin 2.67 Primero se tiene que probar sr = s1 R = (ABC), donde s1 es


o
el semiper
metro del tringulo DEF . Para deducir esta igualdad bastar que
a
a
vea que los radios OA, OB y OC son perpendiculares a EF , F D y DE,
respectivamente. Use tambin que R 2r.
e
Solucin 2.68 Suponga que el ngulo mximo es A y que ste satisface que
o
a
a
e
60 A 90 , entonces la longitud de las alturas hb y c son tambin menores
h
e
hb hc
3
a 1. Utilice ahora el hecho que (ABC) = 2 sen A y que 2 sen A 1. El caso
del tringulo obtuso es ms fcil.
a
a a
Solucin 2.69 Si ABCD es el cuadriltero con lados de longitud a = AB,
o
a
b = BC, c = CD y d = DA.
(i) (ABCD) = (ABC) + (CDA) = ab sen B + cd sen D ab+cd .
2
2
2
(ii) Si ABCD es el cuadriltero mencionado con lados de longitud a, b, c y
a
d, considere el tringulo BC D que resulta de reejar DCB con respecto a la
a
mediatriz de BD. Los cuadrilteros ABCD y ABC D tienen la misma rea
a
a
pero el segundo tiene lados de longitud a, c, b y d, en este orden. Utilice ahora
(i).
(iii) (ABC) ab , (BCD) bc , (CDA) cd y (DAB) da .
2
2
2
2
Solucin 2.70 En el ejemplo 2.7.6 se demostr que
o
o
PA PB PC

R
(pa + pb )(pb + pc )(pc + pa ).
2r

Utilice la desigualdad M G M A.
Solucin 2.71 (i)
o
(ii)
(iii)

PA
pb +pc

P
A
pb pc

PB
pc +pa

P A2
pb pc

P
B
pc pa

PC
pa +pb

P B2
pc pa

33

P
C
pa pb

33

P C2
pa pb

33

P A2 P B 2 P C 2
pb pc pc pa pa pb

PA
PB
PC
pb +pc pc +pa pa +pb

33

P
P
P
A B C
pb pc pc pa pa pb

R
2r

33

4R 2
r

12.

3.

4R
r

6.

Para las ultimas desigualdades en (i) y (iii) se ha utilizado el ejercicio 2.70.

Para la ultima desigualdad en (ii) use el ejemplo 2.7.6.

(iv) Proceda como en el ejemplo 2.7.5, es decir, haga una inversin en una
o
circunferencia de centro P y radio d (arbitrario, por ejemplo d = pb ). Sean A ,
B , C los inversos de A, B, C. Sean p , p , p las distancias de P a los lados
a
c
b
B C , C A , A B , respectivamente.

Pruebe que p = pa P B2P C como sigue. Se tiene que


a
d
p B C = 2(P B C ) =
a

pa P B P C B C
P B P C BC
=
,

P A1
d2

4.2 Soluciones a los ejercicios del cap


tulo 2

179

donde A es el inverso de A1 , el pie de la perpendicular de P sobre BC.


1

Anlogamente, p = pb P C2P A y p = pc P A2P B .


a
c
b
d
d
La desigualdad de Erds-Mordell aplicada en el tringulo A B C , garantiza que
o
a
P A + P B + P C 2(p + p + p ).
c
a
b
Pero como P A P A = P B P B = P C P C = d2 , al sustituir se tiene que
1
1
pb
pc
pa
1
+
+
2
+
+
PA PB PC
PB PC
PC PA PC PA
y esta desigualdad es equivalente a
P B P C + P C P A + P A P B 2(pa P A + pb P B + pc P C).
Finalmente, para concluir use el ejemplo 2.7.4.
Solucin 2.72 Si P es un punto interior o en el per
o
metro del tringulo ABC,
a
utilice la demostracin del teorema 2.7.2.
o
Si ha es la longitud de la altura desde A, se tiene que el rea del tringulo ABC
a
a
satisface que 2(ABC) = aha = apa + bpb + cpc .
Como ha P A + pa (an si pa 0, esto es, si P es un punto que est fuera
u
a
del tringulo, en distinto lado de BC que A), adems la igualdad se da si P
a
a
est sobre la altura por A. Luego aP A + apa aha = apa + bpb + cpc , entonces
a
aP A bpb + cpc .
Puede aplicar esta desigualdad al tringulo AB C simtrico a ABC con resa
e
pecto a la bisectriz interna del ngulo A, donde aP A cpb + bpc , con igualdad
a
cuando AP pase por O.
Anlogamente, bP B apc + cpa y cP C apb + bpa , por lo tanto
a
PA + PB + PC

b c
+
c b

pa +

c a
pb +
+
a c

a b
+
b a

pc .

La igualdad se da cuando P sea el circuncentro O.


Segunda Solucin. Sean L, M y N los pies de las perpendiculares P sobre BC,
o
CA y AB, respectivamente. Sean H y G la proyecciones ortogonales de B y C,
respectivamente, sobre la recta M N . Luego BC HG = HN + N M + M G.
Como BN H = AN M = AP M , los tringulos rectngulos BN H y AP M
a
a
a
son semejantes, por lo que HN = P M BN. De manera anloga, se tiene que
PA
M G = P N CM .
PA
Por el teorema de Ptolomeo, aplicado a AM P N , se tiene que P A M N =
AN P M + AM P N , por lo que
MN =

AN P M + AM P N
,
PA

180

Soluciones a los Ejercicios y Problemas

de donde
BC

AN P M + AM P N
PN
PM
BN +
+
CM.
PA
PA
PA

Por lo tanto,
BC P A P M AB + P N CA.
b
c
a
Luego, P A pb a + pc a . Anlogamente para las otras dos desigualdades.

Solucin 2.73 Considere la sucesin de reexiones del cuadriltero ABCD,


o
o
a
como en la siguiente gura.
P
B
A
B
S

D
S

C
R

A
A

Note que el per


metro de P QRS es la longitud de la l
nea quebrada P QR S P .
B es paralela a AB, que la distancia ms corta es AA ,
Note tambin que A
e
a
como se puede ver, se logra si se proyecta O sobre los lados del cuadriltero.
a
Solucin 2.74 Primero note que (DEF ) = (ABC) (AF E) (F BD)
o
(EDC). Si x = BD, y = CE, z = AF , ax = DC, by = EA y cz = F B,
se tiene que,
z(b y) (F BD)
x(c z) (EDC)
y(a x)
(AF E)
=
,
=
y
=
.
(ABC)
cb
(ABC)
ac
(ABC)
ba
Luego,
(DEF )
(ABC)

y
x
z
y
x
z
1

1
c
b
a
c
b
a
x
y
z
x y z
x y z
1
1
1
+ =2 .
a
b
c
a b c
a b c

= 1
=

4.2 Soluciones a los ejercicios del cap


tulo 2

181

La ultima igualdad se debe a que la concurrencia de las cevianas nos garantiza

y
z
x

a
que ax by cz = 1. Ahora bien, el ultimo producto es mximo cuando
y
z
1
x
u
a = b = c y como los segmentos concurren el valor comn es 2 . Luego, se
tiene que P es el centroide.
Solucin 2.75 Si x = P D, y = P E y z = P F , se tiene que 2(ABC) =
o
ax + by + cz. Por la desigualdad de Cauchy-Schwarz,
(a + b + c)2

a
b
c
+ +
x y z

(ax + by + cz) .

b
c
a
Luego, x + y + z (a+b+c) . La igualdad se da cuando x = y = z, es decir,
2(ABC)
cuando P es el incentro.

Solucin 2.76 Primero vea que BD2 + CE 2 + AF 2 = DC 2 + EA2 + F B 2 ,


o
usando que BD2 DC 2 = P B 2 P C 2 y relaciones similares.
Ahora bien, (BD + DC)2 = a2 , de donde BD2 + DC 2 = a2 2BD DC.
Anlogamente, para los otros dos lados. Luego, BD2 + DC 2 + CE 2 + AE 2 +
a
AF 2 + F B 2 = a2 + b2 + c2 2(BD DC + CE AE + AF F B).
As la suma es m
,
nima cuando (BD DC + CE AE + AF F B) sea mxima.
a
BD+DC 2
a 2
Pero BD DC
= 2 y alcanza el mximo cuando BD = DC.
a
2
Anlogamente, CE = EA y AF = F B, por lo tanto P es el circuncentro.
a
Solucin 2.77 Como
o

(aP D)(bP E)(cP F )


8 (ABC)3
27 abc .

aP D+bP E+cP F
3

2(ABC)
,
3

se

tiene que P D P E P F
Adems, la igualdad ocurre si y slo si
a
o
aP D = bP E = cP F .
Pero c P F = b P E (ABP ) = (CAP ) P est sobre la mediana AA .
a
De manera anloga, vea que P se encuentra en las otras medianas, entonces P
a
es el centroide.
Solucin 2.78 Usando la tcnica para la demostracin del teorema de Leibniz,
o
e
o
1
verique que 3P G2 = P A2 + P B 2 + P C 2 3 (a2 + b2 + c2 ), donde G es el
centroide. Por lo tanto, se tiene que el punto ptimo es P = G.
o
Solucin 2.79 El cuadriltero AP M N es c
o
a
clico y est inscrito en la circunfea
rencia de dimetro AP .
a
La cuerda M N subtiende siempre el ngulo A (o 180 A) por lo que la
a
longitud de M N depende proporcionalmente del radio de la circunferencia que
circunscribe a AP M N .
Tendr la mayor circunferencia cuando el dimetro AP sea lo ms grande posia
a
a
ble. Esto sucede cuando P es diametralmente opuesto a A. En este caso M y

182

Soluciones a los Ejercicios y Problemas

A
N
P

M
B

N coinciden con B y C, respectivamente. Por lo tanto, la cuerda mxima M N


a
es BC.
Solucin 2.80 El circunc
o
rculo de DEF es el c
rculo de los nueve puntos de
ABC, luego intersecta tambin los puntos medios de los lados de ABC y pasa
e
por L, M , N , los puntos medios de AH, BH, CH, respectivamente. Note que
t2 = AL AD, luego
a
t2
a
ha

AL AD
=
AD

R cos A 3R cos

AL =

OA

3
A+B+C
= 3R cos 60 = R.
3
2

Observe que se puede probar un resultado ms fuerte


a

t2
a
ha

= R + r, utilizando

A
F

L E
H

M
O
A

4.2 Soluciones a los ejercicios del cap


tulo 2
el hecho que cos A + cos B + cos C =

r
R

183

+ 1, vea el lema 2.5.2.

Solucin 2.81 (i) Note que


o
pc
pb
pa
+
+
ha hb hc

cpc
apa
bpb
+
+
aha
bhb chc

2(P BC) + 2(P CA) + 2(P AB)


= 1.
2(ABC)

Use ahora que


pb
pc
pa
+
+
ha hb hc

ha hb hc
+
+
pa
pb
pc

9.

(ii) Por la desigualdad M G M A se tiene


27

pa pb pc
ha hb hc

pa
pc
pb
+
+
ha hb hc

= 1,

de donde se obtiene la ultima igualdad de (i).

(iii) Sean x = (P BC), y = (P CA) y z = (P AB). Observe que a(ha pa ) =

aha apa = 2(y + z) 4 yz. Anlogamente, se tiene que b(hb pb ) 4 zx


a

y c(hc pc ) 4 xy. Entonces,


a(ha pa )b(hb pb )c(hc pc ) 64xyz = 8(apa bpb cpc ).
Por lo tanto, (ha pa )(hb pb )(hc pc ) 8pa pb pc .
Solucin 2.82 Suponga que a < b < c, luego de todas las alturas del tringulo
o
a
ABC, AD es la mayor. Si E es la proyeccin de I sobre AD, bastar ver que
o
a
AE AO = R. Recuerde que la bisectriz del ngulo A es tambin bisectriz del
a
e
a
ngulo EAO. Si proyecta I en E sobre el dimetro AA , entonces AE = AE .
a
Ahora vea que AE AO, mostrando que I se encuentra dentro del tringulo
a
acutngulo COF , donde F es la interseccin de AA con BC.
a
o
Para ver que COF es un tringulo acutngulo, use que los ngulos de ABC
a
a
a
cumplen A < B < C entonces 1 B < 90 A, 1 C < 90 A. Use
2
2
tambin que COF = A + C B < 90 .
e
Solucin 2.83 Sea ABC un tringulo con lados de longitud a, b y c. Utilice
o
a
la frmula de Hern para calcular el rea de un tringulo se tiene que
o
o
a
a
(ABC) =

s(s a)(s b)(s c)

donde s =

a+b+c
.
2

(4.7)

184

Soluciones a los Ejercicios y Problemas

Si s y c estn jos, entonces tambin est jo s c. Luego el producto de


a
e
a
16(ABC)2 es mximo cuando (sa)(sb) sea mximo. Esto es, si sa = sb,
a
a
es decir, cuando a = b. Por lo tanto, el tringulo es issceles.
a
o
Solucin 2.84 Sea ABC un tringulo con lados de longitud a, b y c. Como el
o
a
per
metro es jo, lo es tambin el semiper
e
metro, utilizando la ecuacin (4.7),
o
se tiene que 16(ABC)2 es mxima cuando (s a)(s b)(s c) es mximo. El
a
a
producto de estos tres nmeros es mximo cuando (s a) = (s b) = (s c),
u
a
es decir, cuando a = b = c. Por lo tanto, el tringulo es equiltero.
a
a
Solucin 2.85 Si a, b, c son las longitudes de los lados del tringulo, observe
o
a
A+B+C
, ya
que a + b + c = 2R(sen A + sen B + sen C) 6R sen
3
que la funcin sen x es cncava. Adems, se tiene la igualdad cuando sen A =
o
o
a
sen B = sen C.
Solucin 2.86 La desigualdad (lm + mn + nl)(l + m + n) a2 l + b2 m + c2 n
o
es equivalente a
l2 + m2 c2 m2 + n2 a2 n2 + l2 b2
+
+
+30
lm
mn
ln
3
cos AP B + cos BP C + cos CP A + 0.
2
Ahora, use el hecho que cos + cos + cos +
2 cos

+ cos
2
2

3
2

+ sen2

0 es equivalente a

0.

Solucin 2.87 Considere el punto de Fermat F y sean p1 = F A, p2 =


o
1
F B y p3 = F C, entonces observe primero que (ABC) = 2 (p1 p2 + p2 p3 +
a2 +b2 +c2 =

3
e
4 (p1 p2 + p2 p3 + p3 p1 ). Tambin,
2p2 +2p2 +2p2 2p1 p2 cos 120 2p2 p3 cos 120 2p3 p1 cos 120
3
2
1

p3 p1 )sen 120 =

= 2(p2 + p2 + p2 ) + p1 p2 + p2 p3 + p3 p1 .
1
2
3
Ahora usando que x2 + y 2 2xy, se tiene que a2 + b2 + c2 3(p1 p2 + p2 p3 +

4
p3 p1 ) = 3 3 3(ABC) . Luego, a2 + b2 + c2 4 3(ABC).

Ms an, la igualdad a2 +b2 +c2 4 3(ABC) se cumple cuando p2 +p2 +p2 =


a u
3
2
1
p1 p2 + p2 p3 + p3 p1 , es decir, cuando p1 = p2 = p3 y el tringulo es equiltero.
a
a

4.2 Soluciones a los ejercicios del cap


tulo 2

185

Solucin 2.88 Sean a, b, c las longitudes de los lados del tringulo ABC.
o
a
Dena, al igual que en el ejercicio anterior, sean p1 = F A, p2 = F B y p3 = F C.
Recuerde que, por la solucin del ejercicio anterior, se tiene
o

4 3(ABC) = 3(p1 p2 + p2 p3 + p3 p1 ).
Luego, lo unico que debe probar es

3(p1 p2 + p2 p3 + p3 p1 ) (p1 + p2 + p3 )2
pero, esto es equivalente a p1 p2 +p2 p3 +p3 p1 p2 +p2 +p2 que es un resultado
3
2
1
conocido, ver ejercicio 1.27.
Solucin 2.89 Como en el problema de Fermat, hay dos casos, cuando ABC
o
tiene todos sus ngulos menores a 120 o cuando hay un ngulo mayor de 120 .
a
a
, donde C es la imagen
En el primer caso el m
nimo de P A + P B + P C es CC
de A, al rotar la gura con centro en B un ngulo de 60 y en direccin positiva.
a
o
Por la ley de los cosenos se tiene que
(CC )2 = b2 + c2 2bc cos (A + 60 )

= b2 + c2 bc cos A + bc 3sen A

1 2
(a + b2 + c2 ) + 2 3(ABC).
=
2

Ahora, utilice que a2 +b2 +c2 4 3(ABC) para obtener (CC )2 4 3(ABC).
2
Por el teorema 2.4.3 se tiene que (ABC) 3 3r , por lo tanto (CC )2 36r 2 .
En el caso en que A 120 , el punto que resuelve el problema de FermatSteiner es el punto A, por lo que P A + P B + P C AB + AC = b + c. Por lo
tanto, lo unico que se tiene que probar es b + c 6r. Ms an, para esto puede

a u
xyz
usar el hecho que b = x + z, c = x + y y r = x+y+z .
Segunda Solucin. Es claro que P A + pa ha , donde pa es la distancia de
o
P al lado BC y ha es la longitud de la altura desde A. Luego, ha + hb + hc

(P A + P B + P C) + (pa + pb + pc ) 3 (P A + P B + P C), donde la ultima


2
desigualdad se debe al teorema de Erds-Mordell.
o
1
1
1
Ahora usando el ejercicio 1.36 se tiene que 9 (ha + hb + hc )( ha + hb + hc ) =
3
(ha + hb + hc )( 1 ). Por lo que, 9r ha + hb + hc 2 (P A + P B + P C) y de
r
aqu el resultado.

Solucin 2.90 Primero note que (A1 B1 C1 ) = 1 A1 B1 A1 C1 sen B1 A1 C1 .


o
2
Como P B1 CA1 es un cuadriltero c
a
clico de dimetro P C, aplicando la ley de
a
los senos se obtiene que A1 B1 = P C sen C. Anlogamente A1 C1 = P B sen B.
a

186

Soluciones a los Ejercicios y Problemas

Llame Q a la interseccin de BP con el circunc


o
rculo del tringulo ABC, luea
go, se tiene B1 A1 C1 = QCP . En efecto, como P B1 CA1 es un cuadriltero
a
c
clico se tiene que B1 CP = B1 A1 P . Anlogamente, C1 BP = C1 A1 P .
a
Entonces, B1 A1 C1 = B1 A1 P + C1 A1 P = B1 CP + C1 BP , pero
C1 BP = ABQ = ACQ. Por lo tanto, B1 A1 C1 = B1 CP + ACQ =
QCP .
Otra vez, la ley de los senos garantiza que

sen QCP
sen BQC

PQ
PC .

B1

C1
P

A1

(A1 B1 C1 ) =
=
=
=
=

1
A1 B1 A1 C1 sen B1 A1 C1
2
1
P B P C sen B sen C sen QCP
2
PQ
1
P B P C sen B sen C
sen QCP
2
PC
1
P B P Q sen A sen B sen C
2
(R2 OP 2 )(ABC)
.
4R2

La ultima igualdad se da ya que la potencia del punto P , con respecto al

circunc
rculo de ABC, es P B P Q = R2 OP 2 y el rea del tringulo ABC
a
a
2 sen A sen B sen C. Luego, (A B C ) es mxima
est dada por (ABC) = 2R
a
a
1 1 1
cuando P = O, es decir, cuando A1 B1 C1 es el tringulo medial.
a

4.3 Soluciones a los problemas del cap


tulo 3

4.3.

187

Soluciones a los problemas del cap


tulo 3

Solucin 3.1 Considere a = A1 A2 , b = A1 A3 y c = A1 A4 y aplique el


o
teorema de Ptolomeo al cuadriltero A1 A3 A4 A5 para tener que ab + ac = bc,
a
a
a
o equivalentemente que b + c = 1.
B1 B2
A1 A2
Como los tringulos A1 A2 A3 y B1 B2 B3 son semejantes B1 B3 = A1 A3 = a y
a
b
SB +SC
a2
a2
a
=
b . Anlogamente C1 C2 = c . Por lo que
SA
2
2 +c2
(b+c)
b
1
(b+c)2 > 2(b+c)2 = 2 . La tercera igualdad se sigue

de aqu se obtiene B1 B2 =

a2
b2

2 2

2 2

+a
+ a2 = a cb2 c2 b =
c
de ab + ac = bc y la desigualdad por la desigualdad (1.11). La desigualdad es
estricta ya que b = c.
2
2
2
2
2
Note que, a2 + a2 = a + a 2 a = 1 2 a .
b
c
bc
bc
b
c
Ley de los senos aplicada al tringulo A1 A3 A4 lleva a que
a

sen 2
sen 2
7
7
2
2
4 =
2
sen 7 sen 7
2sen 7 sen 7 cos 2
7

a2
bc

sen 2
sen 2
7
7
=
2(1 cos2 2 ) cos 2
2 cos 2 (1 + cos 2 )(1 cos 2 )
7
7
7
7
7

=
>

sen 2
1
7
=
2
2
2
2
4 cos 7 (1 + cos 7 )sen 7
4 cos 7 (1 + cos 2 )
7

21
1
1

.
=
=
2
2
4 cos (1 + cos )
2
4 (1 + )
4
4
2

Luego

a2
b2

a2
c2

2
= 1 2 a < 1 ( 2 1) = 2 2.
bc

Solucin 3.2 Corte el tetraedro por las aristas DA, DB, DC y desdblelo
o
o
en el plano del tringulo ABC. Las caras ABD, BCD y CAD tendrn por
a
a
imagen a los tringulos ABD1 , BCD2 y CAD3 . Vea que D1 , B y D2 son
a
colineales as como tambin lo son D3 , A y D1 , y que B y A son puntos medios

e
1
de D1 D2 y D3 D1 , respectivamente. Luego, AB = 2 D2 D3 y por la desigualdad
del tringulo D2 D3 CD3 + CD2 = 2CD. Por lo tanto AB CD, como se
a
quer
a.
Solucin 3.3 Sea S el rea del tringulo entonces se tienen las siguientes
o
a
a
2S
frmulas sen = 2S , sen = 2S , sen = 2S , y r = S = a+b+c . Use estas
o
bc
ca
ab
s
frmulas para obtener que la desigualdad a probar es equivalente a
o
a
b
c
+
+
bc ca ab

(a + b + c) 9,

188

Soluciones a los Ejercicios y Problemas

la cual puede ser mostrada usando la desigualdad M G M A en cada factor


del lado izquierdo.
Solucin 3.4 Suponga que los c
o
rculos son de radio 1. Sea P el punto comn
u
de los c
rculos y sean A, B, C los otros puntos de interseccin. El rea de los
o
a
gajos ser m
a nima si el punto P est dentro del tringulo ABC (sino rote un
a
a
alrededor de P , y esto reducir el rea).
c
rculo 180
a a

El rea de los gajos es igual a (sen + sen + sen ), donde , , son


a
los ngulos centrales que abren los arcos comunes de los c
a
rculos. Es claro que
. Como la funcin sen x es cncava, el m
+ + = 180
o
o
nimo se logra cuando
= = = , lo que implica que los centros de las circunferencias forman
3
un tringulo equiltero.
a
a
Solucin 3.5 Sea I el incentro del tringulo ABC y dibuje la recta por I
o
a
perpendicular a IC. Sean D , E las interesecciones de esta recta con BC y
CA, respectivamente. Primero pruebe que (CDE) (CD E ) usando que el
a
rea de D DI es mayor que el rea de EE I. Para esto observe que uno de los
a
tringulos DD I, EE I queda en el lado opuesto C con respecto a D E , si por
a
ejemplo, es el tringulo D DI entonces ste tendr rea mayor o igual al rea
a
e
aa
a
del tringulo EE I, luego la desigualdad se sigue. Ahora, muestre que el rea
a
a
r
r
2r 2

de (CD E ) es sen C , para esto ultimo note que CI = sen C y que D I = cos C ,
luego

2r 2
1
2r 2
=
(CD E ) = D E CI =
2r 2 .
C
C
2
sen C
2 sen 2 cos 2

Solucin 3.6 La clave est en notar que 2AX 3(AB + BX), que se puede
o
a
deducir aplicando el teorema de Ptolomeo (ejercicio 2.11) al cuadriltero c
a
clico

4.3 Soluciones a los problemas del cap


tulo 3

189

que resulta de pegar al tringulo ABX un tringulo equiltero AXO de lado


a
a
a
2

AX y luego observar que el dimetro de tal circunferencia es 3 AX, as se


a
tendr que AX(AB + BX) = AX BO AX
a

2
AX.
3

Por lo tanto

2AD = 2(AX + XD) 3(AB + BX) + 2XD

3(AB + BC + CX) + 3XD

3(AB + BC + CD).

Solucin 3.7 Tome el tringulo A B C de rea mxima de entre todos los


o
a
a
a
tringulos que se pueden formar con tres vrtices de los puntos dados; entonces
a
e
su rea satisface que (A B C ) 1. Construya otro tringulo ABC que tenga
a
a
B C como tringulo medial; ste tiene rea (ABC) = 4(A B C ) 4. En
aA
a
e
a
ABC se encuentran todos los puntos. En eferto, si algn punto Q est fuera
u
a
del tringulo ABC, estar en uno de los semiplanos determinados por los lados
a
a
y opuesto al semiplano donde est el tercer vrticie. Por ejemplo, si Q est en el
a
e
a
semiplano determinado por BC, y opuesto a donde est A, el tringulo QB C
a
a
es de rea mayor a A B C , lo cual es una contradiccin.
a
o
1
1
Solucin 3.8 Sea M = 1 + 2 + + n . Queremos probar que M es el valor
o
m
nimo deseado, resultado que se alcanza si hacemos x1 = x2 = = xn = 1.
Usando la desigualdad M GM A, se tiene que xk +(k 1) = xk +1+ +1
k
k

xk 1 1 = kxk , para toda k. Por lo tanto,


k

x1 +

x2 x3
xn
1
n1
2
+ 3 + + n x1 +x2 + +xn
= x1 +x2 + +xn n+M.
2
3
n
2
n

Por otro lado, la desigualdad entre la media armnica y la media aritmtica nos
o
e
lleva a
n
x1 + x2 + + xn
1
1
1 = 1.
n
x + x + + xn
1

Podemos concluir que la expresin dada es al menos n n + M = M . Como


o
hemos visto podemos encontrar M y es el m
nimo deseado.
Segunda Solucin. Aplique la desigualdad M GM A con pesos, a los nmeros
o
u
xj , con pesos
j

tj =

j
P1

, para obtener

xj
j
j

1
j

(x1 x2 ...xn )

1
P 1
j

1
.
j

190

Soluciones a los Ejercicios y Problemas

La ultima desigualdad se sigue de n n

1
x1

x1
n

1
xj

= n.

Solucin 3.9 Note que AF E y BDC son tringulos equilteros. Sean C y


o
a
a
F puntos fuera del hexgono y de manera que ABC y DEF sean tambin
a
e
tringulos equilteros. Como BE es la mediatriz del segmento AD, se sigue que
a
a
C y F son los reejados de C y F en la recta BE. Use ahora que AC BG y
EF DH son c
clicos, para concluir que AG + GB = GC y DH + HE = HF .
1
Solucin 3.10 El teorema de Leibniz garantiza que OG2 = R2 9 (a2 +b2 +c2 ).
o
abc
abc
Como rs = 4R , se tiene que 2rR = a+b+c . Luego, lo que hay que demostrar

es que abc

(a+b+c) (a2 +b2 +c2 )


,
3
3

para esto use M G M A.

Solucin 3.11 El lado izquierdo de la desigualdad se sigue de que


o

1
1 + x0 + x1 + + xi1 xi + + xn (1 + x0 + + xn ) = 1.
2
Para el lado derecho considere i = arcsen (x0 + + xi ), para i = 0, ..., n.
Note que

1 + x0 + + xi1 xi + + xn =

1 + sen i1

= cos i1 .

Slo resta ver ahora que


o
sen i sen i1 = 2 cos

sen i sen i1
cos i1

1 sen i1

< . Pero
2

i i+1
i + i+1
sen
< (cos i1 )(i i1 ).
2
2

Para probar la desigualdad use que el coseno es decreciente y que sen ,


para 0 . Luego
2
sen i sen i1
<
cos i1

i i1 = n 0 =

.
2

2
n
n
n
2
Solucin 3.12 Si
o
i=1 xi = 1 entonces 1 = ( i=1 xi ) =
i=1 xi +
2 i<j xi xj . Por lo tanto, la desigualdad que se pide demostrar es equivalente
a
n
1
x2
i

.
n1
1 ai
i=1

4.3 Soluciones a los problemas del cap


tulo 3

191

Use la desigualdad de Cauchy-Schwarz para ver que


2

xi
i=1

i=1

x2
i
1 ai

n
i=1

(1 ai ) .

Solucin 3.13 Note primero que n xn+1 (xn+1 xi ) = (n 1)x2 .


o
n+1
i=1
La desigualdad a demostrar se reduce a
n
i=1

xi (xn+1 xi )

n 1xn+1 .

Aplique Cauchy-Schwarz con ( x1 , . . . , xn ) y ( xn+1 x1 , . . . , xn+1 xn ).


Solucin 3.14 Recuerde que N tambin es punto medio del segmento que
o
e
une los puntos medios X, Y de las diagonales AC y BD. La circunferencia de
dimetro OM pasa por X y Y , ya que OX y OY son perpendiculares a las
a
diagonales respectivas, y ON es una mediana del tringulo OXY .
a
Solucin 3.15 La desigualdad de la derecha se deduce de wx+ xy + yz + zw =
o
(w + y)(x + z) = (w + y)2 0. Para la desigualdad de la izquierda, note que
|wx + xy + yz + zw| = |(w + y)(x + z)|
1
(w + y)2 + (x + z)2

2
w2 + x2 + y 2 + z 2 = 1.
Tambin puede usar la desigualdad de Cauchy-Schwarz para obtener
e
|wx + xy + yz + zw|2 (w2 + x2 + y 2 + z 2 )(x2 + y 2 + z 2 + w2 ) = 1.
Solucin 3.16 Para la desigualdad de la izquierda, reacomode as
o

an + a2 a1 + a3
an1 + a1
a1 a2 a3 a2
a1 an
+
++
=
+
+
+
++
+ ,
a1
a2
an
a2 a1 a2 a3
an a1
ahora utilice que

x
y

y
x 2.
a1 +a3
a2 +a4
a2 + a3

+a
+a
Llame Sn = ana1 2 +
o
+ + an1n 1 . Pruebe por induccin que
a
Sn 3n.
Primero para n = 3, se necesita ver que b+c + c+a + a+b 9. Si a = b = c,
a
b
c
entonces b+c + c+a + a+b = 6 y la desigualdad es cierta. Suponga ahora que
a
b
c

192

Soluciones a los Ejercicios y Problemas

a b c y que no son todos iguales, entonces hay tres casos: a = b < c,


a < b = c, a < b < c. Pero en cualquiera de ellos se tiene que a b y a < c.
Luego 2c = c + c > a + b y a+b < 2, y como a+b es entero positivo se tiene
c
c
que c = a + b.
b
b
Luego, b+c + c+a + a+b = a+2b + 2a+b + 1 = 3 + 2 a + 2 a . Como 2 a y 2 a
a
b
c
a
b
b
b
b
deben ser enteros positivos y como 2 a 2 a = 4 se tiene o bien que son 2 los
b
dos nmeros o bien uno 1 y el otro 4. Esto quiere decir que la suma es a lo ms
u
a
8, que es menor que 9, luego el resultado.
Continue con la induccin, suponga que Sn1 3(n1). Considere {a1 , . . . , an },
o
si todos son iguales, Sn = 2n y la desigualdad es verdadera. Suponga entonces
que hay al menos dos ai diferentes. Tome el mximo de las ai s; sus vecinos
a
(ai1 , ai+1 ) pueden ser iguales a este valor mximo, pero como hay entre los ai
a
dos diferentes, para algn mximo ai , se tiene que uno de sus vecinos es menor
u
a
que ai . Sin perder generalidad suponga que an es mximo y que alguno de sus
a
vecinos an1 o a1 es menor que an . Luego, como 2an > an1 + a1 se tiene que
+a
an1 +a1
< 2 y entonces an1n 1 = 1, por lo que an = an1 + a1 . Al sustituir
an
a
este valor de an en Sn , obtendr que
a
Sn =

an2 + an1 + a1 an1 + a1


an1 + a1 + a2 a2 + a3
+
+ +
+
=
a1
a2
an1
an1 + a1
1+

an1 + a2 a2 + a3
an2 + a1
+
+ +
+ 1 + 1.
a1
a2
an1

Como Sn1 3(n 1), se concluye que Sn 3n.


Solucin 3.17 Como el cuadriltero OBDC es c
o
a
clico, utilice el teorema de
DC
BD
Ptolomeo para demostrar que OD = R BC + BC , donde R es el circunradio de ABC. Por otro lado, como los tringulos BCE y DCA son semejana
tes, al igual que ABD y F BC, sucede que R BD + DC = R AD + AD .
BC
BC
FC
EB
CF
CF
Anlogamente, tenemos que OE = R BE + BE y OF = R BE + AD .
a
AD
CP
Multiplicando estas igualdades y aplicando M G M A obtendr el resultado.
a
Otra forma de demostrar lo anterior es utilizando inversin. Sean D , E y F los
o
puntos de interseccin de AO, BO y CO con los lados BC, CA y AB, respeco
tivamente. Invierta los lados BC, CA y AB con respecto a (O, R), obtendr los
a
circunc
rculos de los tringulos OBC, OCA y OAB, respectivamente. Luego,
a
OD OD = OE OE = OF OF = R2 . Si x = (ABO), y = (BCO) y
z = (CAO), se tiene que
z+x
AO
,
=

OD
y

BO
x+y
=

OE
z

CO
y+z
.
=

OF
x

4.3 Soluciones a los problemas del cap


tulo 3
Lo que implica, usando M G M A, que
OE OF 8R3 .

R3
OD OE OF

193
8, por lo que, OD

Solucin 3.18 Primero, observe que AY 2R y que ha AX, donde ha is


o
la longitud de la altura sobre BC. Luego, deduzca que
la
sen 2 A

AX
AY sen 2 A
ha
2R sen 2 A
1
sen A
ha
=
como
a sen A
a
2R

=
3

hb
hc
ha
a sen A b sen B c sen C

= 3
como ha = b sen C, hb = c sen A, hc = a sen B.
Solucin 3.19 Sin perder generalidad, podemos suponer x1 x2 xn .
o
Como 1 < 2 < < n se tiene, por la desigualdad del reacomodo (1.2),
A = x1 + 2x2 + + nxn nx1 + (n 1)x2 + + xn = B.
Luego, |A + B| = |(n + 1) (x1 + + xn )| = n + 1, por lo que A + B =
(n + 1). Ahora bien, si A + B = n + 1 se cumple que B n+1 A, y si
2
A + B = (n + 1) sucede que B n+1 A.
2
a
Suponga ahora que n+1 o n+1 estn entre B y A, ya que en caso contrario A
2
2
o B estar en el intervalo n+1 , n+1 . Luego, uno de los valores |A| o |B|
an
2
2
es menor o igual a n+1 y termina el problema.
2
Suponga por lo tanto, B n+1 < n+1 A.
2
2
Sea y1 , ..., yn una permutacin de x1 , ..., xn tal que 1y1 + 2y2 + + nyn = C
o
toma el valor ms grande con C n+1 . Tome i tal que y1 y2 yi
a
2
y yi > yi+1 , y considere
D = y1 + 2y2 + + iyi+1 + (i + 1)yi + (i + 2)yi+2 + + nyn

D C = iyi+1 + (i + 1)yi (iyi + (i + 1)yi+1 ) = yi yi+1 > 0.

Como |yi |, |yi+1 | n+1 , se tiene que D C = yi yi+1 n + 1; luego,


2
D C + n + 1 y entonces C < D C + n + 1 n+1 .
2
a
Por otro lado, D n+1 , pues C es la suma ms grande que es menor a n+1 .
2
2
Luego, n+1 D n+1 y as |D| n+1 .

2
2
2

194

Soluciones a los Ejercicios y Problemas

Solucin 3.20 Entre los nmeros x, y, z dos tienen el mismo signo (suponga
o
u
y
que son x y y), como c = z x + x es positivo, z es positivo.
y
Note que a + b c = 2xy , b + c a = 2yz , c + a b = 2zx son positivos.
z
x
y
Rec
procamente, si u = a + b c, v = b + c a y w = c + a b son
positivos entonces haciendo u = 2xy , v = 2yz , w = 2zx , podemos obtener que
z
x
y
a=

u+w
2

=x

y
z

z
y

, etc.

Solucin 3.21 Primero vea que un hexgono ABCDEF centralmente simtrio


a
e
co tiene lados opuestos paralelos. Esto garantiza que (ACE) = (BDF ) =
(ABCDEF )
. Ahora, si reejamos los vrtices del tringulo P QR con respecto al
e
a
2
centro de simetr del hexgono, obtenemos los puntos P , Q , R que forman
a
a
el hexgono centralmente simtrico P R QP RQ , inscrito en ABCDEF y de
a
e
a
rea 2(P QR).
Solucin 3.22 Sean X = 4 x3 , Xi = X x3 ; es claro que X = 1 4 Xi .
o
i
i=1
i=1 i
3
1
1
Por la desigualdad M G M A, 3 X1 3 x3 x3 x3 = x1 ; anlogamente, para los
a
2 3 4
1
otros
ndices y esto implica que X 4 xi .
i=1
Por la desigualdad de Tchebyshev se obtiene
x2 + x2 + x2 + x2 x1 + x2 + x3 + x4
x3 + x3 + x3 + x3
4
3
2
4
3
2
1
1

4
4
4
Gracias a la desigualdad M GM A se tienen
1 y por lo tanto, X i=1 xi .

x2 +x2 +x2 +x2


1
2
3
4
4

Solucin 3.23 Use la desigualdad de Cauchy-Schwarz con u =


o

x, y, z .
yv=
Solucin 3.24 Si = ACM y = BDM , entonces
o

(x1 x2 x3 x4 )2 =

y1
x1
z1
, ,
y
x
z

M AM B
M CM D

= tan tan

Ahora, use el hecho de que tan tan tan tan3 ++ ,


y + =
3
donde =
Podemos utilizar otro mtodo el cual usa el hecho de que la desigualdad es
e

equivalente a (M CD) 3 3(M AB) que a su vez es equivalente a h+l 3 3,


h
donde l es la longitud del lado del cuadrado y h es la longitud de la altura desde
M sobre AB. Ahora, encuentre la mxima h.
a

4.

4.

PM
Solucin 3.25 Primero note que P L + BM + P N = 1. Ahora, use el hecho de
o
AL
CN
que AL, BM y CN son menores que a.

4.3 Soluciones a los problemas del cap


tulo 3
Solucin 3.26 Como
o
PB
PA

QC
QA

PB
PA

= 1.

QC
QA

1
4

PB
PA

QC
QA

195
2

, es suciente probar que

Trace BB , CC paralelas a la mediana AA de tal forma que B y C estn


e
son semejantes, y tambin los tringulos
en P Q. Los tringulos AP G y BP B
a
e
a

e
AQG y CQC son semejantes, entonces P B = BB y QC = CC . Use sto
PA
AG
QA
AG
= BB + CC .
junto con el hecho de que AG = 2GA
Solucin 3.27 Sean el circunc
o
rculo del tringulo ABC y R su radio. Consia
dere la inversin en . Para cualquier punto P distinto de O, sea P su inverso.
o
El inverso del circunc
rculo de OBC es la recta BC, entonces A , el inverso de
1
A1 , es el punto de interseccin entre el rayo OA1 y BC. Como7
o
P Q =

R2 P Q
OP OQ

para dos puntos P , Q (distintos de O) con inversos P , Q , se tiene


R2 A A
AA
x+y+z
AA1
1
1
=
=
=
,
OA OA
OA1
OA
OA
y+z
1
1
donde x, y, z denotan las reas de los tringulos OBC, OCA y OAB, respeca
a
tivamente. Anlogamente, se tiene que
a
BB1
CC1
x+y+z
x+y+z
y
.
=
=
OB1
z+x
OC1
x+y
Luego,
BB1 CC1
AA1
+
+
= (x + y + z)
OA1
OB1 OC1
7

Vea [6], pg. 132 o [9], pg. 112.


a
a

1
1
1
+
+
y+z z+x x+y

9
.
2

196

Soluciones a los Ejercicios y Problemas

Para la ultima desigualdad, vea el ejercicio 1.44.

Solucin 3.28 Note que el rea del tringulo GBC es, (GBC) =
o
a
a
2(ABC)
aGL
. Anlogamente, GN = 2(ABC) .
a
2 , de donde GL =
3a
3c
Por lo tanto,
(GN L) =
=

(ABC)
3

GL GN sen B
4(ABC)2 sen B
=
2
18ac
(ABC)2 b2
(ABC) b2
4(ABC)2 b2
=
.
=
(18abc)(2R)
9 4R2
(9R abc )(4R)
4R

Anlogamente, (GLM ) =
a

(ABC) c2
94R2

y (GM N ) =

a2 + b2 + c2
4R2

(LM N )
1
=
(ABC)
9

(ABC) a2
.
94R2

Por lo tanto,

R2 OG2
.
4R2

La desigualdad de la derecha es ahora inmediata.


a
a
Para la otra desigualdad, note que OG = 1 OH. Como el tringulo es acutngu3
lo, H est dentro del tringulo y lo ms alejado que H est de O es R. Por lo
a
a
a
a
tanto,
1
1
R2 9 OH 2
R2 9 R2
4
2
(LM N )
=

= > .
2
2
(ABC)
4R
4R
9
27
Solucin 3.29 La funcin f (x) =
o
o
f (ab) + f (bc) + f (ca)
3

1
1+x

es convexa para x > 0. Luego,

ab + bc + ca
3
=
3
3 + ab + bc + ca
3
1
= ,
3 + a2 + b2 + c2
2

la ultima desigualdad se sigue de que ab + bc + ca a2 + b2 + c2 .

Tambin se puede partir de


e
1
1
1
9
9
3
+
+

= .
2 + b2 + c2
1 + ab 1 + bc 1 + ca
3 + ab + bc + ca
3+a
2
La primera desigualdad se sigue de la desigualdad (1.11) y la segunda se sigue
del ejercicio 1.27.
Solucin 3.30 Dena x = b + 2c, y = c + 2a, z = a + 2b. La desigualdad
o
deseada se transforma en
x y
+
y
x

y z
+
z y

x
z
+3
+
x z

y
z x
+ +
x y
z

15,

4.3 Soluciones a los problemas del cap


tulo 3

197

que se resuelve aplicando la desigualdad M G M A.


Otra forma de probar la desigualdad es la siguiente:
b
c
a2
b2
c2
(a + b + c)2
a
+
+
=
+
+

.
b + 2c c + 2a a + 2b
ab + 2ca bc + 2ab ca + 2bc
3(ab + bc + ca)
La desigualdad se sigue inmediatamente de la desigualdad (1.11). Slo falta
o
probar la desigualdad (a + b + c)2 3(ab + bc + ca), que es una consecuencia
de la desigualdad de Cauchy-Schwarz.
Solucin 3.31 Utilice la desigualdad (1.11) o bien utilice la desigualdad de
o
Cauchy-Schwarz con

a , b , c , d
y ( a + b, b + c, c + d, d + a).
a+b
b+c
c+d
d+a
Solucin 3.32 Sean x = b + c a, y = c + a b y z = a + b c. La semejanza
o
entre los tringulos ADE y ABC nos da que
a
per
metro de ADE
2x
DE
=
=
.
a
per
metro de ABC
a+b+c
x(y+z)
Luego, DE = x+y+z , es decir, la desigualdad es equivalente a ver que
x+y+z
. Utilice la desigualdad M G M A.
4

x(y+z)
x+y+z

Solucin 3.33 Tome F sobre AD con AF = BC y dena E el punto de


o
interseccin de BF con AC. Use la ley de los senos en los tringulos AE F ,
o
a
BCE y BDF , para obtener
AF sen F
sen E
sen F
BD
AE
AE
=

=
=
=
C

E
sen E
BC sen B
sen B
FD
EC
entonces, E = E.
Considere despus, a G sobre BD con BG = AD y a H la interseccin de GE
e
o
con la paralela a BC por A. Use que los tringulos ECG y EAH son semejantes,
a
y el teorema de Menelao en el tringulo CAD, con transversal EF B, para
a
concluir que AH = DB. Tendr ahora que BDAH es un paralelogramo, que
a
BH = AD y que BHG es issceles con BH = BG = AD > BE.
o
Solucin 3.34 Observe que ab + bc + ca 3abc si y slo si
o
o
Como
1 1 1
(a + b + c)
+ +
9,
a b
c

1
a

1
b

1
c

3.

198

Soluciones a los Ejercicios y Problemas

debe tenerse que (a + b + c) 3. Entonces


3(a + b + c) (a + b + c)2
=

a3/2 a1/2 + b3/2 b1/2 + c3/2 c1/2

a3 + b3 + c3

1 1 1
+ +
a b
c

3 a3 + b3 + c3 .
Solucin 3.35 Tome yi =
o
desigualdad, es decir,

xi
n1 ,

para i = 1, 2, . . . , n, y suponga falsa la

1
1
1
+
+ +
> n 1.
1 + y1 1 + y2
1 + yn
Luego
1
1 + yi

>

j=i

1
1 + yj

(n 1) n1

=
j=i

yj
1 + yj

y1 yi yn

(1 + y1 ) (1 + yi ) (1 + yn )

donde y1 yi yn es el producto de las ys salvo la yi . Entonces

n
i=1

1
y1 yn
,
> (n 1)n
1 + yi
(1 + y1 ) (1 + yn )

y esto ultimo lleva a que 1 > x1 xn , que es una contradiccin.

o
Solucin 3.36 Use la desigualdad de Cauchy-Schwarz con las colecciones
o

x1
xn
x1 y1 , . . . , xn yn para garantizar que
y
y1 , . . . ,
yn
(x1 + + xn )2 =

Ahora use la condicin


o
desigualdad util (1.11).

2
x1
xn
x1 y 1 + +
xn y n
y1
yn
xn
x1
+ +
(x1 y1 + + xn yn ) .
y1
yn

xi y i

xi . Tambin puede resolverse usando la


e

4.3 Soluciones a los problemas del cap


tulo 3

199

Solucin 3.37 Como abc = 1, tenemos que (a 1)(b 1)(c 1) = a + b +


o
1
a
c ( a + 1 + 1 ). Anlogamente,
b
c
(an 1)(bn 1)(cn 1) = an + bn + cn

1
1
1
+ n+ n
n
a
b
c

La armacin se sigue del hecho de que el lado izquierdo de cada una de las
o
identidades tiene el mismo mismo signo.
Solucin 3.38 Haga la demostracin por induccin en n. El caso n = 1 es
o
o
o
claro. Suponga el resultado cierto para n y vea que es vlida la desigualdad para
a
n + 1.

Como se tiene que n < n2 + i < n + 1, para i = 1, 2, ..., 2n, sucede que
n2 + i =

n2 + i n <

n2 + i +

i
2n

n=

i
.
2n

Luego
(n+1)2

(n+1)2

n2

j +
j=1

j=1

j
j=n2 +1

n2 1
1

+
2
2n

2n

i
i=1

(n + 1)2 1
.
2

Solucin 3.39 Vea la contrarrec


o
proca, esto es, x3 + y 3 > 2, implica que
2

x2 +y 3 < x3 +y 4 . La desigualdad entre medias potenciales x +y 3 x


2
implica que

3
x2 + y 2 (x3 + y 3 )2/3 2 < (x3 + y 3 )2/3 (x3 + y 3 )1/3 = x3 + y 3 .

3 +y 3

Luego x2 x3 < y 3 y 2 y 4 y 3 . La ultima desigualdad se debe al hecho

2 (y 1)2 0.
que y
Segunda Solucin. Como (y 1)2 0, se tiene que 2y y 2 + 1, luego
o
2y 3 y 4 + y 2 . Luego, x3 + y 3 x3 + y 4 + y 2 y 3 x2 + y 2 , ya que
x3 + y 4 x2 + y 3 .
Solucin 3.40 La desigualdad es equivalente a
o
(x0 x1 ) +

1
1
+ (x1 x2 ) + + (xn1 xn ) +
2n.
(x0 x1 )
(xn1 xn )

200

Soluciones a los Ejercicios y Problemas

Solucin 3.41 Como


o
que

a+3b
4

ab3 ,

b+4c
5

bc4 y
11

c+2a
3

19

ca2 , tenemos

17

(a + 3b)(b + 4c)(c + 2a) 60a 12 b 20 c 15 .


2

Ahora, muestre que c 15 a 12 b 20 o, equivalentemente, que c8 a5 b3 .


Solucin 3.42 Se tiene una equivalencia entre las siguientes desigualdades,
o
7(ab + bc + ca) 2 + 9 abc

7(ab + bc + ca)(a + b + c) 2(a + b + c)3 + 9 abc

a2 b + a b2 + b2 c + b c2 + c2 a + c a2 2(a3 + b3 + c3 ).
Para ver la validez de la ultima desigualdad, use la desigualdad del reacomodo

o la desigualdad de Tchebyshev.
Solucin 3.43 Sea E la interseccin de AC y BD. Los tringulos ABE y
o
o
a
DCE son semejantes, entonces
|AB|
|AB CD|
=
.
|AC BD|
|AE EB|
|AB|
Usando la desigualdad del tringulo en ABE, se tiene que |AEEB| 1 y
a
concluya entonces que |AB CD| |AC BD|. Anlogamente, |ADBC|
a
|AC BD|.

Solucin 3.44 Muestre, que siempre se tiene que a1 + +aj j(j+1) an , para
o
2n
j n, de la siguiente forma. Primero pruebe que la desigualdad es vlida para
a
j = n, es decir, a1 + + an n+1 an ; use el hecho de que 2(a1 + + an ) =
2
a1 ++a
(a1 + an1 ) + + (an1 + a1 ) + 2an . Luego, pruebe que si bj = 1++j j ,
o
entonces b1 b2 bn an (para probar por induccin que bj bj+1 ,
n
aj+1
necesitamos probar que, bj j+1 , lo cual se sigue de la primera parte para
n = j + 1).
Damos otra demostracin de a1 + + aj j(j+1) an , nuevamente usando
o
2n
induccin. Es claro que
o
a1 a1
a1 +

a2
a1 a1 a2
a2 a2
=
+
+

+
= a2 .
2
2
2
2
2
2

4.3 Soluciones a los problemas del cap


tulo 3

201

Ahora bien, supongamos que la armacin es vlida para n = 1, . . . , j, es decir,


o
a
a1 a1
a2
a2
a1 +
2
.
.
.
.
.
.
aj
a2
a1 +
+ +
aj .
2
j
Sumando todas las desigualdades anteriores se tiene
ja1 + (j 1)

aj
a2
+ +
a1 + + aj .
2
j

Sumando de ambos lados la identidad


a1 + 2

aj
a2
+ + j = aj + + a1
2
j

se tiene que
(j+1) a1 +

aj
a2
+ +
2
j

(a1 +aj )+(a2 +aj1 )+ +(aj +a1 ) jaj+1 .

Luego,
a1 +

aj
j
a2
+ +

aj+1 .
2
j
j+1

j+1
Finalmente, sumando j+1 en ambos lados de la desigualdad, terminamos el
ultimo paso de la prueba por induccin.

o
Ahora,

a1 +

an
a2
+ +
2
n

1
(a1 + + an ) +
n
1
n

n(n + 1)
an
2n

n1
j=1
n1

+
j=1

1
1

j
j+1

(a1 + + aj )

j(j + 1)
1
an = an .
j(j + 1) 2n

202

Soluciones a los Ejercicios y Problemas

Solucin 3.45
o

1in

xi

x2 + 2
i
1in

= 8

1in

1in

= 8

1i<jn

1i<jn

x2 2
i

xi xj

1i<jn

x2
i

1i<jn

xi xj (x2
1

xi xj

xi xj

+ + x2 )
n

xi xj (x2 + x2 ).
i
j
1i<jn

Para la primera desigualdad se aplica la desigualdad M G M A.


Para determinar cundo la igualdad ocurre, note que en el ultimo paso dos de
a

las xi deben ser diferentes de cero y las n 2 restantes iguales a cero; adems,
a
en el paso en el que se us la desigualdad M G M A, las xi diferentes de cero
o
deben ser iguales. Se puede comprobar que, en tal caso, la constante C = 1 es
8
la m
nima.

Solucin 3.46 Sea 3 a = x y 3 b = y, necesita mostrar que (x2 + y 2 )3


o
2(x3 + y 3 )2 , para x, y > 0.
Por la desigualdad M G M A tenemos
3x4 y 2 x6 + x3 y 3 + x3 y 3

y
3x2 y 4 y 6 + x3 y 3 + x3 y 3 ,

con igualdad si y slo si x6 = x3 y 3 = y 6 o, equivalentemente, si y slo si x = y.


o
o
Sumando estas dos desigualdades y sumando x6 + y 6 a ambos lados, obtendr
a
x6 + y 6 + 3x2 y 2 (x2 + y 2 ) 2(x6 + y 6 + 2x3 y 3 ).
La igualdad ocurre cuando x = y, esto es, cuando a = b.
Solucin 3.47 Denote por S el lado izquierdo de la desigualdad. Como a
o
b c y x y z, por la desigualdad del reacomodo obtiene bz + cy by + cz,
entonces
(by + cz)(bz + cy) (by + cz)2 2((by)2 + (cz)2 ).

4.3 Soluciones a los problemas del cap


tulo 3

203

Dena = (ax)2 , = (by)2 , = (cz)2 , luego,


a2 x2

a2 x2

=
.
(by + cz)(bz + cy)
2((by)2 + (cz)2 )
2( + )
Sumando las otras dos desigualdades similares, obtendr
a
S

1
2

+
+
+ + +

Para terminar la demostracin use la desigualdad de Nesbitt.


o
Solucin 3.48 Si XM es una mediana en el tringulo XY Z, entonces,
o
a
como resultado de utilizar el teorema de Stewart, XM 2 = 1 XY 2 + 1 XZ 2
2
2
1
2
4 Y Z . Sustituya (X, Y, Z, M ) por (A, B, C, P ), (B, C, D, Q), (C, D, A, R) y
(D, A, B, S) en esta frmula y sumando las cuatro ecuaciones obtenidas, se
o
tiene una quinta ecuacin. Multiplicando ambos lados de la quinta ecuacin
o
o
por 4, encontrar que el lado izquierdo de la desigualdad deseada es igual a
a
AB 2 + BC 2 + CD2 + DA2 + 4(AC 2 + BD2 ). Luego, es suciente mostrar que
AC 2 + BD2 AB 2 + BC 2 + CD2 + DA2 . Esta desigualdad es conocida como
la desigualdad del paralelogramo. Para demostrarla, sea O un punto arbitrario
en el plano y, para cada punto X sea x el vector de O a X. Desarrolle cada
uno de los trminos en AB 2 + BC 2 + CD2 + DA2 AC 2 BD2 , por ejemplo,
e
escribiendo
AB 2 = |a b|2 = |a|2 2a b + |b|2
y encontrar que esta expresin es igual a
a
o
|a|2 + |b|2 + |c|2 + |d|2 2(a b + b c + c d + d a a c b d)
= |a + c b d|2 0,
con igualdad si y slo si a + c = b + d, esto es, si el cuadriltero ABCD es un
o
a
paralelogramo.
Solucin 3.49 Dena A = x2 + y 2 + z 2 , B = xy + yz + zx, C = x2 y 2 + y 2 z 2 +
o
2 x2 , D = xyz. Entonces 1 = A + 2B, B 2 = C + 2xyz(x + y + z) = C + 2D, y
z
x4 + y 4 + z 4 = A2 2C = 4B 2 4B + 1 2C = 2C 4B + 8D + 1. Entonces,
la expresin del centro es igual a
o
3 2A + (2C 4B + 8D + 1) = 2 + 2C + 8D 2,
con igualdad si y slo si dos de las variables x, y, z son cero.
o

204

Soluciones a los Ejercicios y Problemas

Ahora, la expresin de la derecha es igual a 2 + B + D. Luego, se tiene que


o
demostrar que 2C + 8D B + D o B 2B 2 3D 0. Utilice la desigualdad
de Cauchy-Schwarz para obtener A B, luego B(1 2B) = BA B 2 .
Entonces es suciente demostrar que B 2 3D = C D 0. Pero C
xyyz + yzzx + zxxy = D lo cual se deduce de la desigualdad de CauchySchwarz.
Solucin 3.50 Suponga que a =
o
equivalente a
x
z
1+
y
y

x
y,

b =

y
x
1+
z
z

y
z

, c =

z
x.

La desigualdad es

z
y
1,
1+
x
x

que se reescribe como (x + z y)(x + y z)(y + z x) xyz. Esta ultima es

vlida si x, y, z son las longitudes de los lados de un tringulo. Vea el ejemplo


a
a
2.2.3.
Falta considerar el caso en que algunas de las variables u = x + z y, v =
x + y z, w = y + z x sean negativas. Si una o tres de ellas son negativas
entonces el lado izquierdo es negativo y la desigualdad es evidente. Si dos de los
valores u, v, w son negativos, por ejemplo u y v, entonces tambin u + v = 2x
e
es negativa; pero x > 0, pero esta ultima situacin no es posible.

o
Solucin 3.51 Note primero que abc a + b + c implica que (abc)2
o
(a + b + c)2 3(a2 + b2 + c2 ), donde la ultima desigualdad se sigue de la

desigualdad (1.11).
Por la desigualdad M GM A, a2 +b2 +c2 3 3 (abc)2 , luego (a2 +b2 +c2 )3
33 (abc)2 . Por lo tanto (a2 + b2 + c2 )4 32 (abc)4 .
Solucin 3.52 Por la desigualdad M G M A,
o
(a + b)(a + c) = a(a + b + c) + bc 2 abc(a + b + c).
Segunda Solucin. Dena x = a + b, y = a + b, z = b + c, tendr que x, y, z
o
a
son las longitudes de los lados del tringulo XY Z, ya que a, b, c son positivos.
a
Entonces, por la frmula del rea del tringulo de la seccin 2.2 la desigualdad
o
a
a
o
xy
es equivalente a 2 (XY Z) . Ahora, use que el rea del tringulo con lados
a
a
de longitud x, y, z es menor o igual a xy .
2
Solucin 3.53 Como xi 0, entonces xi 1 1. Luego, puede usar la
o
desigualdad de Bernoulli para toda i, y obtener
(1 + (xi 1))i 1 + i(xi 1).

4.3 Soluciones a los problemas del cap


tulo 3

205

Sumando estas desigualdades para 1 i n, obtendr el resultado.


a
Solucin 3.54 Restando 2, se obtiene que las desigualdades son equivalentes
o
a
(a + b c)(a b + c)(a + b + c)
1.
0<
abc
La desigualdad de la izquierda es ahora obvia. La desigualdad de la derecha es
el ejemplo 2.2.3.
4/3

a
a4/3 +b4/3 +c4/3 , ser claro como
Solucin 3.55 Si muestra que a2a
o
a
+8bc
obtener el resultado. Esta ultima desigualdad es equivalente a

a4/3 + b

4/3

4/3

+c

a2/3 (a2 + 8bc).

Aplique la desigualdad M G M A en cada factor de


a4/3 + b

4/3

4/3

+c

a4/3

4/3

= b

4/3

+c

a4/3 + a4/3 + b

4/3

4/3

+c

Otra forma de resolver el problema es la siguiente, considere la funcin f (x) =


o
3
1
; esta funcin es convexa para x > 0 (f (x) =
o
> 0). Para 0 < a, b,
5
x
a

c < 1, con a + b + c = 1, se tiene que

4 x
c
b
1
+
. Aplicando
y
z
ax+by+cz
2 + 8ab (antes multiplique por un
c

esto a x = a2 + 8bc, y = b2 + 8ca y z =


factor para tener la condicin a + b + c = 1), obtendr
o
a

a
b
c
1
+
+

.
a2 + 8bc
b2 + 8ca
c2 + 8ab
a3 + b3 + c3 + 24abc

Use tambin que,


e
(a + b + c)3 = a3 + b3 + c3 + 3(a2 b + a2 c + b2 a + b2 c + c2 a + c2 b) + 6abc
a3 + b3 + c3 + 24abc.
Solucin 3.56 Utilizando la desigualdad de Cauchy-Schwarz
o
x
con ai = 1, bi = 1+x2 +x2i++x2 , se tiene que
1

x1
x2
xn
n
2 + 1 + x2 + x2 + + 1 + x 2 + + x 2
1 + x1
n
1
1
2
Luego, es suciente demostrar que
1

1
.
x2
1

a2
i

ai bi
b2 .
i

b2 < 1. Para i = 1, use que b2


1
i

x2
1
1+x2
1

b2 ,
i

206

Soluciones a los Ejercicios y Problemas

Observe que para i 2,


b2 =
i

xi
2 + + x2
1 + x1
i

x2
i
(1 + x2 + + x2 )2
1
i
x2
i
(1 + x2 + + x2 )(1 + x2 + + x2 )
1
1
i1
i
1
1

.
(1 + x2 + + x2 ) (1 + x2 + + x2 )
1
1
i1
i

x2

1
1
Para i = 1, use que b2 1+x2 = 1 1+x2 .
1
1
1
Sume las desigualdades, la suma de la derecha es telescpica, por lo que,
o
n

b2 =
i
i=1

xi
2 + + x2
1 + x1
i

1+

x2
1

1
< 1.
+ + x2
n

Solucin 3.57 Como slo existen dos valores posibles para , , , los tres
o
o
deben ser iguales, o dos iguales y uno diferente de los otros dos. Por lo tanto,
hay dos casos a considerar.
(1) = = . En este caso, se tiene que a + b + c = 0, y por lo tanto
a3 + b3 + c3
abc

=
=

a3 + b3 (a + b)3
ab(a + b)

(a + b)2 a2 + ab b2
ab

3ab
ab

= 9.

(2) Sin prdida de generalidad, puede suponer que = , = , entonces


e
c=a+b y
a3 + b3 + c3
abc

=
=

a3 + b3 + (a + b)3
(a + b)2 + a2 ab + b2
=
ab(a + b)
ab
2 + 2b2 + ab
a b
2a
=2
+
+ 1.
ab
b a

Si a y b tienen el mismo signo, vea que esta expresin no es menor que 5, y su


o
cuadrado es, por lo tanto, no menor a 25. Si los signos de a y b no son el mismo,
2
b
b
b
se tiene a + a 2, por lo tanto 2 a + a + 1 3 y 2 a + a + 1 9.
b
b
b
Entonces, el valor posible ms pequeo es 9.
a
n
1
1
Solucin 3.58 Use la desigualdad M G M A, para ver que b(a+b) + c(b+c) +
o

3
3
1
abc, Y = 3 (a + b)(b + c)(c + a). Use M G M A
a(c+a) XY , donde X =

4.3 Soluciones a los problemas del cap


tulo 3
nuevamente, para obtener que X
27
1
2 (a+b+c)2 .

207

y Y 2 a+b+c . Luego,
3

a+b+c
3

3
XY

Solucin 3.59 La desigualdad es equivalente a a4 +b4 +c4 a2 bc+b2 ca+c2 ab,


o
que se sigue directamente del teorema de Muirhead, ya que [4, 0, 0] [2, 1, 1].
Segunda Solucin.
o
b3
c3
a3
+
+
bc ca ab

a4
b4
c4
+
+
abc abc abc
(a2 + b2 + c2 )2

3abc
a + b + c 3 (a + b + c)
(a + b + c)4
=

27abc
3
abc
a+b+c
(abc)
= a + b + c.
abc
=

En las primeras dos desigualdades aplicamos la desigualdad (1.11), y en la ultima

desigualdad aplicamos la desigualdad M G M A.


2
x
Solucin 3.60 Considere a f (x) como f (x) = 1x . Como f (x) = (1x)3 > 0,
o
f (x) es convexa. Usando la desigualdad de Jensen se obtiene f (x) + f (y) +
f (z) 3f ( x+y+z ). Pero f es creciente para x < 1, y la desigualdad M G M A
3

le ayudar a probar que x+y+z 3 xyz, se tiene f ( x+y+z ) f ( 3 xyz).


a
3
3

Solucin 3.61
o
1
a
+
b+c 2

1
b
+
c+a 2

1
c
+
a+b 2

es equivalente a (2a + b + c)(2b + c + a)(2c + a + b) 8(b + c)(c + a)(a + b).


Ahora, observe que (2a + b + c) = (a + b + a + c) 2 (a + b)(c + a).
Solucin 3.62 La desigualdad del problema es equivalente a la siguiente deso
igualdad
(a + b c)(a + b + c) (b + c a)(b + c + a) (c + a b)(c + a + b)
+
+
9,
c2
a2
b2
2

la cual a su vez es equivalente a (a+b) + (b+c) + (c+a) 12. Como (a + b)2


c2
a2
b2
4ab, (b + c)2 4bc y (c + a)2 4ca, se tiene que
(a + b)2 (b + c)2 (c + a)2
4ab 4bc 4ca
3 (ab)(bc)(ca)
+
+
2 + 2 + 2 12
= 12.
2
2
2
c
a
b
c
a
b
c2 a2 b2

208

Soluciones a los Ejercicios y Problemas

Solucin 3.63 Por la desigualdad M G M A, x2 + x + x 3x. Sumando


o

desigualdades similares para y, z, se obtiene x2 + y 2 + z 2 + 2( x + y + z)


3(x + y + z) = (x + y + z)2 = x2 + y 2 + z 2 + 2(xy + yz + zx).

1
Solucin 3.64 Multiplicando por abc la igualdad 1 = a + 1 + 1 , se tiene
o
b
c

ab
bc
ca
abc =
c + ab
c +
a +
b . Entonces, es suciente demostrar que

c+ ab . Elevando al cuadrado la ultima desigualdad es equivalente a c+ab

ab
1
c + c + 2 ab o c + ab c + ab(1 a 1 ) + 2 ab o a + b 2 ab.
b
Solucin 3.65 Como (1 a)(1 b)(1 c) = 1 (a + b + c) + ab + bc + ca abc
o
y como a + b + c = 2, la desigualdad es equivalente a
1
.
27
Pero a < b + c = 2 a implica que a < 1 y, anlogamente, b < 1 y c < 1,
a
entonces la desigualdad de la izquierda es verdadera. La otra desigualdad se
sigue de la desigualdad M G M A.
0 (1 a)(1 b)(1 c)

Solucin 3.66 Es posible construir otro tringulo AA1 M , de lados AA1 , A1 M ,


o
a
M A de longitud igual a las medianas ma , mb , mc .

B1

C1

A1

3
Adems, (AA1 M ) = 4 (ABC). Entonces la desigualdad que se tiene que dea
mostrar es,

3
1
1
3
1
+
.
+

ma mb mb mc mc ma
4 (AA1 M )
Ahora, la ultima desigualdad ser verdadera si la siguiente desigualdad se sa
a
tisface para las longitudes de los lados de un tringulo a, b, c y rea S, es
a
a
decir,

1
1
1
3 3
+
+

,
ab bc ca
4S

4.3 Soluciones a los problemas del cap


tulo 3
la cual es equivalente a

4 3S

209

9abc
a+b+c

que a su vez es el ejemplo 2.4.6.


1
ab

Solucin 3.67 Sustituya cd =


o
desigualdad se convierte en

y da =

1
bc ,

entonces el lado izquierdo de la

1 + ab
1 + bc
1 + bc
1 + ab
+
+
+
1+a
ab + abc
1+b
bc + bcd
= (1 + ab)

1
1
+
1 + a ab + abc
1
x

Ahora, use la desigualdad

1
y

+ (1 + bc)

4
x+y ,

1
1
+
1 + b bc + bcd

(4.8)
.

para obtener

4
4
+ (1 + bc)
1 + a + ab + abc
1 + b + bc + bcd
1 + ab
1 + bc
+
1 + a + ab + abc 1 + b + bc + bcd
1 + ab
a + abc
+
= 4.
1 + a + ab + abc a + ab + abc + abcd

(Lado izquierdo) (1 + ab)


= 4
= 4

Solucin 3.68 Por el teorema de Stewart se tiene que


o
2
la = bc 1

a
b+c

1
bc
((b + c)2 a2 ) ((b + c)2 a2 ).
2
(b + c)
4

Por la desigualdad de Cauchy-Schwarz se sigue que


2
2
2
(la + lb + lc )2 3(la + lb + lc )
3

((a + b)2 + (b + c)2 + (c + a)2 a2 b2 c2 )


4
3
=
(a + b + c)2 .
4

Solucin 3.69 Como


o

1
1a

1
b+c

la desigualdad es equivalente a

1
1
2
2
2
1
+
+

+
+
.
b+c c+a a+b
2a + b + c 2b + a + c 2c + a + b

210

Soluciones a los Ejercicios y Problemas

Ahora, use que


2

1
x

1
y

4
x+y ,

para ver que,

1
1
1
+
+
b+c c+a a+b

4
4
4
+
+
a + b + 2c b + c + 2a c + a + 2b

lo cual muestra la desigualdad.


Solucin 3.70 Podemos suponer que a b c. Entonces c < a + b y
o

n
n
n

2
2
2
n
=
(a + b + c) >
(2c) = 2cn n bn + cn .
2
2
2
Como a b, se tiene que
b+

a
2

a
= bn + nbn1 + otros trminos positivos
e
2
n
> bn + abn1 bn + an .
2

Anlogamente, ya que a c, se tiene (c + a )n > cn + an ; por lo tanto


a
2

n
1
1
1
2
a
a
n
n n
n
n n
n
n n
+c+
< b+ +
(a + b ) + (b + c ) + (c + a )
2
2
2
n
n
2
2
=1+
.
= a+b+c+
2
2
Segunda Solucin. Recuerde que, a, b, c son las longitudes de los lados de un
o
tringulo si y slo si existen nmeros positivos x, y, z con a = y + z, b = z + x,
a
o
u
1
c = x + y. Como a + b + c = 1, se tiene que x + y + z = 2 .
Ahora, use la desigualdad de Minkowski
1
m

n
m

(xi + yi )
i=1

1
m

xm
i

1
m

n
m
yi

i=1

i=1

para obtener
1

(an + bn ) n = ((y + z)n + (z + x)n ) n (xn + y n ) n + (2z n ) n < c +

2z.

n
2y. Por lo tanto,

1
1
1
2
n
n
n n
n
n n
n
n n
(a + b ) + (b + c ) + (c + a ) < a + b + c + 2(x + y + z) = 1 +
.
2
1

Anlogamente, (bn + cn ) n < a +


a

2x y (cn + an ) n < b +

4.3 Soluciones a los problemas del cap


tulo 3

211

Solucin 3.71 Note primero que si restringe las sumas con i < j, entonces
o
ellas suman la mitad. La suma de la izquierda est elevada al cuadrado y la
a
suma de la derecha no, entonces la desigualdad deseada con sumas restringidas
a i < j, tiene (1/3), en lugar de (2/3), en el lado derecho.
Considere las sumas de todos los |xi xj |, con i < j. El trmino x1 aparece
e
en (n 1) trminos con signo negativo, x2 aparece en un trmino con signo
e
e
positivo y en (n 2) trminos con signo negativo, etc. Entonces, se obtiene que
e
(n 1)x1 (n 3)x2 (n 5)x3 + (n 1)xn =

(2i 1 n)xi .

Ahora, puede usar la desigualdad Cauchy-Schwarz para mostrar que el cuadrado


de esta suma es menor que
x2 (2i 1 n)2 .
i
Analizando la suma del otro lado de la desigualdad deseada, observe inmediatamente que es n x2 ( xi )2 . Nos gustar desaparecer el segundo trmino,
a
e
i
pero esto es fcil, ya que si suma h a cada xi , las sumas en la desigualdad
a
deseada no se afectan, ya que ellas usan slo diferencias de xi . Luego, puede
o
escoger h tal que
xi sea cero. Luego, la demostracin se termina si se puede
o
2
2 = n(n 1) ,
mostrar que (2i 1 n)
3
(2i 1 n)2 = 4
=
=
=

i2 4(n + 1)

i + n(n + 1)2

2
n(n + 1)(2n + 1) 2n(n + 1)2 + n(n + 1)2
3
1
n(n + 1)(2(2n + 1) 6(n + 1) + 3(n + 1))
3
1
n(n2 1).
3

Con lo que se obtiene la desigualdad que se quer


a.
Segunda Solucin. La desigualdad es del tipo de la desigualdad de Cauchyo
Schwarz, y como el problema pide que ser igualdad cuando x1 , x2 , ... , xn
a
sea una progresin aritmtica, es decir, cuando xi xj = r(i j), con r > 0,
o
e
entonces considere la siguiente desigualdad, ya garantizada por la desigualdad
de Cauchy-Schwarz,

i,j

|i j| |xj xj |

i,j

(xi xj )2

i,j

(i j)2 .

Aqu est asegurado que la igualdad se da si y slo si (xi xj ) = r(i j), con
a
o
r > 0.

212

Soluciones a los Ejercicios y Problemas

Como
i,j

(i j)2 = (2n 2) 12 + (2n 4) 22 + + 2 (n 1)2 =

Bastar mostrar que


a
i,j

|i j| |xi xj | =

n
2

i,j

n2 (n2 1)
.
6

|xi xj |. Para ver que esto

ultimo sucede, compare el coeciente de xi en cada lado. En el lado izquierdo

se tiene
(i 1) + (i 2) + + (i (i 1)) ((i + 1) i) + ((i + 2) i) + + (n i)) =
(i 1)i (n i)(n i + 1)
n(2i n 1)

=
.
2
2
2
El coeciente de xi en el lado derecho es,

n
n
n(2i n 1)
1+
1 = ((i 1) (n i)) =
.
2
2
2
=

i<j

j>i

Como son iguales se termina la prueba.

Solucin 3.72 Sean xn+1 = x1 y xn+2 = x2 . Dena


o
ai =
Es claro que

xi
xi+1

bi = xi + xi+1 + xi+2

xi = 3.

bi = 3

ai = 1,

i=1

i=1

i=1

i {1, ..., n}.

La desigualdad es equivalente a
n
i=1

ai
n2

.
bi
3

Usando la desigualdad M G M A, puede deducir que


1
n

n
i=1

bi

b1 b n

b1 b n

n
1
.
3
b1 b n

Por otra parte y usando nuevamente la desigualdad M G M A, se obtiene


n
i=1

n a a
an
n
n2
a1
ai
1
n
n
.
n

=n
=

n
n
bi
b1
bn
3
b1 b n
b1 b n

4.3 Soluciones a los problemas del cap


tulo 3

213

1
Solucin 3.73 Para cualquier a nmero real positivo a + a 2, con igualdad
o
u
si y slo si a = 1. Como los nmeros ab, bc y ca son no-negativos, se tiene
o
u

P (x)P

1
1
+b +c
x2
x
1
1
= a2 + b2 + c2 + ab x +
+ bc x +
x
x

1
x

= (ax2 + bx + c) a

+ ca x2 +

1
x2

a2 + b2 + c2 + 2ab + 2bc + 2ca = (a + b + c)2 = P (1)2 .


La igualdad se tiene si y slo si x = 1 o ab = bc = ca = 0, que por la condicin
o
o
a > 0, implica que b = c = 0. Consecuentemente, para cualquier nmero real
u
positivo x, se tiene
1
P (x)P
(P (1))2 ,
x
con igualdad si y slo si x = 1 o b = c = 0.
o
Segunda Solucin. Use la desigualdad de Cauchy-Schwarz para obtener
o
P (x)P

1
x

1
1
+b +c
2
x
x

2
2
2
a
( ax) + ( bx) + ( c)
x

= (ax2 + bx + c) a
=

Solucin 3.74
o

a
b
ax
+ bx + c c
x
x

2
+ ( c)

= (a + b + c)2 = (P (1))2 .

a2 (b + c) + b2 (c + a) + c2 (a + b)
3

(a + b)(b + c)(c + a)
4

3
a2 b + a2 c + b2 c + b2 a + c2 a + c2 b

2 b + a2 c + b2 c + b2 a + c2 a + c2 b
2abc + a
4
2
2
2
2
2
2
a b + a c + b c + b a + c a + c b 6abc

[2, 1, 0] [1, 1, 1].

La ultima desigualdad se sigue del teorema de Muirhead.

Segunda Solucin. Use la desigualdad (1.11) y la desigualdad de Cauchyo


Schwarz.

214

Soluciones a los Ejercicios y Problemas

Solucin 3.75 Use la desigualdad M G M A en cada denominador, para


o
obtener
1
1
1
1
1
1
+
+

+
+
.
1 + 2ab 1 + 2bc 1 + 2ca
1 + a2 + b2 1 + b2 + c2 1 + c2 + a2
Ahora, usando la desigualdad (1.11), se obtiene
1
1
1
(1 + 1 + 1)2
9
+
+

=
= 1.
2 + b2 1 + b2 + c2 1 + c2 + a2
2 + b2 + c2 )
1+a
3 + 2(a
3+23
Solucin 3.76 La desigualdad es equivalente a cada una de las siguientes
o
desigualdades
x4 + y 4 + z 4 + 3(x + y + z) (x3 z + x3 y + y 3 x + y 3 z + z 3 y + z 3 x)
x3 (x + y + z) + y 3 (x + y + z) + z 3 (x + y + z) + 3(x + y + z) 0
(x + y + z)(x3 + y 3 + z 3 3xyz) 0.

La identidad (1.9) muestra que la ultima desigualdad es equivalente a

2 ((x y)2 + (y z)2 + (z x)2 ) 0.


y + z)

1
2 (x

Solucin 3.77 Sean O y I el circuncentro y el incentro de un tringulo


o
a
acutngulo ABC, respectivamente. Los puntos O, M , X son colineales y, OCX
a
y OM C son tringulos rectngulos semejantes. De donde se tiene
a
a
OM
OC
=
.
OX
OC
OA
Como OC = R = OA, se tiene OM = OX . Luego, OAM y OXA son semeOA
OM
AM
jantes, entonces AX = R .
Ahora es suciente mostrar que OM r. Compare los ngulos OBM y
a
IBM . Como ABC es acutngulo, O y I son puntos interiores de ABC.
a
Ahora, se tiene que OBM = A = 1 (A + B + C) A =
2
2
1
(B + C A) B = IBM , donde la desigualdad se sigue ya que
2
2
C < A. Anlogamente, se tiene que OCM ICM . Luego, el punto O
a
es un punto interior de IBC, de donde OM r.

Solucin 3.78 Dena a = x2 , b = y 2 , c = z 2 , la desigualdad es equivalente a


o
x6 + y 6 + z 6 x4 yz + y 4 zx + z 4 xy.
Esto se sigue del teorema de Muirhead, ya que [6, 0, 0] [4, 1, 1].

4.3 Soluciones a los problemas del cap


tulo 3

215

Solucin 3.79 Use la desigualdad de Cauchy-Schwarz para ver que xy +


o

z = x y + z z x + z y + z = xy + z(x + y + z) = xy + z.

Anlogamente, yz + x yz + x y zx + y zx + y. Por lo tanto,


a

xy + z + yz + x + zx + y xy + yz + zx + x + y + z.
Solucin 3.80 Por el ejemplo 1.4.11, se tiene que
o
a3 + b3 + c3

(a + b + c)(a2 + b2 + c2 )
.
3

Ahora,
a3 + b3 + c3

a+b+c
3

(a2 + b2 + c2 )

abc(ab + bc + ca) ab + bc + ca,

donde se utilizaron la desigualdad M GM A y la desigualdad Cauchy-Schwarz.


Solucin 3.81 Use el ejemplo 1.4.11, obtendr (a + b + c)(a2 + b2 + c2 )
o
a
3 + b3 + c3 ), pero si por hiptesis (a + b + c)2 3(a3 + b3 + c3 ) entonces
3(a
o
a + b + c 1. Por otra parte,
4(ab + bc + ca) 1 a2 + b2 + c2 ab + bc + ca,
por lo tanto 3(ab + bc + ca) 1. Como
1 3(ab + bc + ca) (a + b + c)2 1,
se obtiene a + b + c = 1. Consecuentemente, a + b + c = 1 y 3(ab + bc + ca) =
(a + b + c)2 , lo cual implica a = b = c = 1 .
3
Solucin 3.82 Por la desigualdad de Cauchy-Schwarz se tiene
o
(|a| + |b| + |c|)2 3(a2 + b2 + c2 ) = 9.
Luego, |a| + |b| + |c| 3. De la desigualdad M G M A se tiene
a2 + b2 + c2 3 3 (abc)2
por lo que |abc| 1, que implica abc 1. La desigualdad requerida es
obtenida entonces por adicin.
o
Solucin 3.83 Note que
o
OA1
OB OC BC
(OBC)
4R
=
.
=

AA1
(ABC)
4R1
AB AC BC

216

Soluciones a los Ejercicios y Problemas

Luego, se tiene que demostrar que


OB OC BC + OA OB AB + OA OC AC AB AC BC.
Considere las coordenadas complejas O(0), A(a), B(b), C(c) y obtenga
|b| |c| |b c| + |a| |b| |a b| + |a| |c| |c a| |a b| |b c| |c a|.
Es decir,
|b2 c c2 b| + |a2 b b2 a| + |c2 a a2 c| |ab2 + bc2 + ca2 a2 b b2 c c2 a|,
lo cual es obvio por la desigualdad del tringulo.
a
Solucin 3.84 Sea S = {i1 , i1 + 1, ..., j1 , i2 , i2 + 1, ..., j2 , ..., ip , ..., jp } el orden
o
de S, donde jk < ik+1 para k = 1, 2, ..., p 1. Dena Sp = a1 + a2 + + ap ,
S0 = 0. Entonces

iS

ai = Sjp Sip 1 + Sjp1 Sip1 1 + + Sj1 Si1 1

y
(ai + + aj )2 =

1ijn

0ijn

(Si Sj )2 .

Es suciente demostrar una desigualdad de la forma


p
p+1

(x1 x2 + + (1)

xp )

x2 ,
i

(xj xi ) +

1i<jp

(4.9)

i=1

ya que esto signica olvidar los mismos trminos no-negativos en la expresin


e
o
de la derecha de la desigualdad dada. Entonces la desigualdad (4.9) se reduce a
p

4
1ijp
ji par

xi xj (p 1)

x2 .
i
i=1

Esto puede ser obtenido sumando las desigualdades de la forma 4xi xj 2(x2 +
i
x2 ), i < j, j i =par (para i impar en la desigualdad, xi aparece
j

y para i par, xi aparece

p
2

p1
2

veces

1 veces).

Solucin 3.85 Sean x = a + b + c, y = ab + bc + ca, z = abc. Entonces


o
2 + b2 + c2 = x2 2y, a2 b2 + b2 c2 + c2 a2 = y 2 2xz, a2 b2 c2 = z 2 , luego, la
a

4.3 Soluciones a los problemas del cap


tulo 3

217

desigualdad se convierte en z 2 + 2(y 2 2xz) + 4(x2 2y) + 8 9y, es decir,


z 2 + 2y 2 4xz + 4x2 17y + 8 0. Ahora como a2 + b2 + c2 ab + bc + ca = y
se obtiene x2 = a2 + b2 + c2 + 2y 3y.
Tambin,
e
a2 b2 + b2 c2 + c2 a2 = (ab)2 + (bc)2 + (ca)2
ab ac + bc ab + ac bc

= (a + b + c)abc = xz,
luego, y 2 = a2 b2 + b2 c2 + c2 a2 + 2xz 3xz. De donde,
z 2 + 2y 2 4xz + 4x2 17y + 8 =
+

x
3

8
10
+ (y 3)2 + (y 2 3xz)
9
9

35 2
(x 3y) 0,
9

como era requerido.


Segunda Solucin. Desarrollando el lado izquiedo de la desigualdad, se obtiene
o
la desigualdad equivalente
(abc)2 + 2(a2 b2 + b2 c2 + c2 a2 ) + 4(a2 + b2 + c2 ) + 8 9(ab + bc + ca).
Como 3(a2 + b2 + c2 ) 3(ab + bc + ca) and 2(a2 b2 + b2 c2 + c2 a2 ) + 6

4(ab + bc + ca) (ya que por ejemplo, 2a2 b2 + 2 4 a2 b2 = 4ab), es suciente


mostrar que
(abc)2 + a2 + b2 + c2 + 2 2(ab + bc + ca).

La parte (i) del ejercicio 1.90 nos dice que es suciente probar que (abc)2 + 2

3
3 a2 b2 c2 , pero esto se sigue de la desigualdad M G M A.

Solucin 3.86 Escriba


o
3

3
3

1
3
+ 6(a + b + c) =
3
abc
3

3
=
3
3

1 + 6a2 bc + 6b2 ac + 6c2 ab


=
abc

1 + 3ab(ac + bc) + 3bc(ba + ca) + 3ca(ab + bc)


,
abc

y considere la condicin ab + bc + ca = 1, para obtener


o
3

3
3

1 + 3ab 3(ab)2 + 3bc 3(bc)2 + 3ca 3(ca)2


=
abc

3
=
3
3

4 3((ab)2 + (bc)2 + (ca)2 )


.
abc

218

Soluciones a los Ejercicios y Problemas

Es fcil ver que 3((ab)2 + (bc)2 + (ac)2 ) (ab + bc + ac)2 (use la desigualdad
a
de Cauchy-Schwarz). Entonces es suciente probar que
3

3
3
que es equivalente a (abc)2
la desigualdad M G M A

1
27 .

1
3

,
abc
abc

Pero esta ultima desigualdad es inmediata de

(abc)2 = (ab)(bc)(ca)

ab + bc + ca
3

La igualdad se da si y slo si a = b = c =
o

1
.
27

1
.
3

Solucin 3.87 Por simetr es suciente mostrar que t1 < t2 + t3 . Se tiene


o
a,
n

ti
i=1

i=1

1
1
+
t2 t3

n + t1

1
=n+
ti

1i<jn

1
(t2 + t3 ) +
t1

tj
ti
+
tj
ti

(i,j)=(1,2),(1,3)

tj
ti
+
tj
ti

Usando la desigualdad M G M A, se tiene


1
1
+
t2 t3

tj
2
ti
+ 2, para todo i,j.
, t2 + t3 2 t2 t3 , y
tj
ti
t2 t3

Luego, si a = t1 / t2 t3 > 0 y utilizando la hiptesis se llega a


o
n

n
2

ti

n +1 >
i=1

i=1

t2 t3
n2 n
t1
2
1
n+2
+2
2 = 2a+ +n2 4.
+2
ti
t1
2
a
t2 t3

2
De donde, 2a + a 5 < 0 y esto implica 1/2 < a = t1 / t2 t3 < 2, por lo

que t1 < 2 t2 t3 . Una aplicacin ms de la desigualdad M G M A, implica


o
a

a.
t1 < 2 t2 t3 t2 + t3 , como se quer
Solucin 3.88 Note que 1 + b c = a + b + c + b c = a + 2b 0. Entonces
o

3
a 1+bca

1 + 1 + (1 + b c)
3

=a+

ab ac
.
3

4.3 Soluciones a los problemas del cap


tulo 3

219

Anlogamente,
a

bc ba
b3 1+ca b+
3

ca cb
3
.
c 1+ab c+
3
Sume estas tres desigualdades, y obtendr que
a

a 3 1 + b c + b 3 1 + c a + c 3 1 + a b a + b + c = 1.
Solucin 3.89 Si alguno de los nmeros es cero o si un nmero impar de
o
u
u
nmeros son negativos, entonces x1 x2 x6 0 y la desigualdad es obvia.
u
Por lo tanto, unicamente podemos tener 2 o 4 nmeros negativos entre los

u
nmeros involucrados en la desigualdad. Supongamos que ninguno de ellos es
u
cero y que hay dos nmeros negativos (para el otro caso, cambie el signo de
u
2
2
2
todos los nmeros). Si yi = |xi | entonces, es claro que y1 + y2 + + y6 = 6,
u
y1 + y2 = y3 + + y6 , and that x1 x2 x6 = y1 y2 y6 .
De la desigualdad M G M A se obtiene
y1 y2

y1 + y2
2

= A2 .

Adems, la desigualdad M G M A garantiza que


a
y3 y4 y5 y6

y3 + y4 + y5 + y6
4

y1 + y2
4

1 4
A .
24

1
Por lo tanto, y1 y2 y6 24 A6 .
Por otro lado, la desigualdad de Cauchy-Schwarz implica que,
2
2
2(y1 + y2 ) (y1 + y2 )2 = 4A2
2
2
2
2
4(y3 + y4 + y5 + y6 ) (y3 + y4 + y5 + y6 )2 = 4A2 .

2
2
2
Es decir, 6 = y1 + y2 + + y6 2A2 + A2 = 3A2 y entonces y1 y2 ...y6
3
2
1
1 6
24 A 24 = 2 .
b c
Solucin 3.90 Use la desigualdad de Cauchy-Schwarz con (1, 1, 1) y ( a , c , a )
o
b
para obtener
2

(1 + 1 + 1 )

a2 b2
c2
+ 2+ 2
b2
c
a

a b c
+ +
b c a

220

Soluciones a los Ejercicios y Problemas

La desigualdad M G M A implica que


c2
a2 b2
+ 2+ 2
b2
c
a
Anlogamente,
a

a
c

b
a

c
b

33

abc
bca

a
b

b
c

c
a

abc
bca

33

a b c
+ +
b c a

= 3, luego

= 3. Por lo tanto,

a2 b2
a b c
c2
a b c
+ 2 + 2 + + + 3+ + + .
b2
c
a
c a b
b c a
Sumando

a
c

b
a

c
b

en ambos lados, se obtiene el resultado.

Solucin 3.91 Note que


o
(a2 a + 1) + (a + 1)
a2 + 2
=

2
2

(a2 a + 1)(a + 1) =

1 + a3 .

Despus de sustituir en la desigualdad se tiene que demostrar que


e

(a2

b2
c2
1
a2
+ 2
+ 2
.
2 + 2)
2 + 2)
+ 2)(b
(b + 2)(c
(c + 2)(a2 + 2)
3

Sea x = a2 , y = b2 , z = c2 , luego xyz = 64 y


x
y
z
1
+
+

(x + 2)(y + 2) (y + 2)(z + 2) (z + 2)(x + 2)


3
si y slo si
o
3[x(z + 2) + y(x + 2) + z(y + 2)] (x + 2)(y + 2)(z + 2).
Ahora, 3(xy + yz + zx)+ 6(x+ y + z) xyz + 2(xy + yz + zx)+ 4(x+ y + z)+ 8
si y slo si xy + yz + zx + 2(x + y + z) xyz + 8 = 72, pero por la desigualdad
o
M G M A se tiene que x + y + z 12 y xy + yz + zx 48, lo cual concluye
la demostracin.
o
Solucin 3.92 Observe que
o
x2 (y 2 + z 2 )(x3 1)2
x5 x2
x5 x2
= 3 5
0.
3 2
x5 + y 2 + z 2 x (x + y 2 + z 2 )
x (x + y 2 + z 2 )(x2 + y 2 + z 2 )

4.3 Soluciones a los problemas del cap


tulo 3

221

Luego
x5 x2
x5 + y 2 + z 2

x5 x2
x3 (x2 + y 2 + z 2 )
1
1
x2
2 + y2 + z2
x
x
1
(x2 yz) 0.
x2 + y 2 + z 2

La segunda desigualdad se sigue del hecho de que xyz 1, es decir,


La ultima desigualdad se sigue de la ecuacin (1.8).

1
x

yz.

Segunda Solucin. Primero, observe que


o
x5 + y 2 + z 2 (x2 + y 2 + z 2 )
x2 + y 2 + z 2
x5 x2
=
=1 5
.
x5 + y 2 + z 2
x5 + y 2 + z 2
x + y2 + z2
Ahora tiene que probar que
1
1
1
3
+
+
2
.
x5 + y 2 + z 2 x5 + z 2 + x2 x5 + x2 + y 2
x + y2 + z2
Usando la desigualdad de Cauchy-Schwarz se obtiene
(x2 + y 2 + z 2 )2 (x2 x3 + y 2 + z 2 )(x2
y como xyz 1 entonces x2

1
x3

1
x

1
+ y2 + z2 )
x3

yz, por lo que

(x2 + y 2 + z 2 )2 (x5 + y 2 + z 2 )(yz + y 2 + z 2 )


por lo tanto,
1

5 + y2 + z2
x

yz + y 2 + z 2

(x2 + y 2 + z 2 )2

y 2 +z 2
+ y2 + z2
2
(x2 + y 2 + z 2 )2

x2

3
.
+ y2 + z2

Solucin 3.93 Inicie observando que


o
(1 + abc)

1
1
1
+
+
a(b + 1) b(c + 1) c(a + 1)

+3

1 + abc + ab + a 1 + abc + bc + b 1 + abc + ca + c


+
+
a(b + 1)
b(c + 1)
c(a + 1)

b(c + 1)
1+b
c(a + 1)
1+c
a(b + 1)
1+a
+
+
+
+
+
6.
a(b + 1)
(b + 1)
b(c + 1)
(c + 1)
c(a + 1)
(a + 1)

222

Soluciones a los Ejercicios y Problemas

La ultima desigualdad se sigue despus de aplicar la desigualdad M G M A

e
para seis nmeros.
u
Solucin 3.94 Sea R el circunradio del tringulo ABC. Como BOC = 2A,
o
a
COA = 2B y AOB = 2C se tiene que
(ABC) = (BOC) + (COA) + (AOB)
R2
(sen 2A + sen 2B + sen 2C)
2
2A + 2B + 2C
R2
3 sen

2
3

2
2
R
3 3R2
3 sen
.
=
=
2
3
4
=

La desigualdad se debe a que la funcin sen x es cncava en [0, ].


o
o
Por otro lado, como BOC es issceles, la mediatriz OA de BC es tambin bio
e
sectriz del ngulo BOC por lo que BOA = COA = A; anlogamente,
a
a
COB = AOB = B y AOC = BOC = C.
En el tringulo B OC la altura sobre el lado B C es R y B C = R (tan B +
a
2
2
2
tan C), por lo que el rea del tringulo B OC es (B OC ) = R (tan B+tan C).
a
a
8
2
2
Anlogamente (C OA ) = R (tan C + tan A) y (A OB ) = R (tan A + tan B).
a
8
8
Luego,
(A B C ) = (B OC ) + (C OA ) + (A OB )
=

R2
(tan A + tan B + tan C)
4
R2
3 tan
4

A+B+C
3

R2
3 3R2

=
=
3 tan
.
4
3
4

La desigualdad ocurre por ser convexa la funcin tan x en [0, ].


o
2
De donde se concluye que,

3 3R2

(A B C )
(ABC).
4


a
Solucin 3.95 Note primero que a2 + bc 2a2 bc = 2 ab ca. Anlogao

2 + ca 2 bc ab, c2 + ab 2ca bc; luego se puede asegurar
mente, b

4.3 Soluciones a los problemas del cap


tulo 3

223

que
1
1
1
1
+
+

a2 + bc b2 + ca c2 + ab
2

1
1
1
+ +
ab ca
bc ab
ca bc

Usando la desigualdad de Cauchy-Schwarz de la siguiente manera


1
1
1
+ +
ab ca
bc ab
ca bc

1
1
1
+
+
ab bc ca

1
1
1
+
+
ca ab bc

el resultado se concluye.
Solucin 3.96 Por la desigualdad de Cauchy-Schwarz se obtiene
o

2
2
n
ai

= (n 1)
ai
ai aj
ai
= (n 1)2 A2 .
aj
i=j

i=1

i=j

i=j

Por otro lado,

ai

ai aj =
i=1

i=j

a2
i
i=1

= A2 A.

Solucin 3.97 Sin prdida de generalidad, suponga que a1 an . Sea


o
e
dk = ak+1 ak para k = 1, ..., n. Entonces d = d1 + + dn1 . Para i < j, se
tiene |ai aj | = aj ai = di + + dj1 . Entonces,
n j1

s=
i<j

|ai aj | =

j=2 i=1
n

=
j=2

(di + + dj1 )

(d1 + 2d2 + + (j 1)dj1 )

= (n 1)d1 + (n 2)2d2 + + 1 (n 1)dn1


n1

=
k=1

k(n k)dk .

Como k(n k) (n 1), (ya que (k 1)(n k 1) 0) y 4k(n k) n2


2
(por la desigualdad M G M A), se obtiene que (n 1)d s n4 d .
Para ver cundo la desigualdad del lado izquierdo es igualdad, observe que
a
k(n k) = (n 1) n(k 1) = k2 1 k = 1 o k = n 1, entonces
(n 1)d = s slo si d2 = = dn2 = 0, esto es, a1 a2 = = an1 an .
o

224

Soluciones a los Ejercicios y Problemas

Para ver cundo la segunda desigualdad es igualdad, observe que 4k(nk) = n2


a
k = n k. Si n es impar, la igualdad 4k(n k) = n2 se da unicamente

cuando dk = 0, para toda k, por lo tanto, a1 = = an = 0. Si n es par, es


decir, n = 2k, la unica dk puede ser distinta de cero y entonces a1 = =

ak ak+1 = = a2k .
Solucin 3.98 Considere el polinomio P (t) = tb(t2 b2 ) + bc(b2 c2 ) +
o
ct(c2 t2 ). Este satisface las igualdades P (b) = P (c) = P (b c) = 0, luego
P (t) = (b c)(t b)(t c)(t + b + c), ya que el coeciente de t3 es (b c).
Por lo que
|ab(a2 b2 ) + bc(b2 c2 ) + ca(c2 a2 )| = |P (a)|

= |(b c)(a b)(a c)(a + b + c)|.

El problema es, ahora, encontrar el menor nmero M que cumpla para todos
u
los nmeros a, b, c, la desigualdad
u
|(a c)(a b)(b c)(a + b + c)| M (a2 + b2 + c2 )2 .
Pero si (a, b, c) cumple la desigualdad, tambin, (a, b, c) la cumple para
e
cualquier nmero real . Por lo que se puede suponer, sin perder generalidad,
u
que a2 + b2 + c2 = 1. As el problema se reduce a encontrar el valor mximo de

a
P = |(a b)(a c)(b c)(a + b + c)| para nmeros reales a, b, c que satisfacen
u
a2 + b2 + c2 = 1.
Note que
[3(a2 + b2 + c2 )]2 = [2(a b)2 + 2(a c)(b c) + (a + b + c)2 ]2
8|(a c)(b c)|[2(a b)2 + (a + b + c)2 ]

16 2|(a c)(b c)(a b)(a + b + c)|

= 16 2P.

Las dos desigualdades se obtienen de aplicar la desigualad M G M A.

Lo que garantiza que P 169 2 , y el valor mximo es 169 2 , ya que la igualdad


a
ocurre con a =


3 3+ 6

,
6 2

b=

6 2

yc=

63 3

.
6 2

1
Solucin 3.99 Para a = 2, b = c = 2 y n 3 la desigualdad no es vlida.
o
a

3
a+b+c
Si n = 1, la desigualdad se reduce a abc 1, que se sigue de abc 3 = 1.
Para el caso n = 2, sea x = ab + bc + ca; entonces como a2 + b2 + c2 = (a + b +
c)2 2(ab + bc + ca) = 9 2x y x2 = (ab + bc + ca)2 3(a2 bc + ab2 c + abc2 ) =

4.3 Soluciones a los problemas del cap


tulo 3

225

3abc(a + b + c) = 9abc, la desigualdad es equivalente a abc(9 2x) 3. Pero,

bastar demostrar que x2 (9 2x) 27. Esta ultima desigualdad es a su vez


a

2 0.
equivalente a (2x + 3)(x 3)
Solucin 3.100 Primero use la desigualdad M GM A para ver que ca+c+a
o
3
3 c2 a2 . Por lo que
(a + 1)(b + 1)2
(a + 1)(b + 1)2
(a + 1)(b + 1)2
(b + 1)2

=
=
.
3
ca + c + a + 1
(c + 1)(a + 1)
(c + 1)
3 c2 a2 + 1
Anlogamente para los otros dos sumandos; y entonces se tiene
a
(a + 1)(b + 1)2 (b + 1)(c + 1)2 (c + 1)(a + 1)2

+
+
3
3
3
3 c2 a2 + 1
3 a2 b2 + 1
3 b2 c2 + 1
(c + 1)2
(a + 1)2
(b + 1)2
+
+
.

(c + 1)
(a + 1)
(b + 1)
Ahora aplique la desigualdad (1.11).
Solucin 3.101 Use la transformacin de Ravi a = x + y, b = y + z, c = z + x,
o
o
a
obtendr que x + y + z = 3 y xyz ( x+y+z )3 = 1 . Adems,
a
2
3
8
(a2 + b2 + c2 )(a + b + c) + 4abc
4abc
=
3
3
2 + (z + x)2 + (x + y)2 )(x + y + z) + 4(y + z)(z + x)(x + y)
2((y + z)
=
3
4
3
= ((x + y + z) xyz)
3
13
3 3 1
4
= .

3
2
8
3

a2 + b2 + c2 +

Por lo tanto el valor m


nimo es

13
3 .

Solucin 3.102 Aplique la transformacin de Ravi, a = y + z, b = z + x,


o
o
c = x + y, de tal forma que la desigualdad se puede reescribir como
(2z)4
(2x)4
(2y)4
+
+
(z + x)(2x) (x + y)(2y) (y + z)(2z)
(y + z)(z + x) + (z + x)(x + y) + (x + y)(y + z).

226

Soluciones a los Ejercicios y Problemas

Aplique la desigualdad (1.11) y la desigualdad del ejercicio 1.27, para ver que
(2z)4
(2x)4
(2y)4
+
+
(z + x)(2x) (x + y)(2y) (y + z)(2z)

8(x2 + y 2 + z 2 )2
x2 + y 2 + z 2 + xy + yz + zx
8(x2 + y 2 + z 2 )2
.
2(x2 + y 2 + z 2 )

Por otro lado (y + z)(z + x) + (z + x)(x + y) + (x + y)(y + z) = 3(xy + yz +


zx) + (x2 + y 2 + z 2 ); luego bastar demostrar que 4(x2 + y 2 + z 2 ) 3(xy +
a
yz + zx) + (x2 + y 2 + z 2 ), que se reduce a ver que x2 + y 2 + z 2 xy + yz + zx.
a+b
b+c
c+a
Solucin 3.103 La sustitucin x = ab , y = bc , z = ca , tiene la propiedad
o
o
de que xy + yz + zx = 1. Por la desigualdad de Cauchy-Schwarz, (x + y + z)2

3(xy + yz + zx) = 3, por lo tanto |x + y + z| 3 > 1.

Solucin 3.104 Bastar considerar el caso en que x y z. Entonces


o
a
x = y a, z = y + b con a, b 0.
Por un lado se tiene que, xz = 1 xy yz = 1 (y a)y y(y + b) =
1 2y 2 + ay by y por otro xz = (y a)(y + b) = y 2 ay + by ab. Al sumar
ambas identidades se tiene que 2xz = 1y 2 ab, luego 2xz1 = y 2 ab 0.
1
Si 2xz = 1 entonces y = 0 y xz = 1 una contradiccin, luego xz < 2 .
o
1
1
1
Los nmeros x = y = n y z = 2 (n n ), cumplen x y z y xy +yz+zx = 1.
u
1
1
1
1
Sin embargo, xz = 2n (n n ) = 2 2n2 puede estar tan cerca como se desee
de 1 , por lo que el valor 1 no puede mejorarse.
2
2
Solucin 3.105 Suponga que a = [x] y que r = {x}. Entonces, la desigualdad
o
es equivalente a
a
a + 2r

a
a + 2r

2a + r
r

r
2a + r

9
> .
2

Que se reduce a
2
Pero como

r
a

a
r

r a

+
a r

a
r
+
a + 2r 2a + r

5
> .
2

2, bastar ver que,


a
a
r
3
+
< .
a + 2r 2a + r
2

Pero a+2r a+r y 2a+r a+r; ms an, en las dos desigualdades anteriores
a u
las igualdades no se pueden dar simultneamente (ya que eso implicar que
a
a

4.3 Soluciones a los problemas del cap


tulo 3

227

a = r = 0), por lo que


a
r
a
r
3
+
<
+
=1< .
a + 2r 2a + r
a+r a+r
2
Solucin 3.106 Aplique la desigualdad (1.11) para ver que
o
a+b+c

32
1 1 1
+ +
,
a b
c
a+b+c

3
entonces a+b+c a+b+c . Entonces, bastar demostrar que a + b + c
a
3
2 3(xy + yz + zx), sucede que
Como (x + y + z)

1 1 1
+ +
a b
c

(a + b + c)2

1
1
1
+
+
ab bc ca

3
abc .

3
(a + b + c),
abc

y de aqu es inmediato concluir.


,
Solucin 3.107 Utilizando la desigualdad de Cauchy-Schwarz se tiene que
o
(a + b + 1)(a + b + c2 ) (a + b + c)2 .
Entonces
a2 + b + c
a + b2 + c
a + b + c2
+
+
(a + b + c)2 (a + b + c)2 (a + b + c)2
1
1
1

+
+
1.
a+b+1 b+c+1 c+a+1
Por lo tanto,
2(a + b + c) + (a2 + b2 + c2 ) (a + b + c)2 = a2 + b2 + c2 + 2(ab + bc + ca)
de aqu el resultado.

Solucin 3.108 Para un punto P dentro de ABC, considere el punto Q sobre


o
la mediatriz de BC tal que AQ = AP. Sea S la interseccin de BP con la
o
tangente a la circunferencia en Q. Entonces, SP + P C SC, por lo que
BP + P C = BS + SP + P C BS + SC.
Por otro lado, BS + SC BQ + QC, luego BP + P C es m
nimo si P = Q.
Sea T el punto medio del segmento M N. Como el tringulo AM Q es issceles
a
o
y M T es una de sus alturas, entonces M T = ZQ donde Z es el pie de la altura
de Q sobre AB. Luego M N + BQ + QC = 2(M T + QC) = 2(ZQ + QC) es

228

Soluciones a los Ejercicios y Problemas

m
nimo cuando Z, Q, C sean colineales y esto signica que CZ es la altura.
Por simetr BQ tambin deber ser altura y entonces P es el ortocentro.
a,
e
a
Solucin 3.109 Sea H el ortocentro del tringulo M N P y sean A , B , C
o
a
las proyecciones de H sobre BC, CA, AB, respectivamente. Como el tringulo
a
M N P es acutngulo H est en el interior y tambin en el interior del tringulo
a
a
e
a
ABC, y entonces
x HA + HB + HC HM + HN + HP 2X.
La segunda desigualdad es clara, las otras dos son los siguientes dos lemas.
Lema 1. Si H es un punto interior o sobre los lados de un tringulo ABC, y si
a
, B , C son sus proyecciones sobre BC, CA, AB, respectivamente, entonces
A
x HA + HB + HC , donde x es la longitud de la altura menor de ABC.
Demostracin.
o
HA HB HC
(BHC) (CHA) (AHB)
HA + HB + HC

+
+
=
+
+
= 1.
x
ha
hb
hc
(ABC) (ABC) (ABC)
Lema 2. Si M N P es un tringulo acutngulo y H es su ortocentro, entonces
a
a
HM + HN + HP 2X, donde X es la altura ms grande del tringulo M N P.
a
a
Demostracin. Suponga que M N P, entonces N P P M M N
o
y entonces pasa que X es igual a la altura M M . Debe ver que HM + HN +
HP 2M M = 2(HM + HM ), o equivalentemente que HN + HP
HM + 2HM .
Sea H el punto simtrico de H con respecto a N P ; como M N H P es un
e
cuadriltero c
a
clico, el teorema de Ptolomeo dice que
H M N P = H N M P + H P M N H N N P + H P N P
y entonces se obtiene que H N + H P H M = HM + 2HM .
Solucin 3.110 Suponga, sin prdida de generalidad, que x y z. Por lo
o
e
que x + y z + x y + z, xy zx yz, 2z 2 (x + y) 2y 2 (z + x) 2x2 (y +
21
21
. Por la desigualdad del reacomodo,
z), 21
2z (x+y)

2y (z+x)

2x (y+z)

aplicada dos veces, se tiene que

2yz
2x2 (y + z)

xy + zx
2x2 (y + z)

4.3 Soluciones a los problemas del cap


tulo 3
2x2

Ahora, al sumar

2x2 (y + z)

229

de ambos lados de la desigualdad anterior, se

obtiene
2x2 + 2yz
2x2 (y + z)

2x2 + xy + zx
2x2 (y + z)

2x2 + x(y + z)

2x2 (y + z)
2x3 (y + z)

2x2 (y + z)

= 2( x + y + z) = 2.
Segunda Solucin. Note primero que,
o
2x2 (y

+ z)

x2 x(y + z) + yz

x2 + yz

(x y)(x z)

2x2 (y

+ z)

x(y + z)

2x2 (y + z)

y+z
2
2x2 (y + z)

y+ z
(x y)(x z)
+
.
2
2x2 (y + z)

Anlogamente para los otros dos sumandos; por lo que


a
x2 + yz
2x2 (y

+ z)

(x y)(x z)
2x2 (y

+ z)

x+

y+

z.

Luego, bastar ver que


a
(x y)(x z)
2x2 (y + z)

(y z)(y x)
2y 2 (z + x)

(z x)(z y)
2z 2 (x + y)

0.

2
Sin perder generalidad, suponga que x y z. Luego (xy)(xz) 0, y
2x (y+z)

(y z)(y x)
+ x)

(z x)(z y)
2z 2 (x

= (y z)(x y)

+ y)

(x z)(y z)
2z 2 (x

2y 2 (z

2z 2 (x + y)

+ y)
(x y)(y z)

1
2z 2 (x

+ y)

1
2y 2 (z

+ x)

(y z)(x y)

2y 2 (z + x)
(y z)(x y)
2y 2 (z + x)

0.

230

Soluciones a los Ejercicios y Problemas

La ultima desigualdad es consecuencia de y 2 (z + x) = y 2 z + y 2 x yz 2 + z 2 x =

z 2 (x + y).
Solucin 3.111 Por la desigualdad (1.11)
o
b2
c2
(a + b + c)2
a2
+
+

.
2 + b + c2 2 + c + a2 2 + a + b2
6 + a + b + c + a2 + b2 + c2
Entonces, tiene que probar 6 + a + b + c + a2 + b2 + c2 12, pero como
a2 + b2 + c2 = 3, es suciente mostrar que a + b + c 3. Pero se tiene tambin
e
que (a + b + c)2 = a2 + b2 + c2 + 2(ab + bc + ca) 3(a2 + b2 + c2 ) = 9.
La igualdad se da si y slo si a = b = c = 1.
o
Solucin 3.112 Note primero que
o
1

2bc
2bc
2bc
a bc
=
=
=
.
a + bc
1 b c + bc
(1 b)(1 c)
(c + a)(a + b)

Por lo que la desigualdad es equivalente a


2ca
2ab
3
2bc
+
+
.
(c + a)(a + b) (a + b)(b + c) (b + c)(c + a)
2
Simplique esta ultima desigualdad para obtener que

4 [bc(b + c) + ca(c + a) + ab(a + b)] 3(a + b)(b + c)(c + a),


que a su vez es equivalente a la desigualdad
ab + bc + ca 9abc.
1
Pero esta ultima es consecuencia de (a + b + c)( a +

1
b

+ 1 ) 9.
c

Solucin 3.113 Note que (x y+y z+z x)2 = x2 y+y 2 z+z 2 x+2(xy yz+
o

yz zx + zx xy).
La desigualdad M G M A garantiza que

xy yz = xyz

xy 2

xyz + xy 2
,
2

por lo que,

(x y + y z + z x)2 x2 y + y 2 z + z 2 x + xy 2 + yz 2 + zx2 + 3xyz.

4.3 Soluciones a los problemas del cap


tulo 3

231

Como (x + y)(y + z)(z + x) = x2 y + y 2 z + z 2 x + xy 2 + yz 2 + zx2 + 2xyz, se


tiene que

(x y + y z + z x)2 (x + y)(y + z)(z + x) + xyz


1
(x + y)(y + z)(z + x) + (x + y)(y + z)(z + x)
8
9
=
(x + y)(y + z)(z + x).
8
9
8 y entonces
3
, este valor
2 2

Por lo tanto K 2

igualdad con K =

3
.
2 2

Como para x = y = z, se da la

es el m
nimo.

Segunda Solucin. Aplique la desigualdad de Cauchy-Schwarz de la siguiente


o
forma:

x y+y z+z x = x xy+ y yz+ z zx

(x + y + z)(xy + yz + zx).

Despus de esto, use la desigualdad M G M A varias veces para tener


e

(x + y + z) (xy + yz + zx)
3 xyz
3
3

x2 y 2 z 2 = xyz

(x + y) (y + z) (z + x)
.
2
2
2

Solucin 3.114 El lado izquierdo de la desigualdad se escribir tambin como


o
e
a2 b2 cd+ab2 c2 d+abc2 d2 +a2 bcd2 +a2 bc2 d+ab2 cd2 = abcd(ab+bc+cd+ac+ad+bd).
2

La desigualdad M G M A garantiza que a2 b2 c2 d2 ( a +b +c +d )4 = 1 ,


4
4
1
por lo que abcd 16 . Para ver que el factor (ab + bc + cd + ac + ad + bd) es
menor a 3 se pueden seguir los dos caminos siguientes:
2
Uno es aplicar la desigualdad de Cauchy-Schwarz para tener que
(ab + bc + cd + ac + ad + bd + ba + cb + dc + ca + da + db)
(a2 + b2 + c2 + d2 + a2 + b2 + c2 + d2 + a2 + b2 + c2 + d2 ) = 3.
El otro consiste en aplicar la desigualdad M G M A como sigue:
(ab + bc + cd + ac + ad + bd)

a2 + b2 b2 + c2 c2 + d2
+
+
2
2
2

a2 + c2 a2 + d2 b2 + d2
3
+
+
= .
2
2
2
2

232

Soluciones a los Ejercicios y Problemas

Solucin 3.115 (a) Desarrollando los cuadrados y agrupando los trminos se


o
e
tiene
(1 + x + y)2 + (1 + y + z)2 + (1 + z + x)2
= 3 + 4(x + y + z) + 2(xy + yz + zx) + 2(x2 + y 2 + z 2 ).
Ahora, aplique la desigualdad M G M A para obtener

(x + y + z) 3 3 xyz 3

(xy + yz + zx) 3 3 x2 y 2 z 2 3

(x2 + y 2 + z 2 ) 3 3 x2 y 2 z 2 3.

Luego, (1 + x + y)2 + (1 + y + z)2 + (1 + z + x)2 3 + 4 3 + 2 3 + 2 3 = 27.


La igualdad se da cuando x = y = z = 1.
(b) Nuevamente, desarrollando los cuadrados la desigualdad es equivalente a
3 + 4(x + y + z) + 2(xy + yz + zx) + 2(x2 + y 2 + z 2 )
3(x2 + y 2 + z 2 ) + 6(xy + yz + zx)
y tambin a 3 + 4u u2 + 2v, donde u = x + y + z 3 y v = xy + yz + zx 3.
e
Pero u 3 implica que (u 2)2 1. Luego, (u 2)2 + 2v 1 + 6 = 7.
La igualdad se da cuando u = 3 y v = 3, es decir, cuando x = y = z = 1.
Solucin 3.116 Note que
o
1

1
1
1
=
=
.
1+
+ c)
1 + a(ab + ac)
1 + a(3 bc)
3a + 1 abc

3
La desigualdad M G M A implica que 1 = ab+bc+ca a2 b2 c2 , entonces
3
abc 1. Luego,
1
1
1
=

.
2 (b + c)
1+a
3a + 1 abc
3a
=

a2 (b

Anlogamente,
a
1
1

+ a2 (b

+ c)

1
1+b2 (c+a)

1
3b

1
1+

b2 (c

+ a)

1
1+c2 (a+b)

1
3c .

1
1+

c2 (a +

b)

Por lo tanto,

1
1
1
+
+
3a 3b 3c
1
bc + ca + ab
=
.
3abc
abc

Solucin 3.117 La desigualdad es equivalente a


o
(a + b + c)

1
1
1
+
+
a+b b+c c+a

k + (a + b + c)k = (a + b + c + 1)k.

4.3 Soluciones a los problemas del cap


tulo 3

233

Por otro lado, usando la condicin a + b + c = ab + bc + ca, se tiene que


o
1
1
1
+
+
a+b b+c c+a

=
=
=

a2 + b2 + c2 + 3(ab + bc + ca)
(a + b)(b + c)(c + a)
2 + b2 + c2 + 2(ab + bc + ca) + (ab + bc + ca)
a
(a + b)(b + c)(c + a)
(a + b + c)(a + b + c + 1)
.
(a + b + c)2 abc

Luego,
(a + b + c)
(a + b + c + 1)

1
1
1
+
+
a+b b+c c+a

(a + b + c)2
1,
(a + b + c)2 abc

y como la igualdad se da si y slo si abc = 0, se tiene que k = 1 es el valor


o
mximo.
a
Solucin 3.118 Multiplicando ambos lados de la desigualdad por el factor
o
(a + b + c), se obtiene la desigualdad equivalente
9(a + b + c)(a2 + b2 + c2 ) + 27abc 4(a + b + c)3 ,
la cual es a su vez equivalente a
5(a3 + b3 + c3 ) + 3abc 3(ab(a + b) + ac(a + c) + bc(b + c)).
Por la desigualdad de Schr con n = 1, ver el ejercicio 2.83 , se sigue que
u
a3 + b3 + c3 + 3abc ab(a + b) + bc(b + c) + ca(c + a),
y la desigualdad de Muirhead nos dice que 2[3, 0, 0] 2[2, 1, 0], que es equivalente a
4(a3 + b3 + c3 ) 2(ab(a + b) + ac(a + c) + bc(b + c)).
Sume las ultimas desigualdades y obtendr el resultado.

a
1
1
4
Solucin 3.119 Lema. Si a, b > 0, entonces (ab)2 + a2 + b1 ab .
o
2
Demostracin.
o
2
2 3ab)2
1
1
4
Basta notar que (ab)2 + a2 + b1 ab = (a 2+b(ab)2 .
2
a b2
Sin perder generalidad suponga que z = m
n{x, y, z}; ahora aplique el lema
con a = (x z) y b = (y z), para tener

1
1
4
1
.
+
+

(x y)2 (y z)2 (z x)2


(x z)(y z)

234

Soluciones a los Ejercicios y Problemas

Ahora, ser suciente mostrar que xy + yz + zx (x z)(y z); pero esto es


a
equivalente a 2z(y + x) z 2 , que es inmediato.
Solucin 3.120 Para el caso (i) hay varias maneras de resolverlo.
o
a f orma. Puede mostrase que
1
y2
z2
(yz + zx + xy 3)2
x2
+
+
1=
.
(x 1)2
(y 1)2 (z 1)2
(x 1)2 (y 1)2 (z 1)2
y
x
z
2a f orma. Con la sustitucin a = x1 , b = y1 , c = z1 , la desigualdad
o
por demostrar es a2 + b2 + c2 1, y la condicin xyz = 1 es equivalente a
o
abc = (a 1)(b 1)(c 1) o (ab + bc + ca) + 1 = a + b + c. Con la ayuda de
las identidades anteriores se puede obtener que

a2 + b2 + c2 = (a + b + c)2 2(ab + bc + ca)

= (a + b + c)2 2(a + b + c 1)

= (a + b + c 1)2 + 1
luego

a2 + b2 + c2 = (a + b + c 1)2 + 1.
El caso (ii) se puede demostrar dependiendo de como se lleg a establecer
o
el caso (i). Por ejemplo, si se demostr de la 2a forma, la igualdad se tiene
o
cuando a2 + b2 + c2 = 1 y a + b + c = 1. (En la 1a f orma la igualdad se tiene
cuando xyz = 1 y xy + yz + zx = 3). De las ecuaciones se puede cancelar
una variable, por ejemplo c (y como c = 1 a b, si se demuestra que a
y b nmeros racionales el c ser tambin un nmero racional), para llegar a
u
a
e
u
a2 + b2 + ab a b = 0. Esta identidad se puede trabajar como una ecuacin
o

1a

(1a)(1+3a)

, que sern nmeros racionales


a u
cuadrtica en b con ra b =
a
ces
2
k
si (1 a) y (1 + 3a) son cuadrados de nmeros racionales. Si a = m , entonces
u
sucede que m k y m + 3k son cuadrados de nmeros enteros, por ejemplo, si
u
m = (k 1)2 + k, es claro que m k = (k 1)2 y m + 3k = (k + 1)2 . Luego,
2
k
los nmeros racionales a = m , b = mk+k 1 y c = 1 a b, cuando k varia
u
2m
en los nmeros enteros, son nmeros racionales donde se alcanza la igualdad.
u
u
Hay unas excepciones k = 0, 1, ya que los valores a = 0 1 no son valores
o
permitidos.

Notacin
o

Utilizamos la siguiente notacin estndar:


o
a

N
R
R+

aA
AB
|x|
{x}
[x]
[a, b]
(a, b)
f : [a, b] R
f (x)
f (x)
det A
n
i=1 ai
n
i=1 ai
i=j ai
mx{a, b, . . . }
a
m
n{a, b, . . . }

los nmeros enteros positivos (nmeros naturales)


u
u
los nmeros reales
u
los nmeros reales positivos
u
si y slo si
o
implica
el elemento a pertenece al conjunto A
A es un subconjunto de B
valor absoluto del nmero real x
u
la parte fraccionaria de un nmero real x
u
la parte entera de un nmero real x
u
el conjunto de nmeros reales x tal que a x b
u
el conjunto de nmeros reales x tal que a < x < b
u
la funcin f denida en [a, b] con valores en R
o
la derivada de la funcin f (x)
o
la segunda derivada de la funcin f (x)
o
el determinante de la matriz A
la suma a1 + a2 + + an
el producto a1 a2 an
el producto para toda a1 , a2 , . . . , an excepto aj
el mximo valor entre a, b, . . .
a
el m
nimo valor entre a, b, . . .
la ra del nmero real positivo x
z
u

236

Notacin
o

x
exp x = ex
f (a, b, . . . )

la n sima ra del nmero real positivo x


e
z
u
la funcin exponencial
o
representa la suma de la funcin f evaluada
o

c
clica

en todas las permutaciones c


clicas de las variables a, b, . . .
Utilizamos la siguiente notacin en la seccin del teorema de Mirhead:
o
o
u
!

F (x1 , . . . , xn )

(b) (a)
[b] [a]

la suma de n! trminos obtenidos cuando se evala F en


e
u
todas las posibles permutaciones de (x1 , . . . , xn )
(a) mayoriza a (b)
1
1
xb1 xb2 xbn n!
xa1 xa2 xan
n
n
1 2
1 2
n!
!

Utilizamos la siguiente notacin geomtrica:


o
e
A, B, C
a, b, c
A , B , C
ABC
A
(ABC)
(ABCD...)
ma , mb , mc
ha , hb , hc
la , lb , lc
s
r
R
I, O, H, G
Ia , Ib , Ic

los vrtices del tringulo ABC


e
a
las longitudes de los lados del tringulo ABC
a
los puntos medios de los lados BC, CA y AB
el ngulo ABC
a
el ngulo en el vrtice A o la medida del ngulo en el vrtice A
a
e
a
e
el rea del tringulo ABC
a
a
el rea del pol
a
gono ABCD...
las longitudes de las medianas del tringulo ABC
a
las longitudes de las alturas del tringulo ABC
a
las longitudes de las bisectrices internas del tringulo ABC
a
el semiper
metro del tringulo ABC
a
el inradio del tringulo ABC, el radio del inc
a
rculo
el circunradio del tringulo ABC, el radio del
a
circunc
rculo
el incentro, el circuncentro, el ortocentro y el centroide
del tringulo ABC
a
los centros del exc
rculo del tringulo ABC
a

Utilizamos la siguiente notacin referente a los problemas:


o
IMO
APMO
(pa ao)
s, n

Olimpiada Internacional de Matemticas (por sus siglas en ingls)


a
e
Olimpiada de la Cuenca del Pac
co (por sus siglas en ingls)
e
problema que corresponde a la olimpiada de matemticas
a
celebrada en ese pa en ese ao, en alguna de las etapas
s,
n

Bibliograf
a

[1] Altshiller, N., College Geometry: An Introduction to Modern Geometry of


the Triangle and the Circle. Barnes and Noble, 1962.
[2] Andreescu, T., Feng, Z., Problems and Solutions from Around the World,
Mathematical Olympiads 1999-2000. MAA, 2002.
[3] Andreescu, T., Feng, Z., Lee, G., Problems and Solutions from Around
the World, Mathematical Olympiads 2000-2001. MAA, 2003.
[4] Andreescu, T., Enescu, B., Mathematical Olympiad Treasures. Birkhuser,
a
2004.
[5] Barbeau, E.J., Shawyer, B.L.R., Inequalities. A Taste of Mathematics,
vol. 4, 2000.
[6] Bulajich, R., Gmez Ortega, J.A., Geometr Cuadernos de Olimpiadas
o
a,
de Matemticas, Instituto de Matemticas, UNAM, 2002.
a
a
[7] Bulajich, R., Gmez Ortega, J.A., Geometr Ejercicios y Problemas, Cuao
a.
dernos de Olimpiadas de Matemticas, Instituto de Matemticas, UNAM,
a
a
2002.
[8] Courant, R., Robbins, H., Qu son las matemticas? Fondo de Cultura
e
a
Econmica, 2002.
o
[9] Coxeter, H., Greitzer, S., Geometry Revisited. New Math. Library, MAA,
1967.
[10] Dorrie, H., 100 Great Problems of Elementary Mathematics. Dover, 1965.

238

Bibliography

[11] Engel, A., Problem-Solving Strategies. Springer-Verlag, 1998.


[12] Fomin, D., Genkin, S., Itenberg, I., Mathematical Circles. Mathematical
World, Vol. 7. American Mathematical Society, 1996.
[13] Hardy, G.H., Littlewood, J.E., P`lya, G., Inequalities. Cambridge at the
o
University Press, 1967.
[14] Honsberger, R., Episodes in Nineteenth and Twentieth Century Euclidean
geometry. New Math. Library, MAA, 1995.
[15] Kazarino, N., Geometric Inequalities. New Math. Library, MAA. 1961.
[16] Larson, L., Problem-Solving Through Problems. Springer-Verlag, 1990.
[17] Mitrinovic, D., Elementary Inequalities. Noordho Ltd-Groningen, 1964.
[18] Niven, I., Maxima and Minima Without Calculus.
Expositions, MAA, 1981.

The Dolciani Math.

[19] Shariguin, I., Problemas de geometr Editorial Mir, 1989.


a.
[20] Soulami, T., Les Olympiades de Mathmatiques. Ellipses, 1999.
e
[21] Spivak, M., Calculus. Editorial Benjamin, 1967.


Indice
Concavidad
Funcin
o
interpretacin geomtrica, 30
o
e
Convexidad
Funcin
o
interpretacin geomtrica, 30
o
e
Desigualdad
util, 40

Cauchy-Schwarz, 18
a la Engel, 41
de Bernoulli, 36
de Cauchy-Schwarz, 41
forma de Engel, 41
de Euler, 78
de Hlder, 32
o
de Jensen, 25
de Leibniz, 81
de Minkowski, 32
de Nesbitt, 19, 43, 70, 75
de Ptolomeo, 62
de Schur, 36
de Tchebyshev, 21
del reacomodo, 15
del tringulo, 3
a
forma general, 3
Entre medias potenciales, 37
Hlder generalizada, 37
o

media armnica- media geomtrio


e
ca, 9
media cuadrtica-media aritmtia
e
ca, 23, 42
Media geomtrica-media aritmtie
e
ca
con pesos, 31
media geomtrica-media aritmtie
e
ca, 10, 55
Popoviciu, 38
Young, 31
Discrepancia, 54
Distancia entre dos puntos, 3
Funcin
o
cncava, 27
o
convexa, 24
cuadrtica, 5
a
Lema
de Viviani, 102, 105
Mayor que, 2
Media
aritmtica, 8, 10, 23, 36
e
armnica, 9, 23
o
cuadrtica, 23
a
geomtrica, 8, 10, 23, 36
e
Menor o igual que, 2


Indice

240
Menor que, 2
Problema
de Pompeiu, 62
Fagnano, 102, 108
Fermat-Steiner, 102
Hern, 106
o
con c
rculo, 107
Punto
de Fermat, 103, 105
Recta real, 1
Solucin
o
de Fejr
e
Problema de Fagnano, 111
de Hofmann-Gallai
Problema de Fermat-Steiner,
105
de Schwarz
Problema de Fagnano, 110
de Shwarz
Problema de Fagnano, 109
de Steiner
Problema de Fermat-Steiner,
106
Problema Fermat-Steiner, 108
de Torricelli
Problema de Fermat-Steiner,
102
Teorema
de Erds-Mordell, 9497, 101
o
de Euler, 77
de Leibniz, 80
de Muirhead, 51, 52
de Pappus, 93
Transformacin
o
de Ravi, 64, 85
Tringulo
a
o
rtico, 109, 112

pedal, 114
Valor absoluto, 3

También podría gustarte